Sie sind auf Seite 1von 77

AUDITING THEORY c.

Requisite skill and diligence


PRELIM REVIEWER d. Freedom from undue influence
(875 Multiple Choice Questions)
8. Users of financial statements demand independent audit
because
SET 1 a. Users demand assurance that fraud does not exist.
b. Management may not be objective in reporting.
1. Which of the following engagements is covered by the c. Users expect auditors to correct management errors.
Framework for Assurance Engagements? d. Management relies on the auditor to improve internal
a. Consulting engagements control.
b. Preparation of tax returns
c. Independent financial statements audit 9. An audit of financial statements is conducted to determine
d. Agreed-upon procedures if the
a. Organization is operating efficiently and effectively
2. Independent auditing can best be described as a b. Client's internal control is functioning as intended.
a. Subset or branch of accounting. c. Auditee is following specific procedures or rules set
b. Professional activity that measures and communicates down by some higher authority
financial accounting data. d. Overall financial statements are stated in
c. A discipline or professional activity that attests to accordance with an identified financial reporting
the fair presentation of financial statements. framework.
d. Regulatory activity that prevents the issuance of
improper financial information. 10. An audit designed to determine the extent to which the desired
results of an activity established by the legislative or other
3. An auditor should not render a report on: authorizing body are being achieved.
a. Client internal control. a. Economy audit
b. Management performance. b. Efficiency audit
c. The achievability of forecasts. c. Program results audit
d. Quarterly financial information. d. Financial-related audit

4. The assumption underlying an audit of financial statements 11. Which of the following methods is most commonly used to
is that they will be used by reduce information risk?
a. Different groups for different purposes. a. Allow users to verify information.
b. The general public in making investment decisions. b. Allow all users to prepare the statements.
c. The board of directors as basis of declaring cash c. Users share information risk with management.
dividends. d. Have the financial statements audited.
d. The regulatory agencies to verify information that is
relevant to their supervisory functions. 12. Which of the following best describes the primary purpose of
audit procedures?
5. On every audit engagement, the CPA should comply with a. To detect errors or fraud
applicable PSAs b. To comply with generally accounting principles
a. Without exception. c. To gather sufficient, appropriate evidence
b. Except in examinations that result in a qualified report. d. To verify the accuracy of account balances
c. Except in examinations of interim financial statements.
d. Except in circumstances where the CPA is associated 13. It is essential that users regard CPA firms as:
with unaudited financial statements. a. Competent.
b. Unbiased.
6. It refers to the attribute or quality of audit evidence. c. Technically proficient.
a. Sufficiency d. All of the above
b. Relevance
c. Reliability 14. Which of the following is least likely an application of
d. Appropriateness maintaining an attitude of professional skepticism?
a. The auditor is alert to audit evidence that contradicts or
7. Which of the following best describes due care? brings into question the reliability of documents or
a. Reasonable infallibility management representations.
b. Tact in avoiding legal liability b. In planning and performing an audit, the auditor

AT – Prelim Rev (875 MCQs) Red Sirug Page 1 of 77


assumes that management is dishonest. 21. No matter how competent a CPA may be, his opinion on
c. The auditor makes a critical assessment, with a financial statements will be of little value to those who rely on
questioning mind, of the validity of audit evidence him unless he
obtained. a. Issues an unqualified opinion
d. The auditor does not consider representations from b. Maintains his independence.
management as substitute for obtaining sufficient c. Maintains a program of continuing education
appropriate audit evidence to be able to draw reasonable d. Serves his clients with professional concern of their best
conclusions on which to base the audit opinion. interests.

15. Which of the following is not one of the concepts in the 22. An audit can have a significant effect on
framework of auditing theory? a. Business risk.
a. Evidence b. Information risk.
b. Ethical conduct c. The risk-free interest rate.
c. Fair presentation d. All of these.
d. Conflict of interest
23. In government auditing, the three elements of expanded
16. To raise the standards of the profession, the Philippine scope auditing are
Accountancy Act of 2004 requires that the CPA profession be a. Goal analysis, audits of operations, audit of systems.
integrated under the rules to be established by b. Financial and compliance, economy and efficiency,
a. The PICPA. program results.
b. The Board of Accountancy. c. Pre-audit, post-audit, internal audit.
c. The Professional Regulation Commission. d. National government audit, local government audit,
d. The Government Association of CPAs. corporation audit.

17. Management assertions are 24. Which of the following types of audits are most similar?
a. Stated in the footnotes to the financial statements. a. Operational audits and compliance audit.
b. Provided to the auditor in the assertions letter, but are not b. Internal audits and independent financial statement audits.
disclosed in the financial statements of the entity. c. Independent financial statement audits and operational
c. Implied or express representations about the audits.
accounts in the financial statements. d. Compliance audits and independent financial
d. Explicitly expressed representations about the financial statement audits.
statements.
25. If it is probable that the judgment of a reasonable person
18. In performing a financial statement audit, which of the would have been changed or influenced by the omission or
following would an auditor least likely consider? misstatement of information, then the information is
a. Internal control. a. Relevant.
b. Compliance with GAAP. b. Pervasive.
c. Fairness of the financial statement amounts. c. Material.
d. Quality of management’s business decisions. d. Significant.

19. When the CPA is not independent, the CPA is precluded from 26. Which of the following is not among the factors that result to
issuing a limitations of audit?
a. Review report. a. Use of testing
b. Compilation report. b. Human error
c. Tax planning report. c. Going concern problem of the assurance client
d. Management advisory report. d. Evidence is basically persuasive rather than conclusive

20. The risk that the auditor will fail to uncover a material 27. Which of the following services is not within the area of public
misstatement is eliminated accounting?
a. If client has effective internal control. a. Attest function
b. If client follows PFRS. b. Taxation service
c. When the auditor has complied with PSA. c. Write-up work for one client
d. Under no circumstances. d. Management advisory services

AT – Prelim Rev (875 MCQs) Red Sirug Page 2 of 77


28. An audit committee must be comprised of outside directors. b. They define the nature and extent of the auditor's
Which of the following is considered an outside director? responsibilities.
a. A consultant to the company. c. They provide guidance to the auditor with respect to
b. A member of company management. planning the audit and writing the audit report.
c. The company's independent auditor. d. They set forth a measure a measure of the quality of
d. A retired executive from another company. the performance of audit procedures.

29. "The auditor would ordinarily expect to find evidence to 35. Auditing includes both a(an)
support management representations and not assume that a. Evaluation process and a reporting process.
they are necessarily correct". This is an example of: b. Investigative process and a reporting process.
a. Due diligence. c. Documentation process and an evaluation process.
b. Unprofessional behavior. d. Documentation process and a reporting process.
c. An attitude of professional skepticism.
d. A rule in the code of Professional Conduct. 36. The audit committee of the board of directors of a company
is responsible for
30. The audit process is a. Hiring the auditor
a. Regulated by the PICPA b. The audit workpapers
b. Performed only by CPAs c. Preparing the financial statements
c. The only service a CPA is allowed to perform by law d. Independence and obtaining evidence
d. A special application of the scientific method of
inquiry 37. Which of the following types of audits would be intended to
determine whether a governmental entity is following sound
31. In determining the scope and nature of services to be procurement practices?
performed in public practice, a CPA firm should a. Program audit
a. Have in place internal control procedures b. Operational audit
b. Only perform accounting related services c. Compliance audit
c. Require independence for all services performed d. Financial statement audit
d. Determine that the performance of all services is
consistent with the firm's members' role as 38. Which of the following is the authoritative body designated to
professionals. promulgate auditing standards?
a. BOA
32. Which of the following is not an assurance that the auditors b. PICPA
give to the parties who rely on the financial statements? c. FRSC
a. Auditors give assurance that the financial statements d. AASC
are accurate.
b. Auditors know how the amounts and disclosures in the 39. Which of the following statements is true?
financial statements were produced. a. An independent auditor must be a CPA.
c. Auditors gathered enough evidence to provide a b. Compliance audits are conducted to determine
reasonable basis for forming an opinion. adherence to rules and regulations set by the auditor.
d. If the evidence allows the auditors to do so, auditors give c. An audit, if properly conducted, ensures that fraud is
assurance in the form of opinion, as to whether the prevented.
financial statements taken as a whole are fairly presented d. After conducting an audit and release of the auditor’s
in conformity with GAAP. report, the primary responsibility on the fairness of the
financial statements is shifted to the auditor.
33. Auditing standards differ from auditing procedures in that
procedures relate to 40. The Philippine Standards on Assurance Engagements
a. Audit judgments. (PSAEs) are to be applied in
b. Audit principles. a. The audit or review of historical financial information.
c. Acts to be performed. b. Assurance engagements dealing with historical financial
d. Measures of performance. information.
c. Compilation engagements and agreements to apply
34. Choose one of the following which would describe best the agreed-upon procedures to information.
phrase "generally accepted auditing standards": d. Assurance engagements dealing with subject matters
a. They identify the policies and procedures for the conduct other than historical financial information.
of the audit.

AT – Prelim Rev (875 MCQs) Red Sirug Page 3 of 77


41. The Philippine Standards on Quality Control (PSQCs) are to 48. Which of the following statements does not properly describe
be applied to an element of the theoretical framework of auditing?
a. Assurance engagements only. a. Remoteness of users.
b. Review engagements only. b. An audit benefits the public.
c. Compilation and review engagements only. c. The data to be audited are verifiable.
d. All services that fall under the AASC's engagement d. Auditor should maintain independence with respect to
standards. the audit client.

42. These statements are issued by the AASC to provide 49. The statement that the reviewer "is not aware of any material
interpretive guidance and practical assistance to auditors in modification that should be made to the financial statements in
the implementation of PSAs and to promote good practice. order for them to be in conformity with GAAP" is known as:
a. PREPSs a. Positive assurance.
b. PAEPs b. Reasonable assurance.
c. PAPSs c. Negative assurance.
d. PRSPSs d. Negligent performance.

43. For the purpose of expressing negative assurance in the 50. An independent audit aids in the communication of economic
review report, the practitioner should obtain sufficient data because the audit
appropriate evidence primarily through a. Lends credibility to the financial statements.
a. Inquiry and confirmation b. Confirms the accuracy of management's financial
b. Confirmation and tests of controls representation.
c. Inquiry and analytical procedures c. Guarantees that financial data are fairly presented.
d. Analytical procedures and substantive tests of details of d. Assures the readers of financial statements that any
transactions and account balances fraudulent activity has been corrected.

44. The BOA vice-chairman shall be 51. The purpose of an audit of financial statements is to
a. Appointed by the PRC a. Obtain an absolute level of assurance that the financial
b. Elected by BOA members statements as a whole are free from material
c. Appointed by the President of the Philippines misstatement.
d. Appointed by the PRC upon recommendation by the b. Relieve management or those charged with governance
PICPA of the responsibility for the preparation and presentation
of the financial statements in accordance with the
45. The term of BOA vice-chairman is applicable financial reporting framework.
a. One year. c. Enhance the degree of confidence of intended users in
b. Three years subject to renewal. the financial statements.
c. At the discretion of BOA members. d. Assure the future viability of the entity by expressing an
d. Same with the term of BOA chairman. opinion on the entity's financial statements.

46. An audit designed to evaluate the efficiency and effectiveness 52. Inherent limitations in an audit stem from the following factors
of an organization or some or part thereof would not come except
under the title of a. Use of testing.
a. Performance audit. b. Incompetence of the auditor.
b. Compliance audit. c. Internal control limitation.
c. Management audit. d. Most audit evidence is persuasive rather than conclusive.
d. Operational audit.
53. Auditing services may include which of the following?
47. In financial statement audits, the audit should be conducted a. Attesting to financial statements
in accordance with b. Evaluation of a division’s performance for management
a. PAS / PFRS c. Examination of the economy and efficiency of
b. Generally accepted auditing standards governmental operations
c. Philippine Standards on Auditing d. All of the above
d. Code of Ethics for CPAs in the Philippines
54. The review of unaudited financial statements consists of:
a. Inquiry of management and analytical procedures.

AT – Prelim Rev (875 MCQs) Red Sirug Page 4 of 77


b. Inquiry of management and documentation of internal 60. The expertise that distinguishes auditors from accountants is
controls. in the
c. Analytical procedures and compliance with laws and a. Ability to interpret PAS/PFRS.
regulations. b. Accumulation and interpretation of evidence.
d. Internal control evaluation and management c. Ability to interpret generally accepted accounting
representation. principles.
d. Requirement to possess education beyond the
55. The Philippine Standards on Auditing issued by AASC: bachelor's degree.
a. Apply to independent examination of financial
statements of any entity when such an examination is 61. The market for auditing services is driven by
conducted for the purpose of expressing an opinion. a. The regulatory authority of the Securities and Exchange
b. Need to be applied on all audit related. Commission.
c. Must not apply to other related activities of auditors. b. A demand by external users of financial statements.
d. Require that in no circumstances would an auditor may c. Pronouncements issued by the Auditing and Assurance
judge it necessary to depart from a PSA, even though Standards Council.
such a departure may result to more effective d. Congress.
achievement of the objective of an audit.
62. An independent audit is important to readers of financial
56. In the case of an audit of financial statements, which of statements because it
the following would not be a valid criterion in ascertaining a. Provides a measure of management's stewardship
the degree of correspondence between assertions and function
established criteria? b. Objectively examines and reports on management's
a. International Accounting Standards financial statements
b. Philippine Accounting Standards c. Reports on the accuracy of information in the financial
c. Philippine Standards on Auditing statements
d. Philippine Financial Reporting Standards d. Measures and communicates the financial data included
in financial statements
57. Which of the following statements about independent
financial statement audit is correct? 63. Independent auditing can best be described as
a. The audit of financial statements relieves management a. A branch of accounting.
of its responsibilities for die financial statements. b. A regulatory function that prevents the issuance of
b. An audit is designed to provide limited assurance that improper financial information.
the financial statements taken as a whole are free from c. A professional activity that measures and communi-
material misstatement. cates financial and business data.
c. The auditor's opinion is not an assurance as to d. A discipline which attests to the results of
the future viability of the entity as well as the accounting and other functional operations and data.
effectiveness and efficiency with which
management has conducted the affairs of the entity. 64. The primary purpose of an independent financial statement
d. The procedures required to conduct an audit in audit is to
accordance with PSAs should be determined by the a. Provide a basis for assessing management's
client who engaged the services of the auditor. performance
b. Comply with government regulatory requirements
58. The reason an independent auditor gathers evidence is to c. Assure management that the financial statements are
a. Detect fraud unbiased and free from material error
b. Evaluate management d. Provide users with an unbiased opinion about
c. Evaluate internal controls the fairness of information reported in the financial
d. Form an opinion on the financial statements statements

59. The auditor communicates the results of his or her work 65. The criteria for evaluating quantitative information vary. For
through the medium of the example, in the case of an independent audit of
a. Engagement letter. financial statements by CPA firms, the criteria are
b. Audit report. usually the:
c. Management letter. a. Philippine Standards on Auditing
d. Audited financial statements. b. Philippine Financial Reporting Standards
c. National Internal Revenue Code

AT – Prelim Rev (875 MCQs) Red Sirug Page 5 of 77


d. Securities and Exchange Commission Regulations c. Regulation within the firm, through the implementation of
a system of quality control.
66. A profession is distinguished by certain characteristics d. All of these.
including:
a. Mastery of a particular intellectual skill, acquired by 71. This body is created to assist the BOA in the attainment of the
training and education. objective to continually upgrade accounting education in the
b. Adherence by its members to a common code of values Philippines, thus making Filipino CPAs globally competitive.
and conduct established by its administrating body, a. PICPA CPE Council
including maintaining an outlook which is essentially b. Education Technical Council
objective. c. Accounting Teachers' Conference
c. Acceptance of a duty to society as a whole. d. Association of CPAs in Education
d. All of these.
72. Which of the following is not a requirement for a person
67. Professional judgment: applying for the CPA Board Examinations?
a. Is not used in making decisions about materiality and a. Good moral character
audit risk. b. Degree of BS Accountancy
b. Is necessary in the evaluation of management's c. Natural-born Filipino citizen
judgments in applying the entity's applicable d. No conviction of any criminal offense involving moral
financial reporting framework. turpitude.
c. Should be exercised in planning and performing an audit
of financial statements but need not be documented. 73. Its function is to conduct a quality review on applicants for
d. Can be used as the justification for the decisions made by registration to practice public accountancy and render a report
the auditor that are not supported by the facts and which shall be attached to the application for registration.
circumstances of the engagement. a. Quality Review Board
b. Quality Review Group
68. The auditor's judgment concerning the overall fairness of c. Board of Quality Reviewers
presentation of financial position, results of operation, and d. Quality Review Committee
changes in cash flow is applied within the framework of
a. Generally accepted auditing standards which include 74. Which of the following is the accredited national professional
the concept of materiality organization of CPAs?
b. Generally accepted accounting principles. a. IFAC
c. Philippine Financial Reporting Standards b. ASC
d. Quality control c. AASB
d. PICPA
69. One of the conditions that give rise to a demand for an
external audit of financial statements is expertise. Which 75. The ETC shall be composed of
of the following best describes the meaning of expertise as a. 7 members.
used in this context? b. 8 members.
a. Auditors usually rely on the work of an expert as c. 15 members
a basis for evaluating some assertions embodied in d. 14 members
the financial statements.
b. Users usually lack the necessary expertise to verify 76. Which of the following does not have representations in the
the reliability of the financial information. FRSC?
c. As experts, auditors are expected to detect all material a. PICPA
misstatements in the financial statements. b. BIR
d. The readers of the financial statements must possess c. BOI
the necessary expertise to be able to understand the d. BOA
financial statements.
77. A partner surviving the death or withdrawal of all the other
70. Regulation of the accounting profession include: partners in a partnership may continue to practice under the
a. Public Regulation as provided for in the Philippine partnership name for a period of not more than ___ years after
Accountancy Act of 2004. becoming a sole proprietor.
b. Regulation within the profession, through the a. 1
implementation of the Code of Ethics. b. 2
c. 3

AT – Prelim Rev (875 MCQs) Red Sirug Page 6 of 77


d. 4 82. Which of the following statements concerning a CPA's
disclosure of confidential client information is ordinarily correct?
78. The death, or disability of an individual CPA and/or the a. Disclosure should not be made even if such disclosure
dissolution and liquidation of a firm or partnership of CPAs will protect the CPA's professional interests in legal
shall be reported to the BOA not later than ___ days from the proceedings.
date of such death, dissolution or liquidation. b. Disclosure should be made only if there is a legal, or
a. 15 professional duty to make the disclosure.
b. 30 c. Disclosure may be made to any party on consent of
c. 60 the client.
d. 90 d. Disclosure may be made to any government agency
without subpoena. .
79. It refers to the inculcation, assimilation and acquisition of
knowledge, skills, proficiency and ethical and moral values, 83. This refers to those persons who hold a valid certificate issued
after the .initial registration of a professional that raise and by the Board of Accountancy.
enhance the professional's technical skills and competence. a. Senior accounting practitioner.
a. Professional Development b. Professional accountant.
b. Continuing Professional Education c. Professional accountant in public practice.
c. Continuing Professional Development d. Audit associate.
d. Professional Growth and Development
84. The principle of professional behavior requires a CPA to
80. The following statements relate to CPE credit units. Which is a. Be straightforward and honest in performing professional
incorrect? services.
a. One credit hour of CPE program, activity or source shall b. Perform professional services with due care, competence
be equivalent to one (1) credit unit. and diligence.
b. Any excess credit units in one year may be carried over c. Be fair and should not allow prejudice or bias, conflict of
to the succeeding years within the three-year period. interest or influence of others to override objectivity.
c. The total CPE credit units for registered accounting d. Act in a manner consistent with the good reputation
professionals shall be sixty (60) credit units for three (3) of the profession and refrain from any conduct which
years, provided that a minimum of fifteen (15) credit units might bring discredit to the profession.
shall be earned in each year.
d. Excess credit units earned may be carried over to the 85. This fundamental principle requires a CPA not to use or
next three-year period including credit units earned disclose information acquired during the course of performing
for doctoral and master's degrees. professional services without proper and specific authority.
a. Objectivity.
81. The following statements relate to the exemption from CPE b. Professional Behavior.
requirements. Which is false? c. Professional Competence and Due care.
a. A registered professional shall be permanently exempted d. Confidentiality.
from CPE requirements upon reaching the age of 65
years old. 86. Which of the following is not a ground for suspension or
b. A registered professional shall be permanently removal of members of BOA?
exempted from CPE requirements upon reaching the a. Negligence in the performance of duties, or lack of
age of 60 years old. professional competence.
c. A registered professional who is working or practicing b. Intolerance of violations of the Philippine
his/her profession abroad shall be temporarily exempted Accountancy Act.
from compliance with CPE requirements during the c. Final judgment of crimes involving moral turpitude.
period of his/her stay abroad provided that he/she has d. Rigging of the CPA licensure examination results.
been out of the country for at least two years
immediately prior to the date of renewal. 87. The Professional Regulatory Board of Accountancy consists of:
d. A registered professional who is furthering his/her studies a. 1 Chairman and 4 members.
abroad shall be temporarily exempted from compliance b. 1 Chairman and 5 members.
with CPE requirements during the period of his/her stay c. 1 Chairman and 6 members.
abroad provided that he/she has been out of the country d. 1 Chairman and 7 members.
for at least two years immediately prior to the date of
renewal. 88. The Chairman and the members of the BOA shall be
appointed by the:

AT – Prelim Rev (875 MCQs) Red Sirug Page 7 of 77


a. Commissioner of the SEC. d. The Professional Regulation Commission.
b. President of the PICPA.
c. President of the Republic of the Philippines. 95. A candidate successfully passes the Board exams if he/she
d. Retiring chairman of the Board of Accountancy. obtains
a. A general average of sixty-five percent (65%).
89. Members of the Professional Regulatory Board of b. A general average of seventy-five percent (75%).
Accountancy shall serve for: c. A general average of seventy-five percent (75%) in at
a. Three years, subject to renewal. least a majority of the subjects given in the Board exams.
b. Seven years, not subject to renewal. d. A general average of seventy-five percent (75%), with
c. Three years, subject to reappointment. no grades lower than sixty-five percent (65%) in any
d. Three years, not subject to reappointment. given subject.

90. Which of the following is not mentioned as a power of the 96. In the event a candidate obtains the rating of seventy-five
Board? percent (75%) and above in at least a majority of subjects as
a. To monitor the conditions affecting the practice of provided for in RA9298, and one subject has a rating of 64%,
accountancy. he shall be considered as:
b. To supervise the registration, licensure and practice of a. A failed candidate.
accountancy. b. A conditioned candidate.
c. To sentence to imprisonment CPAs who have been c. A successful passer of the CPA Board exams.
found guilty of violations of the Accountancy Act. d. Cannot be determined without additional information
d. To issue, suspend, revoke, or reinstate the Certificate of
Registration for the practice of the accountancy 97. A certificate under seal, bearing a registration number, issued
profession. to an individual, by the PRC, upon recommendation by the
BOA, signifying that the individual has complied with all the
91. Which of the following is the sectoral organization for CPAs in legal and procedural requirements for such issuance:
Public Practice? a. Certificate of accreditation
a. ACPACI b. Certificate of registration
b. ACPAPP c. Certificate of quality review
c. ACPAE d. Certificate of identification
d. GACPA
98. This bears the registration number, date of issuance with an
92. This is the standard-setting body that replaces the Accounting expiry date, due for periodic renewal, duly signed by the
Standards Council (ASC) and is tasked to develop and issue Chairperson of the PRC and issued by the PRC to a
standards which will represent generally accepted accounting registered CPA upon payment of the registration fees:
principles in the Philippines: a. Certificate of registration
a. Accounting Standards Board. b. Certificate of accreditation
b. Financial Reporting Standards Committee. c. Professional identification card
c. Financial Reporting Standards Council. d. Professional tax receipt
d. Auditing and Assurance Standards Council.
99. A professional identification card has a validity of:
93. This standard-setting body replaces the Auditing Standards a. Six years
and Practices Council (ASPC) and is tasked to develop and b. Five years
issue Philippine Standards on Auditing and related c. Four years
Interpretations: d. Three years
a. Accounting Standards Board.
b. Financial Reporting Standards Committee. 100. Foreign CPAs
c. Financial Reporting Standards Council. a. Are allowed to practice accountancy in the Philippines
d. Auditing and Assurance Standards Council. without restriction.
b. Are never allowed to practice accountancy in the
94. The Chairpersons of the FRSC and the AASC shall be Philippines.
appointed by: c. May be allowed to practice accountancy in the Philippines,
a. The Philippine Institute of Certified Public Accountants. but only in areas outside public practice.
b. The President of the Republic of the Philippines. d. May be allowed to practice accountancy in the
c. The Professional Regulatory Board of Accountancy. Philippines, subject to the rules and regulations on
reciprocity.

AT – Prelim Rev (875 MCQs) Red Sirug Page 8 of 77


a. The financial statements are client's responsibility.
SET 2 b. The audit was conducted in accordance with PSAs
c. Client is guilty of contributory negligence.
1. These are independent professional services that improve the d. The audit was conducted in accordance with generally
quality of information for decision-making. accepted accounting principles.
a. Audit services
b. Assurance services 8. The auditor considers materiality from:
c. Attestation services a. AASC perspective
d. Management consultancy b. Client perspective
c. Independence perspective
2. Which of the following types of audits is performed most d. Reasonable user perspective
frequently by CPAs on a fee basis and for more than one
client? 9. Which of the following is more difficult to evaluate
a. Government audits objectively?
b. Compliance audits a. Compliance with government regulations
c. Internal audits b. Efficiency and effectiveness of operations
d. Financial statement audits c. All of the above are equally difficult
d. Presentation of financial statements in accordance
3. The independent auditor lends credibility to client with the PFRS
financial statements by
a. Stating in the auditor’s management letter that the 10. The practitioner makes a critical assessment of validity of
examination was made in accordance with PSAs audit evidence, with a questioning mind and being alert to
b. Attaching an auditor’s opinion to the client’s conditions which may indicate possible misstatement due to
financial statements. error or fraud.
c. Testifying under oath about client financial statements. a. Objectivity and integrity
d. Maintaining clear-cut distinctions between b. Professional skepticism
management representations and auditor’s c. Independence is mental attitude
representations. d. Sufficient appropriate evidence

4. Internal auditing relates to an 11. In the auditing environment, failure to meet the PSAs is
a. Audit which is performed by a practitioner as an often
independent contractor. a. An accepted practice.
b. Audit which is incidentally concerned with the b. A suggestion of negligence.
detection and prevention of fraud. c. Conclusive evidence of negligence.
c. Audit which serves the needs of management d. Tantamount to criminal behavior.
d. Audit wherein the auditor should be independent of
management both in fact and in mental attitude 12. The following phrases relate to the definition of auditing.
Which one is incorrect?
5. Independent auditors cannot be totally independent a. Systematic process
a. Because they don’t audit financial statements b. Objectively obtaining and obtaining evidence
b. Since they do not possess the CPA license c. Assertions about economic actions and events
c. As long as employer-employee relationship exists d. Degree of correspondence between assertions
d. Unless their immediate supervisor is a CPA and GAAS

6. The Philippine Standards on Auditing issued by the Auditing 13. Non-assurance engagements include all of the following
and Assurance Standards Council (AASC) except
a. Must be followed in all situations. a. Compliance audit.
b. Are interpretations of generally accepted auditing b. Management consulting.
standards c. Agreed-upon procedures.
c. Are optional guidelines which an auditor may choose to d. Preparation of tax returns where no conclusion is
follow or not follow when conducting an audit. expressed.
d. Are the equivalent of laws for audit practitioners.
14. The subject matter of an assurance engagement can take the
7. The auditor's best defense when material misstatements in following forms except
the financial statements are not uncovered in the audit is that a. The entity’s internal control.

AT – Prelim Rev (875 MCQs) Red Sirug Page 9 of 77


b. Historical or prospective financial statements. c. The inherent limitations of control systems.
c. Evaluation of a capital investment proposal. d. All of the above
d. Performance of an entity that could indicate efficiency
and effectiveness. 22. If the auditor learns that an entity is inappropriately using the
auditor’s name in association with financial information, he/she
15. In performing an assurance engagement, a professional should:
accountant typically a. Remain silent.
a. Provides management consulting advice. b. Seek legal advice if necessary.
b. Supplies litigation support services. c. Require management to cease doing so.
c. Assesses control risk at a low level. d. Inform any known third party users of the inappropriate
d. Expresses a conclusion about an assertion. use of his/her name in connection with the information.

16. Which of the following is not an assurance engagement? 23. A CPA should not submit unaudited financial statements of a
a. Risk Assessment Service non-public entity to a client or others unless, as a minimum,
b. Information System Reliability Service the CPA complies with the provisions applicable to
c. Business Performance Measurement a. Audit engagements.
d. Management Consulting Service b. Review engagements.
c. Assurance engagements.
17. The three commonly-sought types of assurance services are d. Compilation engagements.
a. Audits, reviews, and compilations.
b. Audits, reviews, and other assurance services. 24. You own Dude, Inc., which manufactures wooden tables. You
c. Audits, compilations, and other assurance services. need to hire some accountants to prepare your monthly
d. Reviews, compilations, and other assurance services. financial statements. The preparation of your financial
statements is referred to as a(n)
18. Which of the following statements is true? a. Audit.
a. Assurance engagements do not require independence. b. Review.
b. The term “auditor” is broader in scope compared to the c. Compilation.
term “practitioner.” d. Special report.
c. Assurance engagements performed by professional
accountants are intended to enhance the credibility 25. Distribution of a report is always restricted when
of information. a. Negative assurance is given.
d. The degree or level of assurance that may be provided by b. A review has been performed.
the practitioner is inversely related to the scope of c. There is a positive expression of opinion.
procedures performed and their results. d. Agreed-upon procedures have been performed.

19. Which of the following forms may be the subject matter of 26. Which of the following methods is most commonly used to
assurance engagements? reduce information risk?
a. Behavior a. Allow all users to prepare the statements.
b. Historical financial information b. Allow users to verify information.
c. Systems and processes c. Have the financial statements audited.
d. All of the above d. Users share information risk with management.

20. Engagement risk is influenced by the risks associated with the 27. The phrase in our opinion in the auditor’s report is intended to
following except inform users that auditors
a. Nature and form of the subject matter. a. Guarantee fair presentation of the financial statements.
b. Nature and form of the criteria applied to the subject b. Act as insurers of the accuracy of the financial statements.
matter. c. Certify the material presented in the statements by
c. Unreasonably low professional fee. management.
d. Nature and extent of the process used to collect and d. Based their conclusions about the statements on
evaluate evidence. professional judgment.

21. Absolute assurance is not attainable as a result of such 28. The need for assurance services arises because:
factors as: a. There is a potential bias in providing information.
a. The use of judgment. b. Economic transactions are less complex than they were a
b. The use of selective testing. decade ago.

AT – Prelim Rev (875 MCQs) Red Sirug Page 10 of 77


c. There is a consonance of interests of the preparer and b. Fine or imprisonment
the user of the financial statements. c. Revocation of CPA certificate
d. Most users today have access to the system that d. Suspension of CPA certificate
generates the financial statements they use.
36. Which of the following is an incorrect statement relating to the
29. This tax service includes the determination of the tax theoretical framework of auditing?
consequences of planned or potential transactions, followed a. Effective internal control structure reduces the probability
by making suggestions on the most desirable course of action of fraud or irregularities in an organization.
such as to legally minimize the tax liability while achieving the b. Application of generally accepted accounting principles
client’s objectives. results in a fair presentation of financial statements.
a. Tax shelter c. When examining financial data for the purpose of
b. Tax evasion expressing an independent opinion thereon, the auditor
c. Tax planning acts exclusively in the capacity of an auditor.
d. Tax compliance d. In collecting evidence, auditors should maintain an
attitude of trust about their clients' assertions.
30. Which of the following characteristics is not considered
necessary in determining whether the criteria are suitable? 37. The market for auditing services is driven by
a. Reliability a. Congress at the state level
b. Relevance b. A demand by external users of financial statements
c. Neutrality c. The regulatory authority of the Securities and Exchange
d. Sufficiency Commission
d. Pronouncements issued by the Auditing Standards and
31. In an assurance engagement, this refers to the information Practices Council
obtained by the practitioner in arriving at the conclusions on
which the conclusion is based. 38. Which of the following statements reflects an auditor's
a. Criteria responsibility for detecting fraud and error?
b. Assertions a. An auditor is responsible for detecting employee errors
c. Evidence and simple fraud, but not for discovering fraud involving
d. Generally accepted auditing standards employee collusion or management override.
b. An auditor should design the audit to provide
32. Compilation is an example of which one of the following types reasonable assurance of detecting errors and fraud
of services? that are material to the financial statements.
a. Auditing c. An auditor should plan the audit to detect errors and fraud
b. Review that are caused by departures from the applicable FRF.
c. Consulting d. An auditor is not responsible for detecting errors and
d. Accounting fraud unless the application of PSAs and PAPs would
result in such detection.
33. The Framework of Philippine Standards on Auditing
distinguishes audits from related services. Related services 39. The adequacy of disclosures in the financial statements and
include all of the following, except footnotes is the primary responsibility of the:
a. Reviews. a. Client
b. Compilations. b. Auditor in charge of field work
c. Agreed-upon procedures. c. Partner assigned to engagement
d. Management consulting. d. Staff member who drafted the statements

34. Assurance services involve which of the following? 40. Generally accepted auditing standards do not require auditors
a. Relevance as well as reliability. of financial statements to
b. Nonfinancial information as well as traditional financial a. Assess the risk of occurrence of errors and fraud
statements. b. Design audits to provide reasonable assurance of
c. Electronic databases as well as printed reports. detecting errors and frauds
d. All of the above. c. Report all the findings of errors and fraud to police
authorities
35. Which of the following is not one of the penalties that can be d. Understand the nature or errors and fraud
imposed by the BOA?
a. Reprimand

AT – Prelim Rev (875 MCQs) Red Sirug Page 11 of 77


41. The risk that the auditor's own work will lead to the decision b. A person may serve the BOA for not more than twelve
that material misstatements do not exist in the financial years.
statements, when in fact such misstatements do exist, is c. A member of the BOA may continuously serve office
a. Control risk for more than nine years.
b. Detection risk d. No person who has served two (2) successive complete
c. Inherent risk terms shall be eligible for reappointment until the lapse of
d. Audit risk one (1) year.

42. With respect to the concept of materiality, which one of the 47. The BOA shall submit to the PRC the ratings obtained by each
following statements is correct? candidate within how many calendar days after the
a. Materiality is a matter of professional judgment. examination?
b. Materiality is determined by reference to AASC's a. 5 days
guidelines. b. 10 days
c. Materiality depends on the nature of a transaction rather c. 15 days
than the peso amount of the transaction. d. 30 days
d. Materiality depends only on the peso amount of an item
relative to other items in the financial statements. 48. Which of the following is not one of the grounds for
proceedings against a CPA?
43. The following statements relate to the Board of Accountancy. a. Insanity.
Which statement is correct? b. Immoral or dishonorable conduct.
a. The Board consists of a Chairman and six members. c. Gross negligence or incompetence in the practice of his
b. The chairman and members are appointed by the profession.
President of the Philippines upon recommendation of d. Engaging in public practice while being employed in
PICPA. a private enterprise.
c. Majority of the board members shall as much as possible
be in public practice. 49. The primary duty to enforce the provisions of RA 9298 and its
d. The Professional Regulation Commission may remove IRR rests with
from the Board any member whose certificate to practice a. The PRC
has been removed or suspended. b. The BOA
c. The AASC
44. The APO shall submit its nominations with complete d. The PRC and BOA
documentation to the Commission not later than ______ prior
to the expiry of the term of an incumbent chairman or member. 50. The PICPA shall renew its Certificate of Accreditation once
a. 30 days every how many years after the date of the Resolution
b. 60 days granting the petition for re-accreditation and the issuance of
c. 90 days the said certificate upon submission of the requirements?
d. 120 days a. 2 years
b. 3 years
45. A member of the BOA shall, at the time of his/her appointment, c. 4 years
possess the following qualifications, except d. 6 years
a. Must be a natural-born citizen and resident of the
Philippines. 51. The practice of accountancy includes the following except:
b. Must not be a director or officer of the APO at the time of a. Serving as audit examiner for the Commission on Audit.
his/her appointment. b. Working as a Dean of a College that grants a degree of
c. Must be of good moral character and must not have been BS Accountancy.
convicted of crimes involving moral turpitude. c. Being appointed as the marketing manager of a
d. Must be a duly registered CPA with more than ten (10) business enterprise.
years of work experience in any scope of practice of d. Provision of assurance services to more than one client
accountancy. and on a fee basis.

46. Which statement is incorrect regarding the term of office of the 52. Which of the following is one of the functions of Quality
chairman and the members of the Board of Accountancy Review Committee:
(BOA)? a. To promulgate accounting and auditing standards that will
a. The Chairman and members of the Board shall hold be generally accepted in the Philippines.
office for a term of three years. b. To conduct a review on applicants for registration to

AT – Prelim Rev (875 MCQs) Red Sirug Page 12 of 77


practice public accountancy and render a report a. The attestation standards do not permit an attest
which shall be attached to the application for engagement to be part of a business acquisition study or
registration. a feasibility study.
c. To adopt a Code of Ethics for the practice of accountancy. b. The attestation standards provide a framework for
d. To evaluate periodically the performance of educational the attest function beyond historical financial
institutions offering accountancy education. statements.
c. The requirement that the practitioner be independent in
53. No matter competent a CPA may be, his opinion on mental attitude is omitted from the attestation standard.
financial statements will be of little value to those who rely d. None of the standards of fieldwork in generally accepted
upon him unless he: auditing standards are included in the attestation
a. Maintains his independence. standards.
b. Maintains professional competence.
c. Maintains a program of continuing education. 59. The exercise of due professional care requires that an auditor
d. Serves his clients with professional concern. a. Uses error-free judgment.
b. Considers internal control, including tests of controls.
54. Which of the following statements about independent financial c. Be responsible for fulfilling his or her duties
statement audit is incorrect? diligently and carefully.
a. The term "scope of the audit" refers to audit procedures d. Examines all corroborating evidence available.
deemed necessary in the circumstances to achieve the
objective of the audit. 60. The objectives of the Philippine accountancy Act of 2004 are
b. The auditor's opinion enhances the credibility of the the following except:
financial statements. a. The standardization and regulation of accounting
c. The phrase used to express the auditor's opinion is education.
"present fairly, in all material respects". b. The examination for registration of certified public
d. The risk that the auditor will fail to uncover material accountants.
misstatement is eliminated when the auditor c. The integration of the accounting profession.
conducts the audit in accordance with PSAs. d. The supervision, control, and regulation of the practice of
accountancy in the Philippines.
55. For what reason does an independent auditor gather
evidence? 61. A CPA is in public accounting practice when he/she
a. To assess management performance a. Holds, or is appointed to, in an accounting professional
b. To detect fraud group in government or in a government-owned and/or
c. To detect misstatements controlled corporation where decision making requires
d. To form an opinion on the financial statements professional knowledge in the science of accounting.
b. Represents his/her employer before government
56. In conducting an appraisal of the economy and efficiency with agencies on tax and other matters related to accounting.
which company resources are used, an internal auditor's c. Represents his/her clients before government
responsibility is to: agencies on tax and other matters related to
a. Verify the accuracy of asset valuation. accounting.
b. Verify the existence of assets. d. Teaches accounting, auditing, management advisory
c. Determine whether operating standards have been services, accounting aspect of finance, business law,
established. taxation, and other technically related subjects.
d. Review the reliability of operating information.
62. The underlying reason for a code of professional conduct for
57. Because an external auditor is paid a fee by a client any profession is
company, he a. That it is required by Congress.
a. Is absolutely independent and may conduct an audit. b. That it provides a safeguard to keep unscrupulous people
b. May be sufficiently independent to conduct an audit. out.
c. Is never considered to be independent. c. The need for public confidence in the quality of
d. Must receive approval of the Securities and Exchange service of the profession.
Commission before conducting an audit. d. That it allows Professional Regulation Commission to
have a yardstick to measure deficient performance.
58. Which of the following is a conceptual difference between the
attestation standards and generally accepted auditing 63. If a firm, or a network firm, has a direct financial interest in an
standards? audit client of the firm, the self-interest threat created would be

AT – Prelim Rev (875 MCQs) Red Sirug Page 13 of 77


so significant no safeguard could reduce the threat to an 69. CPAs should not be liable to any party if they perform their
acceptable level. The action appropriate to permit the firm to services with:
perform the engagement would be to a. Ordinary negligence.
a. Dispose of the financial interest. b. Regulatory providence.
b. Dispose of a sufficient amount of it so that the remaining c. Due professional care.
interest is no longer material. d. Good faith.
c. Either a or b
d. Neither a nor b 70. Under the Code of Ethics, a CPA may not:
a. Perform bookkeeping services for an audit client.
64. Which of the following statements is incorrect? b. Perform advisory services for an audit client.
a. CPAs lose their independence if they acquire any direct c. Have any direct financial interest in a client during
financial interest in a client. the period covered by the financial statements.
b. CPAs lose their independence if they acquire any d. Having any joint, closely held investments with a
indirect financial interest in a client. principal stockholder of an audit client during the
c. CPAs lose their independence if they acquire a material period of audit engagement which is not material in
indirect financial interest in a client. relation to his net worth.
d. CPAs lose their independence if they have a material
direct financial interest in a client. 71. The Code of Ethics would be violated if a CPA accepted a
fee for services and the fee was:
65. A violation of the ethical standards would most likely have a. Fixed by a public authority.
occurred when a CPA b. Based on a price quotation submitted in competitive
a. Made arrangement with a bank to collect notes issued by bidding.
a client in payment of fees due. c. Based on the results of judicial proceedings in a tax
b. Joined an accounting firm made up of three non-CPA matter.
practitioners. d. Payable after a specified finding was attained in a
c. Issued an unqualified opinion on the 2010 financial review of financial statements.
statements when fees for the 2009 audit were unpaid.
d. Purchased a bookkeeping firm's practice of monthly write- 72. The CPA shall not concurrently engage in any business or
ups for a percentage of fees received over a three-year occupation which impairs his objectivity in rendering
period. professional services or which is inconsistent with his
practice or employment. This provision of the code of
66. The confidential relationship applies to ethics is applicable to:
a. All services provided by CPAs. a. Only to CPAs engaged in public accountancy.
b. Only audit and attestation services. b. CPAs engaged in public accountancy or employed in
c. Audit and tax services, but no MAS services. a private enterprise.
d. Audit and MAS services, but not tax services. c. CPAs engaged in public accountancy or employed in
a private enterprise or government agency.
67. A CPA’s retention of records as a means of enforcing d. All CPAs engaged in public accountancy, or
payment of an overdue audit fee is an act that is: employed in a private enterprise, a government
a. A violation of generally accepted auditing standards. agency or an educational institution.
b. Considered acceptable by the Code of Ethics.
c. Ill advised since it would impair the CPA’s 73. Ultimately, the decision as to whether the CPA is
independence with respect to the client. independent or not, will be made by the:
d. Considered discreditable to the profession. a. Audit Committee
b. Client
68. Which of the following acts of the CPA would be most c. Public
likely a violation of the Code of Ethics? d. Auditor
a. Assisting a client in preparing a financial forecast.
b. Acting as auditor of a non-charitable organization with 74. In which of the following instances would the
audit client serving as president. independence of the CPA not be considered to be
c. Accepting a fee in a tax matter that is contingent upon impaired? The CPA has been retained as the auditor of a
the result of an administrative proceeding. brokerage firm
d. Having an immaterial loan to the president of an a. Which owes the CPA audit fees for more than one
audit. year.
b. In which the CPA has a large active margin account.

AT – Prelim Rev (875 MCQs) Red Sirug Page 14 of 77


c. In which the CPA’s brother is the controller.
d. Which owes the CPA audit fees for the current 80. A practicing CPA is allowed by the CPA Code of Ethics to
year services and just filed a petition for do the following except:
bankruptcy. a. Announce the change in office location in a
newspaper.
75. A CPA, while performing an audit, strives to achieve b. List his firm name in the building lobby directory in
independence in appearance in order to: good taste and modest size.
a. Reduce risk and liability. c. Include his tax account number and membership in
b. Become independent in fact. PICPA on his stationary.
c. Comply with GAAS. d. List his office telephones in the PLDT directory to
d. Maintain public confidence in the profession. box or bold type.

76. Contingent fees charged by CPAs engaged in tax practice 81. Without the consent of the client, a CPA should not
are permitted under the rules of professional conduct disclose confidential information contained in working
because: papers to a:
a. This practice establishes fees which are a. Voluntary quality control review board.
commensurate with the value of the services. b. CPA firm that has purchased the CPA’s
b. Attorneys in tax practice customarily set contingent accounting practice.
fees. c. National court that has issued a valid subpoena.
c. Determination by taxing authorities are a matter of d. Disciplinary body created under statute.
judicial proceedings which do not involve third parties.
d. The consequences are based upon findings of 82. A CPA who is engaged to prepare an income tax return
judicial proceedings or the findings of tax has a duty to prepare it in such a manner that the tax is:
authorities. a. The legal minimum.
b. Computed in conformity with generally accepted
77. Ethically, the auditor could: accounting principles.
a. Advertise only as to his expertise in preparing income c. Supported by the client’s audited financial statements.
tax returns. d. Not subject to change upon audit.
b. Base his audit fee on a percentage of the proceeds of
his client’s stock issue. 83. The principle of Professional behavior requires a CPA to:
c. Perform an examination for a financially a. Perform professional services with due care,
distressed client at less than his customary fees. competence and diligence.
d. Own preferred stock in a corporation which is an audit b. Be straightforward and honest in performing
client. professional services.
c. Act in a manner consistent with the good reputation
78. The Code of Ethics contains both general ethical of the profession and refrain from any conduct
principles that are aspirational in character and also a: which might bring discredit to the profession.
a. List of violations that would cause the automatic d. Be fair and should not allow prejudice or bias, conflict of
suspension of the CPA’s license. interest or influence of others to override objectivity or
b. Description of the CPA’s procedures for responding to affect his professional or business judgments.
an inquiry from trial board.
c. Set of specific, mandatory rules describing levels 84. The communication to the public of facts about a CPA which
of conduct the CPA must observe. are not designed for the deliberate promotion of that CPA.
d. List of specific crimes that would be considered as a. Advertising
acts discreditable to the profession. b. Solicitation
c. Indirect promotion
79. According to the Code of Ethics, the characteristics d. Publicity
distinguishing the accountancy profession include the
following except: 85. Which of the following does not belong to the group:
a. To meet public interest requirement. a. Independence of mind
b. Adherence by its members to a common code of values b. Independence of fact
and conduct established by its administering body. c. Independence in mental attitude
c. Acceptance of a duty to society as a whole. d. Independence in appearance
d. Mastery of a particular intellectual skill, acquired by
training and education. 86. Safeguards that may eliminate or reduce threats to

AT – Prelim Rev (875 MCQs) Red Sirug Page 15 of 77


independence to an acceptable level fall in the following d. A CPA running for public office uses the professional
categories except: designation "CPA" after his name on posters employed in
a. Safeguards created by the profession, legislation or connection with his election campaign.
regulation
b. Firm-wide safeguards 92. A CPA in public accounting is prohibited from performing
c. Engagement specific safeguards which of the following actions?
d. Safeguards created by the client. a. Permit the publication of his being the author of a book.
b. Be a party to a stratagem which permits a non-CPA to
87. A CPA firm would be reasonable assured of meeting its practice accountancy.
responsibility to provide professional services that conform c. Allow his wife to acquire shares in a corporation's capital
with professional standards by: stock.
a. Adherence to GAAS d. Act as a stock and transfer agent.
b. Maintaining an attitude of independence in its
engagements. 93. A CPA who has been retained by a client that operates in an
c. Having an appropriate a system of quality control. industry totally new to him
d. Joining professional societies that enforce ethical a. May not accept such an engagement.
standards. b. May accept the engagement only if the accounting firm
specializes in the audit of commercial banks.
88. The Philippine Code of Ethics requires that lead engagement c. May accept the engagement after attaining a suitable
partners of listed entities be rotated at least once every: level of understanding of the transactions and
a. 2 years accounting practices unique to commercial banking.
b. 3 years d. May accept the engagement because training as a CPA
c. 5 years transcends unique industry characteristics.
d. 7 years
94. Which of the following is not an objective of requiring
89. The following are represented both to the Financial Reporting registration of individual CPAs and Firms of partnership of
Standards Council (FRSC) and Auditing and Assurance CPAs engaged in public accounting practice?
Standards Council (AASC), except: a. The required registration will give equal opportunities
a. Bangko Sentral ng Pilipinas to CPAs in the practice of their profession.
b. Securities and Exchange Commission b. It will enable the Board of Accountancy to formulate and
c. Bureau of Internal Revenue implement rules and regulations more effectively for the
d. Board of Accountancy enhancement and maintenance of high professional,
ethical, and technical standards of the accounting
90. Which of the following auditor concerns most likely could be profession.
so serious that the auditor concludes that a financial statement c. To protect the public against fraud, deception, unethical
audit cannot be conducted? practices and from the consequences of ignorance,
a. The entity has no formal written code of conduct incompetence and incapacity in the practice of public
b. The integrity of the entity’s management is suspect accounting.
c. Procedures requiring segregation of duties are subject to d. It will help PRC and BOA to identify and impose
management override corresponding sanctions and penalties on individual
d. Management fails to modify prescribed controls for CPAs.
changes in conditions
95. As used in Republic Act No. 9298, this term refers to the
91. Which of the following is a violation of the code of professional area of practice of accountancy
ethics for certified public accountants? a. Section
a. A CPA permits his/her name to be used in a client's b. Sector
advertising as having verified financial data and/or c. Segment
statistical facts with respect to the client's products. d. Line
b. Based on information obtained in an audit, a CPA
reports an illegal act of his client to government 96. Which of the following does not constitute a practice of
authorities. accountancy?
c. Three years after a partner has retired, the remaining a. A person holding out himself as one skilled in the
partners continue to practice under a firm name that knowledge, science and practice of accounting and as a
includes the name of the retired partner. The retired qualified person to render professional services as a CPA
partner has severed all connections with the CPA firm. to more than one client.

AT – Prelim Rev (875 MCQs) Red Sirug Page 16 of 77


b. A person representing his/her employer before 2. The level of assurance provided by an audit of detecting a
government agencies on tax arid other accounting related material misstatement is referred to as:
matters. a. Positive assurance.
c. A person in educational institution teaching accounting, b. High assurance.
auditing, business law, taxation or other technically c. Reasonable assurance.
related subjects. d. Negative assurance.
d. A person is appointed as a marketing director of a
government owned and controlled corporation. 3. Of the following, which is the broadest concept?
a. Internal control audit.
97. The criteria for evaluating quantitative information vary. For b. Audit of financial statements.
example, in the audit of historical financial statements by CPA c. Attestation services.
firms, the criteria are usually: d. Assurance services.
a. Generally accepted auditing standards.
b. Acceptable financial reporting framework 4. In performing attestation services, a CPA will normally:
c. Regulations of the Securities and Exchange Commission a. Recommend uses for information.
d. Regulations of the Bureau of Internal Revenue b. Improve the quality of information, or its context, for
decision makers.
98. The firm is to be staffed by personnel who have attained and c. Perform market analyses and cost estimates.
maintained the technical standards and professional d. States a conclusion about a written assertion.
competence required to enable them to fulfill their
responsibilities with due care is the objective of what quality 5. Which of the following is not one of the requirements before
control policy? accepting an assurance engagement?
a. Ethical requirements a. The practitioner should be competent and independent.
b. Leadership responsibilities b. The responsible party and the intended user of
c. Assignment assurance report should be from different
d. Monitoring organizations.
c. The practitioner should accept the engagement only if the
99. In pursuing a CPA firm's quality control objectives, a CPA firm subject matter is the responsibility of another party
may maintain records indicating which partners or employees d. The practitioner should accept the engagement only if the
of the CPA firm were previously employed by the CPA firm's subject matter is identifiable and in the form that can be
clients. Which quality control element would this be most subjected to evidence gathering procedures.
likely to satisfy?
a. Monitoring. 6. The following statements relate to the three parties involved in
b. Human resources. an assurance engagement. Which is correct?
c. Ethical requirements. a. The responsible party and the intended users should
d. Engagement performance. always be from different entities.
b. A practitioner should not accept an assurance en-
100. An engagement quality control review is required to be gagement when the subject matter of the engagement
performed: requires specialized skills and knowledge beyond those
a. Immediately after the re-assessment of control risk. ordinarily possessed by the practitioner.
b. At engagement completion after the report is issued. c. A responsible party is the person who is responsible
c. For all audits of financial statements of listed entities. for the subject matter or the subject matter
d. For all types of audits, regardless of the subject matter of information.
the engagement. d. In all assurance engagements, the responsible party is
the engaging party, i.e., the party that engages the
practitioner
SET 3
7. The assurance report is often addressed to the intended users
1. The degree of certainty that the practitioner has attained and which may be:
wishes to convey is: a. The party responsible for the subject matter.
a. A conveyance. b. Established by agreement between the professional
b. An assertion. accountant and the responsible party.
c. A declaration. c. Both A and B
d. An assurance. d. Neither A nor B

AT – Prelim Rev (875 MCQs) Red Sirug Page 17 of 77


8. When performing an assurance service, professional d. Assist the client in the preparation of the financial
accountants use standards or benchmarks to evaluate or statements.
measure the subject matter of an assurance engagement.
This is known as: 14. When providing consulting services, the CPA acts primarily as
a. PFRS. a(n):
b. Assertion. a. Independent practitioner.
c. Criteria. b. Expert on compliance with industry standards.
d. Conclusion. c. Objective advisor on the use of information.
d. Confidential reviewer.
9. Which of the following services provides positive assurance
through attestation? 15. Which of the following statements concerning consulting
a. Tax services services is incorrect?
b. Review a. The performance of consulting services for audit clients
c. Auditing does not, in and of itself, impair the auditor's
d. Accounting services independence.
b. Consulting services differ fundamentally from the CPA's
10. Positive assurance is expressed through: function of attesting to the assertions of other parties.
a. Attestation c. Consulting services ordinarily involve external
b. Declaration reporting.
c. Conclusion d. Most CPAs, including those who provide audit and tax
d. Stating whether anything has come to the auditor's services, also provide consulting services to their clients.
attention that indicates that the financial statements are
not presented fairly. 16. In an engagement to perform agreed-upon procedures, an
auditor is engaged to:
11. Which of the following statements best describes review a. Use accounting expertise as opposed to auditing
services? expertise to collect, classify, and summarize financial
a. Review engagements focus on providing assurance on information.
the internal controls of a public company. b. Provide a moderate level of assurance that the
b. Review engagements focus on providing assurance on information is free of material misstatement.
the assertions contained in the financial statements of a c. Provide a high, but not absolute, level of assurance that
public company. the information is free of material misstatement.
c. Review engagements focus on providing advice in a d. Carry out those procedures of an audit nature to
three-party contract. which the auditor and the entity and any appropriate
d. Review engagements focus on providing limited third parties have agreed and to report on factual
assurance on financial statements of a private findings.
company.
17. Assurance engagements should exhibit the following elements
12. A review of a company's financial statements by a CPA firm: except
a. Is similar in scope to an audit and adds similar credibility a. A subject matter
to the statements. b. Suitable criteria
b. Is significantly less in scope than an audit and results in a c. Evidence
report which provides positive assurance, although not d. Appropriate professional fees
absolute assurance.
c. Concludes with the issuance of a report expressing the 18. Which of the following is not an element of assurance
CPA's opinion as to the fairness of the financial engagements?
statements. a. Subject matter
d. Is designed to provide only moderate assurance. b. Evidence
c. Engagement process
13. The objective of a review of financial statements is to: d. Suitable criteria
a. Express an opinion on the overall financial statements.
b. Carry out audit procedures agreed on with the client and 19. Assurance services involve which of the following?
other users of report. a. Relevance as well as reliability.
c. State whether anything has come to the auditor's b. Nonfinancial information as well as traditional financial
attention that indicates that the financial statements statements.
are not presented fairly. c. Electronic databases as well as printed reports.

AT – Prelim Rev (875 MCQs) Red Sirug Page 18 of 77


d. All of the above. attainable or cost beneficial as a result of the following factors,
except
20. Assurance engagement include the following, except a. The use of selective testing.
a. An engagement conducted to provide a high level of b. The fact that much of the evidence available to the
assurance that the subject matter conforms in all material practitioner is persuasive rather than conclusive.
respects with identified suitable criteria. c. The practitioner may not have the required assurance
b. An engagement conducted to provide a moderate level of knowledge and skills to gather and evaluate evidence.
assurance that the subject matter is plausible in the d. The use of judgment in gathering and evaluating
circumstances. evidence and forming conclusions based on that evi-
c. An engagement in accordance with the Philippine dence.
Standard on Assurance Engagement(s).
d. An engagement to perform agreed-upon procedures. 26. Which of the following standards are to be applied, as
appropriate, in the audit of historical financial information?
21. The subject matter of an assurance engagement may include a. PSAEs
I. Historical or prospective financial information b. PSREs
II. Internal controls c. PSAs
III. Compliance with regulation d. PSRSs
a. I and II only
b. I and III only 27. Which of the following standards are to be applied to
c. II and III only compilation engagements and engagements to perform
d. I, II, and III agreed-upon procedures to information?
a. PSRSs
22. In assertion-based assurance engagements, the evaluation or b. PSAs
measurement of the subject matter against criteria is c. PSAEs
performed by the d. PSREs
a. Intended users
b. Responsible party 28. The Philippine Standards on Review Engagements (PSREs)
c. Practitioner are to be applied in
d. AASC a. The audit of historical financial information.
b. Assurance engagements dealing with subject matters
23. Which of the following statements is true concerning evidence other than historical financial information.
in an assurance engagement? c. The review of both historical and prospective financial
a. The reliability of evidence is influenced not by its nature information.
but by its source. d. The review of historical financial information.
b. Sufficiency is the measure of the quantity of evidence.
c. Obtaining more evidence may compensate for its poor 29. PSRE 2400 (Engagements to Review Financial Statements),
quality. as amended by the AASC in February 2008, applies to
d. Appropriateness is the measure of the quality of evidence, a. Reviews of any historical financial information of an audit
that is, its reliability and persuasiveness. client.
b. Reviews of historical financial or other information by a
24. An unqualified conclusion is not appropriate for either practitioner other than the entity's auditor.
reasonable or limited assurance engagement when: c. Reviews of historical financial or other information of an
I. Circumstances prevent the practitioner from obtaining audit client.
evidence required to reduce assurance engagement risk to d. Reviews of any historical financial information by a
the appropriate level. practitioner other than the entity's auditor.
II. The responsible party or the engaging party imposes a
restriction that prevents the practitioner from obtaining 30. When performing a compilation engagement, the accountant
evidence required to reduce assurance engagement risk to is required to
the appropriate level. a. Make inquiries of management to assess the reliability
a. I only and completeness of the information provided.
b. II only b. Assess internal controls.
c. Either I or II c. Verify matters and explanations.
d. Neither I nor II d. Obtain a general knowledge of the business and
operations of the entity.
25. Reducing assurance engagement risk to zero is very rarely

AT – Prelim Rev (875 MCQs) Red Sirug Page 19 of 77


31. Each page of the financial information compiled by the d. Defines and describes the elements and objectives of
accountant should include the following reference, except an assurance engagement, and identifies
a. "Unaudited" engagements to which PSAs, PSREs, and PSAEs
b. "Compiled, Negative Assurance Expressed" apply.
c. "Compiled without Audit or Review"
d. "Refer to Compilation Report" 37. The procedures employed in doing compilation are:
a. Designed to enable the accountant to express a limited
32. An auditor may accept an engagement to perform specified assurance
procedures on the specific subject matter of specified b. Designed to enable the accountant to express a negative
elements, accounts, or items of a financial statement if assurance
a. The report does not list the procedures performed. c. Not designed to enable the accountant to express
b. The auditor is also the entity's continuing auditor. any form of assurance
c. The financial statements are prepared in accordance with d. Less extensive than review procedures but more
a special purpose framework. extensive than agreed-upon procedures
d. Use of the report is restricted.
38. Which of the following procedures is normally performed in
33. An engagement to perform agreed-upon procedures may connection with a compilation engagement?
involve the auditor in performing certain procedures a. Inquire of management about subsequent events
concerning b. Making inquiries of management concerning actions
I. Individual items of financial data. taken at board meeting
II. A single financial statement. c. Applying analytical review procedures
III. A complete set of financial statements. d. Collect, classify and summarize financial information.
a. I and II only
b. II and III only 39. An engagement to apply agreed-upon procedure engagement
c. I and III only may be accepted, provided
d. I, II, and III a. The CPA has audited the financial statements of the
client.
34. The following contain basic principles, essential procedures b. The CPA is independent with respect to the client.
and related guidance, consistent with the concepts in the c. The distribution of the report will be limited only to
Framework, for the performance of assurance engagements, specified parties involved.
except d. The adequacy of the procedures to be performed will be
a. PSAs determined by the CPA.
b. PSREs
c. PSAEs 40. Pronouncements of Auditing and Assurance Standards
d. PSRSs Council (AASC) do not cover
a. Review engagement
35. CPAs in public practice who perform assurance engagements b. Compilation engagement
are governed by the following, except c. Consultancy
a. Framework for Assurance Engagements d. Agreed-upon procedures engagement
b. Philippine Standards on Quality Control
c. Code of Ethics for Professional Accountants in the 41. What is the most likely course of action that will be taken by
Philippines an auditor in assessing management integrity?
d. AASC's Engagement Standards (PSAs, PSREs, a. Tour the client premises.
PSAEs, and PSRSs) b. Research the prospective client’s related parties.
c. Research the background and histories of officers
36. The Philippine Framework for Assurance Engagements d. Review the minutes of the board of directors
a. Provides a frame of reference for CPAs in public practice
when performing audits, reviews, and compilations of 42. Which of the following is an example of an assurance
historical financial information. engagement?
b. Contains basic principles, essential procedures, and a. Management advisory services
related guidance for the performance of assurance b. Reporting on financial statements prepared using
engagements. other comprehensive basis of accounting.
c. Establishes standards and provides procedural c. compilation of financial information
requirements for the performance of assurance en- d. preparation of tax returns
gagements.

AT – Prelim Rev (875 MCQs) Red Sirug Page 20 of 77


43. Which of the following services provides a moderate level of
assurance about the client's financial statements? 48. Which of the following factors would most likely cause a CPA
a. Forecasts and projections to decide not to accept a new audit engagement?
b. Compliance with contractual agreement a. The CPA’s lack of understanding of the prospective
c. Review client’s internal auditor’s computer-assisted audit
d. Compilation techniques.
b. Management’s disregard of its responsibility to
44. When an independent auditor is approached to perform an maintain an adequate internal control environment.
audit for the first time, he/she should make inquiries of the c. The CPA’s inability to determine whether related-party
predecessor auditor. Inquiries are necessary because the transactions were consummated on terms equivalent to
predecessor may be able to provide the successor with arm’s-length transactions.
information that will assist the successor in determining d. Management’s refusal to permit the CPA to perform
whether substantive tests before the year-end.
a. A certain amount of audit fee will be acceptable to the
client. 49. The following are fundamental principles that are mostly
b. An unqualified opinion may be issued. applied when performing procedures on acceptance of new
c. The audit documentation by the predecessor auditor can clients except:
be utilized. a. Confidentiality.
d. The engagement proposal should be accepted. b. Professional competence.
c. Independence.
45. Before accepting an audit engagement, a successor auditor d. Professional competence and due care.
should make specific inquiries of the predecessor auditor
regarding the predecessor's 50. Before continuing the client relationship, the auditor should
a. Evaluation of all matters of continuing accounting perform the following except:
significance. a. Communicate with the predecessor auditor.
b. Understanding as to the reasons for the change of b. Evaluate client in case of change of client’s business.
auditors. c. Evaluate client’s integrity.
c. Awareness of the consistency in the application of GAAP d. Evaluate financial condition of the client.
between periods.
d. Opinion of any subsequent events occurring since the 51. A successor auditor most likely would make specific inquiries
predecessor's audit report was issued. of the predecessor auditor regarding
a. Specialized accounting principles of the client's industry.
46. Before accepting an engagement to audit a new client, a CPA b. Disagreements with management as to auditing
is required to obtain procedures.
a. An understanding of the prospective client’s industry and c. The competency of the client's internal audit staff.
business. d. The uncertainty inherent in applying sampling procedures.
b. The prospective client’s signature to the engagement
letter. 52. Which of the following will an auditor least likely discuss with
c. A preliminary understanding of the prospective client’s the former auditors of a potential client prior to acceptance?
control environment. a. Disagreements with management regarding accounting
d. The prospective client’s consent to make inquiries of principles
the predecessor auditor, if any. b. Integrity of management
c. Fees charged for services
47. Before accepting an audit engagement, a successor auditor d. Reasons for changing audit firms
should make specific inquiries of the predecessor auditor
regarding 53. Prior to the acceptance of an audit engagement with a client
a. Disagreements the predecessor had with the client who has terminated the services of the predecessor auditor,
concerning auditing procedures and accounting the CPA should
principles. a. Accept the engagement without contacting the
b. The predecessor’s evaluation of matters of continuing predecessor auditor since the CPA can include audit
accounting significance. procedures to verify the reason given by the client for the
c. The degree of cooperation the predecessor received termination.
concerning the inquiry of the client’s lawyer. b. Contact the predecessor auditor without advising the
d. The predecessor’s assessments of inherent risk and prospective client and request a complete report of the
judgments about materiality. circumstance leading to the termination with the

AT – Prelim Rev (875 MCQs) Red Sirug Page 21 of 77


understanding that all information disclosed will be kept c. Procedures to be undertaken to discover litigation, claims,
confidential. and assessments.
c. Advise the client of the intention to contact the d. Pending legal matters to be included in the inquiry of the
predecessor auditor and request permission for the client's attorney.
contact.
d. Not communicate with the predecessor auditor because 59. A written understanding between the auditor and the client
this would in effect be asking the auditor to violate the concerning the auditor's responsibility for the discovery of
confidential relationship between auditor and client. illegal acts is usually set forth in a(an)
a. Letter of audit inquiry.
54. Auditing standards require a successor auditor to b. Client representation letter.
communicate with the predecessor auditor. The Code of c. Management letter.
Professional Conduct requires confidentiality; therefore, the d. Engagement letter.
client's permission must be obtained before the
communication can be made by 60. The scope and nature of an auditor's contractual obligation to
a. The predecessor auditor. a client is ordinarily set forth in the
b. The successor auditor. a. Opinion paragraph of the auditor’s report.
c. Both the successor and predecessor auditor. b. Management letter.
d. Neither, since this is one of the exceptions to c. Engagement letter.
confidentiality in the Code of Professional Conduct. d. Scope paragraph of the auditor’s report.

55. Which of the following is not done during the client selection 61. The primary reason an engagement letter is obtained by audit
and retention phase of planning? firms prior to starting the work is that
a. Obtain and review financial information a. It provides an insurance policy for companies entering
b. Consider the need for special skills into the agreement.
c. Ensure that the firm has sufficient resources to complete b. It clarifies the responsibilities of management and
the engagement in a timely manner. those of the auditor.
d. Obtain an understanding of internal controls c. It communicates the type of opinion that will be rendered
on the engagement
56. Which of the following factors most likely would influence an d. It defines the firm's policies and procedures regarding
auditor’s determination of the auditability of an entity’s new clients.
financial statements?
a. The complexity of the accounting system. 62. An engagement letter should be written before the start of an
b. The existence of related-party transactions. audit because
c. The adequacy of the accounting records. a. It specifies the client's responsibility for preparing
d. The operating effectiveness of control procedures. schedules and making the records available to the auditor.
b. It may limit the auditor's legal liability by specifying the
57. Prior to beginning fieldwork on a new audit engagement in auditor's responsibilities.
which a CPA does not possess industry expertise, the CPA c. It specifies the basis for billing the audit for the upcoming
should year.
a. Obtain knowledge of matters that relate to the nature d. All of the above
of the entity's business and industry.
b. Reduce audit risk by lowering the preliminary levels of 63. Which of the following matters is generally included in an
materiality. auditor’s engagement letter?
c. Engage financial experts who are familiar with the nature a. Management’s responsibility for the entity’s
of the industry. compliance with laws and regulations.
d. Design special substantive tests to compensate for the b. The factors to be considered in setting preliminary
lack of industry expertise. judgments about materiality.
c. Management’s vicarious liability for illegal acts committed
58. The element of the audit planning process most likely to be by its employees.
agreed upon with the client before implementation of the audit d. The auditor’s responsibility to search for significant
strategy is the determination of the internal control deficiencies.
a. Timing of inventory observation procedures to be
performed. 64. Which of the following is not included in the engagement letter?
b. Evidence to be gathered to provide a sufficient basis for a. Objectives of the engagement
the auditor's opinion. b. Management's responsibilities

AT – Prelim Rev (875 MCQs) Red Sirug Page 22 of 77


c. A clear explanation of the services to be performed on a. While preferably written, it may be oral; but in all
the engagement cases it should be documented in the working papers.
d. Representations that the financial statements were b. While preferably written, it may be oral, in which case it
prepared in accordance with generally accepted need not be documented in the working papers.
accounting principles c. The understanding may be in any form, such as oral or
written.
65. After an auditor had been engaged to perform the first audit d. The understanding must be obtained in written form and
for a nonpublic entity, the client requested to change the included in the working papers.
engagement to a review. In which of the following situations
would there be a reasonable basis to comply with the client's 70. In auditing the financial statements of Star Corp., Land
request? discovered information leading Land to believe that Star’s prior
a. Management refused to sign the client representation year’s financial statements, which were audited by Tell,
letter. require substantial revisions. Under these circumstances,
b. The client's bank required an audit before committing Land should
to a loan, but the client subsequently acquired a. Notify Star’s audit committee and stockholders that the
alternative financing. prior year’s financial statements cannot be relied on.
c. The auditor was prohibited by the client from b. Request Star to reissue the prior year’s financial
corresponding with the client's legal counsel. statements with the appropriate revisions.
d. The auditing procedures were substantially complete and c. Notify Tell about the information and make inquiries about
the auditor determined that an unqualified opinion was the integrity of Star’s management.
warranted, but there was a disagreement concerning the d. Request Star to arrange a meeting among the three
audit fee. parties to resolve the matter.

66. Which of the following statements would least likely appear in 71. Orange Corp. has a few large accounts receivable that total
an auditor’s engagement letter? P2,000,000. Yellow Corp. has a large number of small
a. Fees for our services are based on our regular per diem accounts receivable that also total P2,000,000. The
rates, plus travel and other out-of-pocket expenses. importance of an error in any one account is, therefore,
b. During the course of our audit we may observe greater for Orange Corp. than for Yellow Corp. This is an
opportunities for economy in, or improved controls over, example of the auditor's concept of:
your operations. a. Materiality.
c. After performing our preliminary analytical b. Reasonable assurance.
procedures we will discuss with you the other c. Comparative analysis
procedures we consider necessary to complete the d. Relative risk.
engagement.
d. Our engagement is subject to the risk that material 72. The relationship between materiality and audit risk is:
misstatements or fraud, if they exist, will not be detected. a. Indeterminable
b. Direct.
67. Arrangements concerning which of the following are least c. Inverse.
likely to be included in engagement letter? d. Materiality and audit risk have no relationship
a. Internal auditors
b. Predecessor auditor 73. An auditor should design a written audit program so that:
c. Stockholders a. All material transactions will be selected for substantive
d. Appraisers testing.
b. Substantive tests prior to the balance sheet date will be
68. When an auditor believes that an understanding with the client minimized.
has not been established, he or she should ordinarily c. The audit procedures selected will achieve specific
a. Perform the audit with increased professional skepticism. audit objectives.
b. Decline to accept or perform the audit. d. Each account balance will be tested under either tests of
c. Review the client’s financial reports. controls or tests of transactions.
d. Obtain information about the client’s business and the
industry where it operates. 74. Audit programs are modified to suit the circumstances on
particular engagements. A complete audit program for an
69. Select the proper reply as to the allowable form of the engagement generally should be developed
understanding with a client when an audit is being performed. a. Prior to beginning the actual audit work.

AT – Prelim Rev (875 MCQs) Red Sirug Page 23 of 77


b. After the auditor has completed an evaluation of the d. Reading the minutes of stockholder and director meetings
existing internal accounting control. to discover whether any unusual transactions have
c. After reviewing the client's accounting records and occurred.
procedures.
d. When the audit engagement letter is prepared. 80. A CPA is conducting the first audit of a client’s financial
statements. The CPA hopes to reduce the audit work by
75. Which of the following is not a potential effect of an auditor's consulting with the predecessor auditor and reviewing the
decision that a lower acceptable audit risk is appropriate? predecessor's working papers. This procedure is
a. More evidence is required. a. Acceptable if the CPA refers in the audit report to reliance
b. Less evidence is required. upon the predecessor auditor's work.
c. Special care is required in assigning experienced staff. b. Required if the CPA is to render an unqualified opinion.
d. Review of the working papers by personnel who were not c. Unacceptable because the CPA should bring an
assigned to the engagement. independent viewpoint to a new engagement.
d. Acceptable if the client and the predecessor auditor
76. To obtain an understanding of a continuing client’s business in agree to it.
planning an audit, an auditor most likely would
a. Read specialized industry journals. 81. PSA 315 requires that the auditor should obtain an
b. Review prior year working papers and the permanent understanding of relevant industry, regulatory and other
file for the client. external factors including the applicable financial reporting
c. Reevaluate the client’s internal control environment. framework. Which of the following is not among the items that
d. Perform tests of details of transactions and balances. relate to industry conditions?
a. Energy, supply and cost
77. The management responsibility to detect and prevent fraud b. Inflation and currency revaluation
and error is accomplished by c. Market and competition
a. Having an annual audit of financial statements. d. Cyclical or seasonal activity
b. Implementing adequate quality control system.
c. Implementing adequate accounting and internal 82. The risk of material financial statement misstatement may be
control system. greater when the following conditions exist except
d. Issuing a representation letter to the auditor. a. When there is greater management intervention to
specify the accounting treatment.
78. Which of the following statements best describes the auditor's b. When there is sufficient personnel with appropriate
responsibility to detect material errors and fraud? accounting and financial reporting skills.
a. The auditor is responsible for the failure to detect material c. When there is greater manual intervention for data
errors and frauds only when such failure results from the collection and processing.
misapplication of generally accepted accounting d. Complex calculations or accounting principles is involved.
principles.
b. The auditor is responsible for the failure to detect material 83. The following are assurance engagements except
errors and frauds only when the auditor fails to confirm a. Tax consulting
receivables or observe inventories. b. Financial statements audit
c. The audit should be designed to provide reasonable c. Review of financial statements
assurance that material errors and fraud are detected. d. Information system reliability services
d. Extended auditing procedures are required to detect
unrecorded transactions even if there is no evidence that 84. Engagements frequently performed by professional
material errors and frauds may exist. accountants that are not assurance engagements include the
following except
79. Which of the following procedures would an auditor be most a. Compilation
likely to perform in planning a financial statement audit? b. Agreed-upon procedures.
a. Reviewing investment transactions of the audit period to c. Compliance audit
determine whether related parties were created. d. Management consulting.
b. Performing analytical procedures to identify areas
that may represent specific risks. 85. When the professional accountant has obtained sufficient
c. Obtaining a written representation letter from the client to appropriate evidence to conclude that the subject matter
emphasize management's responsibilities. conforms in all material respects with identified suitable criteria,
he or she can provide what level of assurance?
a. None

AT – Prelim Rev (875 MCQs) Red Sirug Page 24 of 77


b. High section of the telephone book.
c. Absolute b. He obtained a loan from a bank under the normal lending
d. Moderate procedures, terms and requirements of that bank.
c. He holds one share of the client's capital stock.
86. The Framework of PSA applies to d. He failed to disclose a client's departure from GAAP.
a. Taxation
b. Consultancy 93. When CPAs are able to maintain an independence attitude in
c. Accounting advice fulfilling their responsibility, it is referred to as independence in
d. Compilation a. Fact.
b. Appearance.
87. Which of the following procedures ordinarily performed during c. Conduct.
an audit are also performed in review? d. Total.
a. Assessment of accounting and internal control systems
b. Test of controls 94. When the users of financial statements have confidence in the
c. Tests of records and of responses to inquiries independence of the CPA, it is referred to as in independence
d. Inquiry and analytical procedures in
a. Fact.
88. Which statement is incorrect regarding the pronouncements of b. Appearance.
AASC? c. Conduct.
a. The PSAs and Interpretations may also have application, d. Total.
as appropriate, to other related activities of auditors.
b. PSAs contain basic principles and essential procedures 95. Which of the following statements is incorrect?
(identified in bold type black lettering) together with a. CPAs lose their independence if they acquire any direct
related guidance in the form of explanatory and other financial interest in a client.
material. b. CPAs lose their independence if they have a material
c. PSAs need only be applied to material matters. direct financial interest in a client.
d. The Interpretations have the same authority as the c. CPAs lose their independence if they acquire any
PAPSs. indirect financial interest in a client.
d. CPAs lose their independence if they acquire a material
89. The amount of audit fees depend largely on the indirect financial interest in a client.
a. Size and capitalization of the company under audit.
b. Amount of profit for the year. 96. Which of the following statements is not a distinction between
c. Availability of cash. independent auditing and internal auditing?
d. Volume of audit work and degree of competence and a. Independent auditors represent third party users external
responsibilities involved. to the auditee entity, whereas internal auditors report
directly to management.
90. In determining audit fees, an auditor may take into account b. Internal auditors are employees of the auditee, whereas
each of the following except independent auditors are independent contractors.
a. Volume and intricacy of work involved. c. The internal auditor's span of coverage goes beyond
b. Number and cost of manhours needed. financial auditing to encompass operational and
c. Degree of responsibility assumed. performance auditing.
d. Size and amount of capital of client. d. Although independent auditors strive for both validity
and relevance of evidence, internal auditors are
91. Under this method of billing a client, the external auditors concerned almost exclusively with validity.
charges on the basis of time spent by principals/partners,
supervisors, seniors and juniors at predetermined rates 97. Which of the following is a correct qualification of the
agreed upon with the client Chairman and Two Commissioners of the Commission on
a. Maximum fee basis Audit?
b. Flat sum basis a. A citizen of the Philippines.
c. Retainer basis b. At least 40 years of age upon appointment.
d. Per diem basis c. Must not have been candidates for any elective
position preceding appointment.
92. Which of the following will impair the independence of a CPA d. CPA’s with no less than 5 years of auditing experience or
in public practice? members of Philippine bar who have been engaged in
a. He has his name and address listed on a one-page law practice for at least 5 years.

AT – Prelim Rev (875 MCQs) Red Sirug Page 25 of 77


b. Is not legally protected and can be subpoenaed by a
98. The 1986 Constitution provides that the Chairman and competent court.
Commissioners of the Commission on Audit shall be c. Can only be released for peer reviews after receiving
a. All lawyers permission from the client.
b. All Certified Public Accountants d. Should be conveyed to the public if it affects the
c. Two lawyers and one CPA "correctness" of the financial statements.
d. One or two lawyers and one or two CPAs for a total of
three 3. If an auditor had a substantial stock investment in a client that
s(he) was auditing, which of the following would be true?
99. Which statement is correct regarding the relationship between a. The auditor would lack independence.
internal auditing and the external auditor? b. The auditor would be violating the PASB standards.
a. Some judgments relating to the audit of the financial c. The auditor would be violating the Institute of
statements are those of the internal auditor. Management Accounting standards
b. The external audit function's objectives vary according to d. The auditor would be violating the IIA standards.
management's requirements.
c. Certain aspects of internal auditing may be useful in 4. The CPA must not subordinate his or her professional
determining the nature, timing and extent of external judgment to that of others in every
audit procedures. a. Engagement.
d. The external auditor is responsible for the audit opinion b. Audit engagement.
expressed, however that responsibility may be reduced c. Engagement except tax services.
by any use made of internal auditing. d. Engagement except management advisory services.

100. Which of the following is incorrect regarding the Philippine 5. Which of the following is an indication of lack of objectivity of
Standards on Assurance Engagements (PSAE)? an auditor?
a. It provides an overall framework for assurance a. The auditor believes that accounts receivable may
engagements intended to provide either a high or not be collectible, but accepts management's opinion
moderate level of assurance. without an independent evaluation.
b. When a professional accountant is engaged to perform b. In preparing client's tax return, the CPA encourages client
an assurance engagement for which specific standards to take a deduction which the CPA believes is valid, but
exist, those standards apply. for which there is some but not complete support.
c. It provides basic principles and essential procedures c. Both a and b above would be a violation
for engagements intended to provide a moderate d. Neither would be a violation
level of assurance.
d. If no specific standards exist for an assurance 6. A CPA in public practice shall not disclose any confidential
engagement, PSAE apply. client information without the specific consent of the client.
The confidentiality rule is violated if CPA disclosed
information without client's consent as a result of a
SET 4 a. Subpoena or summons.
b. Peer review.
1. A professional accountant has a professional duty or right to c. Request by client's largest stockholder.
disclose confidential information in each of the following, d. Complaint filed with the trial board of the Board of
except: Accountancy.:
a. To comply with technical standards and ethics
requirements. 7. The confidential relationship applies to
b. To disclose to BIR fraudulent scheme committed by a. All services provided by CPAs.
the client on payment of income tax. b. Only audit and attestation services.
c. To comply with the quality review of a member body or c. Audit and tax services, but no MAS services.
professional body d. Audit and MAS services, but not tax services.
d. To respond to an inquiry or investigation by a member
body or regulatory body. 8. The confidential relationship will be violated if, without client's
permission, the CPA provides working papers about client to
2. Which of the following best describes the passing of a. A court of law which subpoenas them.
confidential information from a client to its auditor? The b. Another CPA firm as part of a peer review.
information: c. Another CPA firm which has just purchased the
a. Should in no circumstances be conveyed to third parties. CPA's entire practice.

AT – Prelim Rev (875 MCQs) Red Sirug Page 26 of 77


d. An investigative or disciplinary body which is conducting d. The CPA, when questioned in court, admits to knowledge
a review of the CPA's practice. of certain illegal acts perpetrated by the client.

9. A member in public practice may perform for a contingent fee 14. An auditor who accepts an audit engagement and does not
any professional services for a client for whom the member possess the industry expertise of the business entity, should
or member's firm performs a. Engage, financial experts familiar with the nature of the
a. An audit. business entity.
b. A review. b. Obtain a knowledge of matters that relates to the
c. A compilation used only by management. nature of the entity's business.
d. An audit of prospective financial information. c. Refer a substantial portion of the audit to another cpa
who will act as the principal auditor.
10. Which one of the following contingent fee is allowed? d. First inform management that an unqualified opinion
a. All services performed by a CPA firm. cannot be issued.
b. Non-attestation services.
c. Non-attestation services, unless the CPA firm was 15. In determining estimates of fees, an auditor may take into
also performing attestation services for the same account each of the following, except the:
client. a. Value of the service to the client
d. Attestation services. b. Degree of responsibility assumed by undertaking the
engagement.
11. A violation of the ethical standards would most likely have c. Skills required to perform the service.
occurred when a CPA d. Attainment of specific findings.
a. Made arrangement with a bank to collect notes issued by
a client in payment of fees due. 16. A CPA, while performing an audit, strives to achieve
b. Joined an accounting firm made up of three non-CPA independence in appearance in order to
practitioners. a. Reduce risk and liability.
c. Issued an unqualified opinion on the 2006 financial b. Comply with the generally accepted standards of
statements when fees for the 2005 audit were unpaid. fieldwork.
d. Purchased a bookkeeping firm's practice of monthly write- c. Become independent in fact.
ups for a percentage of fees received over a three-year d. Maintain public confidence in the profession
period.
17. The IFAC Code of Professional Conduct will ordinarily be
12. The concept of materiality would be least important to an considered to have been violated when the member
auditor when considering the represents that specific consulting services will be performed
a. Decision whether to use positive or negative for a stated fee and it is apparent at the time of the
confirmations of accounts receivable. representation that the
b. Adequacy of disclosure of a client's illegal act. a. Actual fee would be substantially higher.
c. Discovery of weaknesses in a client's internal control b. Actual fee would be substantially lower than the fees
structure. charged by other members for comparable services.
d. Effects of a direct financial interest in the client upon c. Fee was a competitive bid.
the CPA's independence. d. Member would not be independent.

13. Which of the following is a violation Confidentiality rule of the 18. In which of the following instances would the independence of
Code of Ethics? the CPA not be considered to be impaired? The CPA has
a. The CPA, in response to a court subpoena, submits been retained as the auditor of a brokerage firm
auditor-prepared workpapers as evidence of possible a. Which owes the CPA audit fees for more than one year.
illegal acts perpetrated by the client. b. In which the CPA has a large active margin account.
b. The CPA discloses to the board of directors a scheme c. In which the CPA's brother is the controller.
concocted by top management to intentionally inflate d. Which owes the CPA audit fees for services in the
earnings. current year and has just filed a petition for
c. The CPA warns Client B as to the inadvisability of bankruptcy.
acquiring Client A. The CPA bases this warning on
knowledge of Client A's financial condition and a 19. In performing an audit, Jackson, CPA, discovers that the
belief that the management of Client A lacks integrity. professional competence necessary for the engagement is
This knowledge was obtained by the CPA as a result lacking. Jackson informs management of the situation and
of auditing Client A during the past several year is. recommends another local firm, and management engages

AT – Prelim Rev (875 MCQs) Red Sirug Page 27 of 77


this other firm. Under these circumstances, c. Audit has been performed by persons having adequate
a. Jackson may request compensation from the other technical training and proficiency as auditors.
firm for any professional services rendered to it in d. Auditor's system of quality control has been maintained at
connection with the engagement. a high level.
b. Jackson may accept a referral fee from the other firm.
c. Jackson has violated the AICPA Code of Professional 24. Risk assessment procedures include the following, except
Conduct because of nonfulfillment of the duty of a. Analytical procedures.
performance. b. Confirmation of accounts receivable.
d. Jackson's lack of competence should be construed to be c. Observation and inspection.
a violation of generally accepted auditing standards. d. Inquiries of management

20. Which of the following fee arrangements is in violation of the 25. PSA 315 requires:
Code of Professional Conduct? a. Obtaining an understanding of the entity and its
a. A fee based on whether the CPA's report on the environment
client's financial statements results in the approval of b. Identifying and assessing the risks of material
a bank loan. misstatement
b. A fee based on the outcome of a bankruptcy proceeding. c. Discussion among engagement team members about the
c. A fee based on the nature of the service rendered and risk of material misstatement in the financial statements.
the CPA's particular expertise instead of the actual time d. All of the above
spent on the engagement.
d. A fee based on the fee charged by the prior auditor. 26. A time budget is an estimate of the total hours an audit is
expected to take. The following are among the factors to be
21. Richard, CPA, performs accounting services for Norton considered in developing this budget, except
Corporation. Norton wishes to offer shares to the public and a. Client's size as indicated by its gross assets, sales,
asks Richard to audit the financial statements. Richard refers number of employees.
Norton to Cruz, CPA, who is more competent in the area of b. Location of client facilities.
registration statements. Cruz performs the audit of Norton's c. The competence and experience of available staff.
financial statements and subsequently thanks Richard for the d. Whether the audit is performed during the interim or
referral by giving Richard a portion of the audit fee. Richard at year-end.
accepts the fee. Who, if anyone, has violated professional
ethics? 27. The extent of audit planning will vary according to the
a. Only Richard following:
b. Both Richard and Cruz a. Auditor’s experience with the entity.
c. Only Cruz b. The nature and complexity of the audit engagement.
d. Neither Richard nor Cruz c. Size of the entity.
d. All of the above.
22. Which of the following would not fit the description of a
related-party transaction? 28. The audit plan should (select the exception)
a. An unusually large sale of merchandise to the a. Be flexible
company's best and largest customer. b. Precede performance of procedures
b. Sales of merchandise between a parent company and its c. Succeed action
subsidiary. d. Be cost-beneficial
c. Exchanges of equipment between two companies owned
by the same person. 29. The establishment of an overall audit strategy involves
d. Loans to corporate officers at market rates of interest with I. Determining the characteristics of the engagement that
a regular repayment schedule. define its scope.
II. Ascertaining the reporting objectives of the engagement
23. The audit work performed by each assistant should be to plan the timing of the audit and the nature of the
reviewed to determine whether it was adequately performed communications required.
and to evaluate whether the III. Considering the important factors that will determine the
a. Results are consistent with the conclusions to be focus of the engagement team's efforts.
presented in the auditor's report. a. I and II only
b. Audit procedures performed are approved in the b. I and III only
professional standards. c. II and III only
d. I, II, and III

AT – Prelim Rev (875 MCQs) Red Sirug Page 28 of 77


35. Auditors perform analytical procedures in the planning stage
30. In the planning stage of an audit engagement, the auditor is of an audit for the purpose of:
required to perform audit procedures to obtain an a. Determining which of the financial statement assertions
understanding of the entity and its environment, including its are the most important for the client's financial statements.
internal control. These procedures are called b. Determining the nature, timing, and extent of audit
a. Substantive tests procedures for auditing the inventory.
b. Tests of controls c. Deciding the matters to cover in an engagement.
c. Risk assessment procedures d. Identifying unusual conditions that deserve more
d. Dual-purpose tests auditing effort.

31. In planning the audit engagement, the auditor should consider 36. For initial audits, additional matters the auditor may consider
each of the following, except in the overall audit strategy and audit plan include the
a. Matters relating to the entity's business and the industry following except
in which it operates. a. Major issues including the application of accounting
b. Materiality level and audit risk. principles or any auditing and reporting standards
c. The kind of opinion (unmodified, qualified, or adverse) discussed with management.
that is likely to be expressed. b. Confirmation of material accounts receivable balance
d. The entity's accounting policies and procedures. at the end of the year.
c. Planned audit procedure to obtain sufficient appropriate
32. The auditor's understanding of the entity and its environment audit evidence regarding opening balances.
consists of an understanding of the following aspects except d. Assignment of firm personnel with appropriate levels of
a. Industry, regulatory, and other external factors, including capabilities and competence to respond to anticipated
the applicable financial reporting framework. significant risks.
b. Nature of the entity, including the entity's selection and
application of accounting policies. 37. Which of the following would not be found in the corporate
c. Measurement and review of the entity's financial charter?
performance. a. The date of incorporation.
d. Entity's selection and screening process of b. The rules and procedures adopted by the
marketing and production personnel. stockholders.
c. The kinds and amount of capital stock authorized.
33. PSA 315 requires that the auditor should obtain an d. The types of business activity that the corporation is
understanding' of relevant industry, regulatory and other allowed to conduct.
external factors including the applicable financial reporting
framework. Which of the following is not an example of 38. Which of the following would not usually be included in the
matters relating to regulatory environment that the auditor minutes of the board of directors and/or stockholders?
would usually consider? a. Declaration of dividends.
a. Regulatory framework for a regulated industry. b. Authorization of long-term loans.
b. Product technology relating to the entity's product. c. Authorization of individuals to sign checks.
c. Legislation and regulation significantly affecting the d. The duties and powers of the corporate officers.
entity's operation.
d. Taxation. 39. Which of the following is not an inherent risk that is common to
all clients in certain industries?
34. PSA requires that the auditor should obtain an understanding a. Accounts receivable collection in the consumer loan
of the entity's objectives and strategies, and the related industry.
business risks that may result in material misstatement of the b. Potential inventory obsolescence in the fashion clothes
financial statements. Which of the following is not an example industry.
of business risks that may have financial consequences and c. Brand loyalty in the cosmetics industry.
may affect the financial statements? d. Reserve for loss in the casualty insurance industry.
a. A contracting customer base due to industry
consolidation that may increase the risk of misstatement 40. Transactions with related parties are important to the auditors
associated with the valuation of receivables. because they will be disclosed in the financial statements if
b. Contracting economy. material. Generally accepted accounting principles would not
c. New accounting requirements. require disclosure of
d. Use of new IT. a. Loans to officers during the year which had been
repaid before the balance sheet date.

AT – Prelim Rev (875 MCQs) Red Sirug Page 29 of 77


b. The nature of the related-party relationship. d. Operational plans.
c. A description of transactions, including peso amounts.
d. The amounts due from and to related parties. 47. The auditor's understanding of the entity and its environment
consists an understanding of the following aspects:
41. Which of the following would not be classified as a related- a. Industry, regulatory and other external factors, including
party transaction? the applicable financial reporting framework
a. Sales of merchandise between affiliated companies. b. Nature of the entity, including the entity's selection and
b. An advance of one week's salary to an employee. application of accounting policies
c. Loans or credit sales to the principal owner or client. c. Objectives and strategies and the related business risks
d. Exchanges of equipment between two companies owned that may result in a material misstatement of the financial
by the same person. statements
d. All of these.
42. Most auditors assess inherent risk as high for related parties
and related-party transactions because 48. There are fundamental principles that the professional
a. Of the accounting disclosure requirement. accountant has to observe when performing assurance
b. Of the lack of independence between the parties. engagements. The requirement of which principle is of
c. Both a and b. particular importance in an assurance engagement in ensuring
d. It is required by generally accepted accounting principles. that the conclusion of the professional accountant has value to
the intended user?
43. Experience has shown that certain conditions in an a. Integrity
organization are symptoms of possible management fraud. b. Confidentiality
Which of the following conditions would not be considered an c. Professional competence
indicator of possible fraud? d. Objectivity
a. Managers are regularly assuming subordinates' duties
b. Managers are subject to formal performance reviews 49. Existing accountant, as defined in the Code of Ethics, means
on a regular basis. a. A professional accountant employed in industry,
c. Managers are dealing in matters outside their profit commerce, the public sector or education.
center's scope b. Those persons who hold a valid certificate issued by the
d. Managers are not complying with corporate directives Board of Accountancy.
and procedures c. A professional accountant in public practice
currently holding an audit appointment or carrying
44. The nature of the entity refers to the following, except: out accounting, taxation, consulting or similar
a. The types of investments that it is making and plans to professional services for a client.
make. d. A sole proprietor, or each partner or person occupying a
b. Other external factors, such as general economic position similar to that of a partner and each staff in a
conditions. practice providing professional services to a client
c. The way that the entity is structured and how it is irrespective of their functional classification (e.g., audit,
financed. tax or consulting) and professional accountants in a
d. The entity's operations, its ownership, and governance. practice having managerial responsibilities.

45. These result from significant conditions, events, 50. How did the Code of Ethics define public interest?
circumstances, actions or inactions that could adversely affect a. A distinguishing mark of a profession is the acceptance of
the entity’s ability to achieve its objectives and execute its its responsibility to the public.
strategies or through the setting of inappropriate objectives b. The accountancy profession's public consists of clients,
and strategies. credit grantors, governments, employers, employees,
a. Business failure investors, the business and financial community, and
b. Information risk others who rely on the objectivity and integrity of
c. Business obstacles professional accountants.
d. Business risk c. The collective well-being of the community of people
and institutions the professional accountant serves.
46. These are the operational approaches by which management d. The standards of the accountancy profession are
intends to achieve its objectives. heavily determined by the public interest.
a. Business risk approaches
b. Strategies 51. Independent auditors of financial statements perform audits
c. Planning methods that reduce and control

AT – Prelim Rev (875 MCQs) Red Sirug Page 30 of 77


a. Business risk faced by investors statements.
b. Information risk faced by investors b. Confirms the accuracy of the financial statements.
c. Complexity of financial statements c. Enhances the credibility of the financial statements.
d. Timeliness of financial statements d. Assures the readers that fraudulent activities of
employees have been detected.
52. Which of the following statements is true?
a. Professional standards prohibit CPAs from performing 58. Most of the independent auditor's work in formulating
non-assurance engagements. an opinion on financial statements consists of
b. Absolute assurance is attainable owing to the fact that a. Studying and evaluating internal control
much of the evidence available to the CPA is persuasive b. Obtaining and examining evidential matter
rather than conclusive. c. Examining cash transactions
c. The CPA’s conclusion provides a level of assurance d. Comparing recorded accountability with assets
about the subject matter.
d. The responsible party expresses a conclusion that 59. The overall objective of internal auditing is to
provides a level of assurance as to whether the subject a. Attest to the efficiency with which resources are
matter conforms, in all material respects, with the employed.
identified suitable criteria. b. Ascertain that controls are costs justified.
c. Provide assurance that financial data have been
53. Which of the following best describes the operational audit? accurately recorded.
a. It requires the constant review by internal auditors of the d. Assist members of the organization in the effective
administrative controls as they relate to operations of discharge of their responsibilities.
the company.
b. It concentrates on implementing financial and 60. Internal auditing is an independent appraisal function
accounting control in a newly organized company. established within an organization to examine and evaluate its
c. It concentrates on seeking out aspects of activities. To that end, internal auditing provides assistance to
operations in which waste would be reduced by the a. External auditors
introduction of controls. b. Stockholders
d. It attempts and is designed to verify the fair c. Management and the board of directors
presentation of a company's results of operations. d. Government

54. What is the proper organizational role of internal auditing? 61. An audit which is undertaken in order to determine whether
a. To serve as an independent, objective assurance and the auditee is following specific procedures or rules set down
consulting activity that adds value to operations. by some higher authority is classified as a(n)
b. To assist the external auditor in order to reduce external a. Audit of financial statements.
audit fees. b. Compliance audit.
c. To perform studies to assist in the attainment of more c. Operational audit.
efficient operations. d. Production audit.
d. To serve as the investigative arm of the audit committee
of the board of directors. 62. The most important function of operational audit report is to:
a. Direct management to take specified actions.
55. Which of the following is not one of the limitations of an audit? b. State the auditor’s opinion or conclusion.
a. The use of testing c. Report findings and recommendations.
b. Limitations imposed by client d. Report the objective of the audit.
c. Human error
d. Nature of evidence that the auditor obtains 63. Operational audit differs in many ways from an audit of
financial statements. Which of the following is the best
56. Which of the following terms does not belong to the group example of one of these differences?
a. Financial audit a. The usual audit of the financial statement covers four
b. Internal audit basic statements, whereas operational audit is usually
c. External audit limited either the statement of financial position or the
d. Independent audit
income statement.
57. By providing high level of assurance on audit reports on b. Operational audit do not necessarily result in the
financial statements, the auditor preparation of a report.
a. Guarantees the fair presentation of the financial c. The operational audit deals with pre-tax income.

AT – Prelim Rev (875 MCQs) Red Sirug Page 31 of 77


d. The boundaries of an operational audit are often generally accepted accounting principles.
drawn from an organization chart and are not limited d. Financial statements are fairly presented.
to a single accounting period.
70. As used in auditing, which of the following statements best
describes "assertions”?
64. Which of the following actions would be an appropriate
a. Assertions are the auditor's findings to be communicated
response by companies to improve the public's perception of
in his audit report.
their financial reporting?
b. Assertions are found only in the notes to the financial
a. Requiring internal auditors to report all significant findings of
statements.
fraud and illegal activity to the company president.
c. Assertions are the representations of management as to
b. Increased adoption of audit committees.
the reliability of the information system.
c. Keeping external and internal auditing work separated to
d. Assertions are the representations of management as
maintain independence.
to the fairness of presentation of the financial
d. None of the above.
statements.
65. Which of the following is considered a primary reason for
71. The internal auditing department's responsibility for deterring
creating an internal audit department?
fraud is to
a. To safeguard resources entrusted to the organization.
a. Exercise operating authority over fraud prevention
b. To evaluate and improve the effectiveness of
activities.
control processes.
b. Establish an effective internal control system.
c. To ensure the accuracy, reliability, and timeliness of
c. Maintain internal control.
financial and operating data used in management's
d. Examine and evaluate the system of internal control.
decision making.
d. To relieve management of the responsibility for
72. Internal auditors review the adequacy of the company's
establishing effective controls.
internal control system primarily to
a. Help determine the nature, timing, and extent of tests
66. What is the overall objective of internal auditing?
necessary to achieve audit objectives.
a. To attest to the efficiency with which resources are used.
b. Determine whether the internal control system ensures
b. Ascertain that the cost of internal control is justified.
that financial statements are fairly presented.
c. To ascertain that financial statements present
c. Determine whether the internal control system
accurately the financial position, operating results, and
provides reasonable assurance that the company's
changes in cash and stockholders' equity.
objectives and goals are met efficiently and
d. To help members of the organization to effectively
economically.
discharge their responsibilities.
d. Ensure that material weaknesses in the system of internal
control are corrected.
67. Which of the following types of audit uses laws and
regulations as its criteria?
73. Internal auditors should review the means of physically
a. Operational audit
safeguarding assets from losses arising from
b. Financial statement audit
a. Procedures that are not cost justified.
c. Compliance audit
b. Exposure to the elements.
d. Performance audit
c. Underusage of physical facilities.
d. Misapplication of accounting principles.
68. An audit that involves obtaining and evaluating evidence
about the efficiency and effectiveness of an entity's operating
74. The essence of the attest function is to
activities in relation to specified objectives is a(n):
a. Detect fraud.
a. External audit
b. Examine individual transactions so that the auditor can
b. Compliance audit
certify as to their validity.
c. Operational audit
c. Determine whether the client's financial statements
d. Financial statement audit
are fairly stated.
d. Ensure the consistent application of correct accounting
69. A operational audit is primarily oriented toward
procedures.
a. Future improvements to accomplish management’s
goals.
75. A typical objective of an operational audit is to determine
b. Past protection provided by existing internal control.
whether an entity's
c. Operational information is in accordance with
a. Internal control structure is adequately operating as

AT – Prelim Rev (875 MCQs) Red Sirug Page 32 of 77


designed d. A means of assurance that internal accounting controls are
b. Operational information is in accordance with functioning as planned.
generally accepted accounting principles.
c. Specific operating units are functioning efficiently 80. Governmental audi ting often ex tends beyond
and effectively examinations leading to the expression of opinion on the
d. Financial statements present fairly the results of fairness of financial presentation and includes audits of
operations efficiency, economy, effectiveness, and also:
a. Accuracy
76. Audits of financial statements include an expression of a b. Evaluation
conclusion about which of the following financial statement c. Compliance
characteristics? d. Internal control
a. Governance.
b. Reliability. 81. The internal auditing profession has advanced primarily as a
c. Relevance. result of
d. Timeliness. a. Increased interest by Bachelor of Science in Accoun-
tancy (BSA) graduates and experienced auditors.
77. There are four conditions that give rise to the need for b. The limitation of financial statement audit scope.
independent audits of financial statements. One of these c. Job qualification specifications that include added emphasis
conditions is consequence. In this context, consequence on background knowledge and skills.
means that the: d. Increased complexity and sophistication of business
a. Users of the statements may not fully understand the operations.
consequences of their actions.
b. Auditor must anticipate all possible consequences 82. A typical objective of an operational audit is for the auditor to
of the report issued. a. Determine whether the financial statements fairly
c. Impact of using different accounting methods may present the entity's operations.
not be fully understood by the users of the statements. b. Evaluate the feasibility of attaining the entity's
d. Financial statements are used for important operational objectives.
decisions. c. Make recommendation for improving performance.
d. Report on the entity's relative success in maximizing
78. One of the conditions that give rise to a demand for an profits.
external audit of financial statements is expertise. Which
of the following best describes the meaning of expertise as 83. An objective of a performance audit is to determine
used in this context? whether an entity's
a. Auditors usually rely on the work of an expert as a. Operational information is in accordance with
a basis for evaluating some assertions embodied in government auditing standards.
the financial statements. b. Specific operating units are functioning
b. The readers of the financial statements must possess economically and efficiently.
the necessary expertise to be able to understand the c. Financial statements present fairly the results of
financial statements. operations.
c. Users usually lack the necessary expertise to verify d. Internal control is adequately operating as designed.
the reliability of the financial information.
d. As experts, auditors are expected to detect all material 84. What is the responsibility of an auditor who is engaged to
misstatements in the financial statements. audit the financial statements of a government entity?
a. Assess control risk with respect to each component of
79. Operational audits generally have been conducted by internal and internal control.
COA auditors, but may be performed by certified public b. Assume responsibility for assuring that the entity complies
accountants. A primary purpose of an operational audit is to with applicable laws and regulations.
provide: c. Obtain an understanding of the possible financial
a. A measure of management performance in meeting statement effects of laws and regulations having direct
organizational goals. and material effects on amounts reported.
b. The results of internal examinations of financial and d. Design the audit to provide reasonable assurance that the
accounting matters to a company's top-level management. statements are free of material misstatements resulting
c. Aid to the independent-auditor, who is conducting the from illegal acts having direct or indirect effects.
examination of the financial statements.
85. Solicitation consists of the various means that CPA firms use

AT – Prelim Rev (875 MCQs) Red Sirug Page 33 of 77


to engage new clients. Which one-of the following would not c. Audit committee.
be an example of solicitation? d. Public.
a. Accepting new clients that approach the firm.
b. Taking prospective clients to lunch. 91. When a CPA who is not independent is associated with
c. Offering seminars on current tax law changes to potential financial statements, he would be precluded from expressing
clients. an opinion because
d. Advertisements in the yellow pages of a phone book. a. The public would be aware of his lack of independence
and would place little or no faith in his opinion.
86. Which of the following activities is not prohibited for the CPA b. He would place himself in the position of suffering an
firm's attestation service clients? adverse decision in a possible liability suit.
a. Competitive bidding on audit jobs. c. He would be in the position of auditing his own work.
b. Contingent fees on audit jobs. d. Any auditing procedures he might perform would not
c. Commissions for obtaining client services on audit jobs. be in accordance with generally accepted auditing
d. Referral fees on audit jobs. standards.

87. If requested to perform a review engagement for a nonpublic 92. In which of the following circumstances would a CPA be
entity in which an accountant has an immaterial direct financial bound by ethics to refrain from disclosing any confidential
interest, the accountant is information obtained during the course of a professional
a. Independent because the financial interest is immaterial engagement?
and, therefore, may issue a review report. a. The CPA is issue a summons enforceable by a court
b. Not independent and, therefore, may not be associated order which orders the CPA to present confidential
with the financial statements. information.
c. Not independent and, therefore, may not issue a b. A major stockholder of a client company seeks
review report. accounting information from the CPA after
d. Not independent and, therefore, may issue a review management declined to disclose the requested
report, but may not issue an auditor's opinion. information.
c. Confidential client information is made available as part of
88. Which of the following most completely describes how a quality review of the CPA's practice by a peer review
independence has been defined by the CPA profession? team authorized by the PICPA.
a. Performing an audit from the viewpoint of the public. d. An inquiry by a disciplinary body of PICPA requests
b. Avoiding the appearance of significant interests in the confidential client information.
affairs of an audit client.
c. Possessing the ability to act with integrity and 93. Which of the following best describes why publicly-traded
objectivity. corporations follow the practice of having the outside auditor
d. Possessing the ability to act professionally and appointed by the board of directors or elected by the
accordance with a professional code of ethics. stockholders?
a. To comply with the regulations of the Accounting
89. To emphasize auditor independence from management, Standards Council.
many corporations follow the practice of b. To emphasize auditor independence from the
a. Appointing a partner of the CPA firm conducting the management of the corporation.
examination to the corporation's audit committee. c. To encourage a policy of rotation of the independent
b. Establishing a policy of discouraging social contact auditors.
between employees of the corporation and the staff of the d. To provide the corporate owners with an opportunity to
independent auditor. voice their opinion concerning the quality of the auditing
c. Requesting that a representative of the independent firm selected by the directors.
auditor be on hand at the annual stockholders' meeting.
d. Having the independent auditor report to an audit 94. The objective of governmental effectiveness or program
committee of outside members of the board of auditing is to determine if the desired results of a program
directors. are being achieved. What is the first step in conducting
such an audit?
90. In determining independence with respect to any audit a. Identify the legislative intent of the program being
engagement, the ultimate decision as to whether or not the audited.
auditor is independent must be made by the b. Collect quantifiable data on the program's success
a. Auditor. or failure.
b. Client. c. Determine the time frame to be audited.

AT – Prelim Rev (875 MCQs) Red Sirug Page 34 of 77


d. Evaluate the system used to measure results. engagement to engagement. The best representation of this
application is that, from one audit engagement to the next,
95. Which of the following statements is a standard a. Both auditing standards and auditing procedures are
applicable to financial statement audits in accordance applied uniformly.
with Government Auditing Standards? b. Auditing standards are applied uniformly but auditing
a. An auditor should briefly describe in the auditor's report the procedures are optional.
method of statistical sampling used in per forming tests c. Auditing standards are applied uniformly but auditing
of controls and substantive tests. procedures may vary.
b. An auditor should report on the scope of the d. Auditing standards may vary but auditing procedures are
auditor's testing of internal control. applied uniformly.
c. An auditor should determine the extent to which
the entity's programs achieve the desired level of 100. The CPA should not undertake an engagement if his fee is to
results. be based upon
d. An auditor should assess whether the entity has a. A percentage of audited net income.
reportable measures of economy and efficiency b. Per diem rates plus expenses.
that are valid and reliable. c. The findings of a tax authority.
d. The complexity of the service rendered.
96. Which of the following is incorrect about responsibility for
financial statements?
a. Management is responsible for fair presentation of the SET 5
financial statements.
b. Auditor is responsible for expressing an opinion on the 1. The establishment of the overall audit strategy involves
financial statements. a. Determining the characteristics of the engagement that
c. Audit of financial statements does not reduce defines its scope.
management's responsibility. b. Ascertaining the reporting objectives of the engagement
d. Fair presentation of financial statements is an implicit to plan the timing of the audit and the nature of the
part of the auditor's responsibility. communications required.
c. Considering the important factors that will determine the
97. Which of the following is correct? focus of the engagement team's efforts.
a. The evidence which the auditor accumulates remains the d. All of the answers
same from audit to audit, but the general objectives vary,
depending on the circumstances. 2. Which of the following is least likely to be required on an audit?
b. The general audit objectives remain the same from a. Review accounting estimates for biases
audit to audit, but the evidence varies, depending on b. Test appropriateness of journal entries and adjustment
the circumstances. c. Make a legal determination of whether fraud has
c. The circumstances may vary from audit to audit, but the occurred
evidence accumulated remains the same. d. Evaluate the business rationale for significant, unusual
d. The general audit objectives may vary from audit to audit, transactions
but the circumstances remain the same.
3. Which of the following should be included in the audit plan?
98. The work of each assistant needs to be reviewed by personnel I. The nature, timing and extent of planned risk assessment
of at least equal competence. Which of the following is not one procedures.
of the objectives of this requirement? II. The nature, timing and extent of planned further audit
a. The conclusions expressed are consistent with the result procedures at the assertion level.
of the work performed and support the opinion. a. I only
b. The work performed and the results obtained have been b. II only
adequately documented. c. Both I and II
c. The objectives of the audit procedures have been d. Neither I nor II
achieved.
d. All available evidences have been obtained, 4. Which of the following matters would an auditor least likely
evaluated and documented. consider when setting the direction of the audit?
a. The selection of the engagement team and the
99. Every independent audit engagement involves both auditing assignment of audit work to the team members.
standards and auditing procedures. The relationship between
the two may be illustrated by how they apply from

AT – Prelim Rev (875 MCQs) Red Sirug Page 35 of 77


b. The engagement budget which includes consideration of 9. Which of the following statements concerning analytical
the appropriate amount of time to allot for areas where procedures is true?
there may be higher risks of material misstatement. a. Analytical procedures are more efficient, but not more
c. The availability of client personnel and data. effective, than tests of details of transactions.
d. The manner in which the auditor emphasizes to b. Analytical procedures used as risk assessment
engagement team members the need to maintain a procedures use data aggregated at a high level.
questioning mind and to exercise professional skepticism c. Analytical procedures can replace tests of controls in
in the gathering and evaluation of audit evidence. gathering audit evidence to support the assessed level of
control risk.
5. Which of the following matters would an auditor most likely d. Analytical procedures usually involve comparisons of
consider when establishing the scope of the audit? ratios developed from recorded amounts with ratios
a. Audit areas where there is a higher risk of material developed by management.
misstatement.
b. The expected audit coverage, including the number 10. In relation to audit planning, the auditor should document the
and locations of the entity's components to be following:
included. a. The overall audit strategy.
c. The entity's timetable for reporting, such as at interim and b. The detailed audit plan.
final stages. c. Significant changes made during the audit engagement.
d. The discussion with the entity's management concerning d. All of the answers
the expected communications on the status of audit work
throughout the engagement and the expected 11. Audit programs generally include procedures to test actual
deliverables resulting from the audit procedures. transactions and resulting balances. These procedures are
primarily designed to
6. An auditor should consider materiality when a. Detect irregularities that result in misstated financial
I. Determining the nature, timing, and extent of audit statements.
procedures. b. Test the adequacy of internal control.
II. Evaluating the effect of misstatements. c. Gather corroborative evidence.
a. I only d. Obtain information of informative disclosures.
b. II only
c. Both I and II 12. An audit process is a well-defined methodology for organizing
d. Neither I nor II an audit to ensure that
a. The evidence gathered is both sufficient and competent.
7. It is the amount or amounts set by the auditor at less than b. All appropriate audit objectives are specified.
materiality for the financial statements as a whole to reduce to c. All appropriate audit objectives are met.
an appropriately low level the probability that the aggregate of d. All of these
uncorrected and undetected misstatements exceeds
materiality for the financial statements as a whole. 13. For a particular assertion, control risk is the risk that
a. Lower materiality a. Controls will not detect a material misstatement that
b. Lesser materiality occurs.
c. Performance materiality b. Audit procedures will fail to detect a weak control system.
d. Materiality c. The prescribed control procedures will not be applied
uniformly.
8. A basic premise underlying analytical procedures is that d. A material misstatement will occur in the accounting
a. The study of financial ratios is an acceptable alternative process.
to the investigation of unusual fluctuations.
b. Plausible relationships among data may reasonably 14. Which of the following is most likely to require special planning
be expected to exist and continue in the absence of considerations related to asset valuation?
known conditions to the contrary. a. Accelerated depreciation methods are used for
c. These procedures cannot replace tests of details of amortizing the costs of factory equipment.
transactions and balances. b. The client has recently purchased an expensive copy
d. Statistical tests of financial information may lead to the machine.
detection of material misstatements in the financial c. Assets costing less than P5,000 are expensed even
statements. when the expected life exceeds one year.
d. Inventory is comprised of diamond rings.

AT – Prelim Rev (875 MCQs) Red Sirug Page 36 of 77


15. The responsibility for the detection and prevention of errors, d. Noncompliance
fraud and noncompliance with laws and regulations rests with:
a. Auditor 21. The primary factor that distinguishes errors from fraud is
b. Client's legal counsel a. Whether the misstatement is perpetrated by an employee
c. Internal auditor or by a member of management
d. Client management b. Whether the underlying cause of misstatement is
intentional or unintentional.
16. The following statements relate to the auditor's responsibility c. Whether the misstatement is concealed.
for the detection of errors and fraud. Which of the statement d. Whether the underlying cause of misstatement relates to
is correct? misapplication of accounting principles or to clerical
I. Due to the inherent limitations of the audit, there is a processing.
possibility that material misstatements in the financial
statements may not be detected. 22. The factor that distinguishes an error from fraud is
II. The subsequent discovery of material misstatement of a. Materiality.
the financial information resulting from fraud or error does b. Intent.
not, in itself, indicate that the auditor failed to follow the c. Whether it is peso amount or a process.
basic principles and essential procedures of an audit. d. Whether it is a caused by the auditor or the client.
a. I only
b. II only 23. Fraudulent financial reporting is often called
c. Both statements are true a. Misappropriation or theft of assets
d. Both statements are false b. Management fraud
c. Defalcation
17. Which of the following is not an assurance that the auditors d. Employee fraud
give to the parties who rely on the financial statements?
a. Auditors know how the amounts and disclosures in the 24. In comparing management fraud with employee fraud, the
financial statements were produced. auditor's risk of failing to discover the fraud is
b. Auditors gathered enough evidence to provide a a. Greater for employee fraud because of the larger number
reasonable basis for forming an opinion. of employees in the organization.
c. Auditors give assurance that the financial statements b. Greater for management fraud because of
are accurate. management's ability to override existing internal
d. If the evidence allows the auditors to do so, auditors give controls.
assurance in the form of opinion, as to whether the c. Greater for employee fraud because of the higher crime
financial statements taken as a whole are fairly presented rate among blue collar workers.
in conformity with GAAP. d. Greater for management fraud because managers are
inherently smarter than employees.
18. Material misstatements in financial statements may arise from
all of the following except 25. Which of the following statements about fraud or error is
a. Fraud incorrect?
b. Error a. The responsibility for the prevention and detection of
c. Limitations of the audit fraud and error rests with management.
d. Noncompliance with laws and regulations b. The auditor should plan and perform the audit with an
attitude of professional skepticism, recognizing that
19. Which of the following is an example of an error? conditions or events may be found that fraud or error may
a. Defalcation exist.
b. Misapplication of accounting policies. c. The likelihood of detecting fraud is ordinarily higher
c. Suppression or omission of the effects of transactions than that of detecting error.
from the records or documents. d. The auditor is not and can not be held responsible for the
d. Recording of transactions without substance. prevention of fraud and error.

20. An intentional act by one or more individuals among 26. Which of the following is an "error" as distinguished from
management, employees, or third parties which results in "fraud"?
misrepresentation of financial statements refers to a. Lapping
a. Illegal acts b. Embezzlement of company's fund
b. Error c. Clerical mistakes in the processing of transactions
c. Fraud d. Window dressing

AT – Prelim Rev (875 MCQs) Red Sirug Page 37 of 77


b. There is inadequate working capital due to declining
27. Which of the following could be an example of fraud? profits or too rapid expansion.
a. Misappropriation of assets or group of assets. c. The client is heavily dependent on one or a few products
b. Errors in the application of the accounting principles. or customers.
c. Clerical errors in accounting data underlying the financial d. There is a significant and prolonged understaffing of
statements. the accounting department.
d. Misinterpretation of facts that existed when financial
statements were prepared. 34. Which of the following is most likely to be a response to the
auditor's assessment that the risk of material misstatement
28. Which of the following is an example of fraudulent financial due to fraud for the existence of inventory is high?
reporting? a. Observe test counts of inventory at certain locations
a. Misappropriating collections on accounts receivable on an unannounced basis.
b. Using inappropriate assumptions in accounting b. Perform analytical procedures rather than taking test
estimate counts.
c. Stealing inventory c. Request that inventories be counted prior to year end.
d. Payments to fictitious employees or vendors d. Request that inventory counts at the various locations be
counted on different dates so as to allow the same
29. These refer to events or conditions that may indicate an auditor to be present at every count.
incentive or pressure to commit fraud or provide an
opportunity to commit fraud. 35. The term used to refer to acts of omission or commission by
a. Fraud conditions the entity being audited, either intentional or unintentional,
b. Fraud risk factors which are contrary to the prevailing laws and regulations is
c. Fraudulent activities a. Misappropriation.
d. Fraud events b. Fraud.
c. Illegal acts.
30. The following are fraud risk factors except: d. Noncompliance.
a. Incentive or pressure to commit fraud
b. Opportunity to commit fraud 36. Noncompliance with laws and regulations are also called
c. Attitude or rationalization to commit fraud a. Irregularities
d. All of the above b. Illegal acts
c. Misappropriation
31. Certain characteristics or circumstances may increase the d. Defalcation
susceptibility of assets to misappropriation. Opportunities to
misappropriate assets increase due to the following except: 37. The responsibility for the detection and prevention of
a. Inventory items that are small in size, of high value, or in noncompliance with laws and regulations rests with
high demand. a. The auditor
b. Known or anticipated future employee layoffs. b. The client's legal counsel
c. Easily convertible assets, such as bearer bonds, c. The auditor’s legal counsel
diamonds, or computer chips. d. The client management
d. Fixed assets which are small in size, marketable, or
lacking observable identification of ownership. 38. The term “noncompliance” as used in PSA 250 refers to acts
of omission or commission by the entity being audited, either
32. The auditor is most likely to presume that a high risk of fraud intentional or unintentional, which are contrary to the
exists if prevailing laws or regulations. Such acts include the following
a. The client is a multinational company that does business except
in numerous foreign countries. a. Transactions entered into by the entity
b. The client does business with several related parties. b. Transactions entered into in the name of the entity
c. Inadequate segregation of duties places an employee c. Transactions entered into by the entity on its behalf by its
in a position to perpetrate and conceal thefts. management or employees
d. Inadequate employee training results in lengthy EDP d. Personal misconduct (unrelated to the business
exception reports each month. activities of the entity) by the entity’s management or
employees.
33. All of the following conditions are indicators of possible
pressures on an entity except
a. The industry in which the entity operates is declining.

AT – Prelim Rev (875 MCQs) Red Sirug Page 38 of 77


39. As part of audit planning, CPAs should design audit programs a. Organizational level to which the internal auditor reports.
for each individual audit and should include audit steps and b. Quality of working paper documentation, reports, and
procedures to recommendations.
a. Ensure that only material items are audited. c. Influence of management on the internal auditor's duties.
b. Provide assurance that the objectives of the audit are d. Entity's commitment to integrity and ethical values.
met.
c. Detect and eliminate fraud. 45. Given that an audit in accordance with generally accepted
d. Increase the amount of management information auditing standards is influenced by the possibility of material
available. errors and fraud, the auditor should conduct the audit with an
attitude of
40. How can the audit program best be described at the beginning a. Professional responsiveness.
of the audit process? b. Conservative advocacy.
a. Conclusive c. Professional skepticism.
b. Confirmed d. Objective judgment.
c. Optional
d. Temporary 46. The primary difference between financial statement errors and
fraud is that
41. Internal auditing can affect the scope of the external auditor's a. Errors are intentional misstatements by management,
audit of financial statements by while fraud involves unintentional mistakes or omissions.
a. Decreasing the external auditor's need to perform b. Errors are more likely to provide an indication that an
detailed tests. illegal act has occurred.
b. Eliminating the need to observe the physical inventory c. Errors are unintentional mistakes or omissions, while
taking. fraud involves intentional misstatements.
c. Allowing the external auditor to limit his/her audit to the d. There is no difference as errors and fraud have the same
performance of substantive test procedures. meaning.
d. Limiting direct testing by the external auditor to
management assertions not directly tested by internal 47. Which of the following statements is correct relating to the
auditing. auditor’s consideration of fraud?
a. The auditor’s interest in fraud consideration relates
42. The external auditor should obtain a sufficient understanding to fraudulent acts that cause a material misstatement
of the internal audit function because of financial statements.
a. The understanding of the internal audit function is an b. A primary factor that distinguishes fraud from error is that
important substantive test to be performed by the external fraud is always intentional, while errors are generally, but
auditor. not always, intentional.
b. The audit programs, working papers, and reports of c. While an auditor should be aware of the possibility of
internal auditors may often be used as a substitute for the fraud, management, and not the auditor, is responsible
work of the external auditor's staff. for detecting fraud.
c. The procedures performed by the internal audit staff may d. Fraud always involves a pressure or incentive to commit
eliminate the external auditor's need for considering fraud, and a misappropriation of assets.
internal control.
d. The work performed by internal auditors may be a 48. Which of the following factors or conditions is an auditor least
factor in determining the nature, timing, and extent of likely to plan an audit to discover?
the external auditor's procedures. a. High turnover of senior management.
b. Inadequate monitoring of significant controls.
43. In determining whether the work of the internal auditors is c. Inability to generate positive cash flows from operations.
likely to be adequate for purposes of the audit, the external d. Financial pressures affecting employees.
auditor shall evaluate the internal auditor's
a. Efficiency and experience 49. Which of the following statements concerning audit planning is
b. Competence and objectivity incorrect?
c. Independence and review skills a. Planning is a continual and iterative process.
d. Training and supervisory skills b. Planning is a discrete phase of an audit.
c. In a recurring audit, planning often begins shortly after (or
44. In assessing the technical competence of an internal auditor, in connection with) the completion of the previous audit
an external auditor most likely would obtain information about and continues until the completion of the current audit
the engagement.

AT – Prelim Rev (875 MCQs) Red Sirug Page 39 of 77


d. In planning an audit, the auditor considers the timing of c. Internal control over shipping, billing, and recording
certain planning activities and audit procedures that are of sales revenue is weak.
to be completed prior to the performance of further audit d. The client has lost a major customer accounting for
procedures. approximately 30% of annual revenue.

50. The generally accepted auditing standards of fieldwork pertain 55. Which of the following is true?
most directly to: a. Auditors are responsible for detecting all fraudulent
a. Improving internal control as a result of the audit. financial reporting.
b. Due professional care in the performance of the audit. b. Auditors must specifically consider fraud risk from
c. The required training and proficiency of the auditors. overstating liabilities.
d. The planning of the audit. c. Auditors must specifically consider fraud risk from
management override of controls.
51. With respect to the auditor's planning of a year-end d. All of the above are true
examination, which of the following statements is always true?
a. An engagement should not be accepted after the fiscal 56. Which of the following is an example of fraudulent financial
year-end. reporting?
b. It is an acceptable practice to carry out substantial a. The treasurer diverts customer payments to his personal
parts of the examination at interim dates. due, concealing his actions by debiting an expense
c. An inventory count must be observed at the balance account, thus overstating expenses.
sheet date. b. Company management changes inventory count tags
d. The client's audit committee should not be told of the and overstates ending inventory, while understating
specific audit procedures that will be performed. cost of goods sold.
c. An employee steals inventory and the “shrinkage” is
52. An initial (first-time) audit requires more audit time to complete recorded in cost of goods sold.
than a recurring audit. One of the reasons for this is that d. An employee steals small tools from the company and
a. New auditors are usually assigned to an initial audit. neglects to return them; the cost is reported as a
b. A larger proportion of customer accounts receivable need miscellaneous operating expense.
to be confirmed on an initial audit.
c. The client's business, industry, and internal control 57. The type of transactions that ordinarily have a high inherent
are unfamiliar to the auditor and need to be carefully risk because they involve management judgments or
studied. assumptions are referred to as
d. Predecessor auditors need to be consulted. a. Estimation transactions.
b. Nonroutine transactions.
53. Which of the following is not required by PSA No. 315, c. Related-party transactions.
"Consideration of Fraud in a Financial Statement Audit"? d. Routine transactions.
a. Conduct a continuing assessment of the risks of material
misstatement due to fraud throughout the audit. 58. Professional skepticism
b. Conduct a discussion by the audit team of the risks of a. Assumes that management is either dishonest or
material misstatement due to fraud. assumes unquestioned honesty.
c. Conduct inquiries of the audit committee as to their views b. Either assumes that management is honest or dishonest.
about the risks of fraud and their knowledge of any fraud c. Neither assumes that management is dishonest nor
or suspected fraud. assumes unquestioned honesty.
d. Conduct the audit with professional skepticism, d. None of the above is a correct statement
which includes an attitude that assumes balances are
incorrect until verified by the auditor. 59. Which of the following conditions identified during fieldwork of
an audit is most likely to affect the auditor’s assessment of the
54. Inherent risk is defined as the susceptibility of an account risk of misstatement due to fraud?
balance or class of transactions to error that could be material a. Year-end adjusting journal entries.
assuming that there were no related internal controls. Of the b. Checks for significant amounts outstanding at year-end.
following conditions, which one does not increase inherent risk? c. Missing documents.
a. The board of directors approved a substantial bonus for d. Computer generated documents.
the president and chief executive officer, and also
approved an attractive stock option plan for themselves. 60. Which of the following best describes what is meant by the
b. The client has entered into numerous related party term “fraud risk factor?”
transactions during the year under audit.

AT – Prelim Rev (875 MCQs) Red Sirug Page 40 of 77


a. Factors whose presence indicates that the risk of fraud is
high. 65. When considering fraud risk factors relating to management’s
b. Factors whose presence requires modification of planned characteristics, which of the following is least likely to indicate
audit procedures. a risk of possible misstatement due to fraud?
c. Factors whose presence often have been observed in a. Failure to correct known reportable conditions on a timely
circumstances where frauds have occurred. basis.
d. Reportable conditions identified during an audit. b. Use of unusually conservative accounting practices.
c. Nonfinancial management’s preoccupation with the
61. Which of the following may cause management to intentionally selection of accounting principles.
understate profits? d. Significant portion of management’s compensation
a. Management wants to create "cookie jar" reserves for a represented by bonuses based upon achieving unduly
rainy day. aggressive operating results.
b. The company is under scrutiny by tax authorities.
c. The company is suffering a large loss and wants to take a 66. With respect to errors and fraud, the auditor should plan to
"big bath." a. Search for errors or fraud that would have a material
d. All of the above effect on the financial statements.
b. Discover errors or fraud that would have a material
62. Which of the following characteristics most likely would effect on the financial statements.
heighten an auditor’s concern about the risk of intentional c. Search for errors that would have a material effect and for
manipulation of financial statements? fraud that would have either material or immaterial effects
a. Turnover of senior accounting personnel is low. on the financial statements.
b. Insiders recently purchased additional shares of the d. Search for fraud that would have a material effect and for
entity’s stock. errors that would have either material or immaterial
c. Management places substantial emphasis on meeting effects on the financial statements.
earnings projections.
d. The rate of change in the entity’s industry is slow. 67. Auditor responsibility for identifying "direct effect" illegal acts
differs from their responsibility for detecting
63. Which of the following statements reflects an auditor’s a. Errors.
responsibility for detecting misstatements due to errors and b. Indirect-effect illegal acts.
fraud? c. Fraud.
a. An auditor is responsible for detecting employee errors d. Management fraud.
and simple fraud, but not for discovering fraud involving
employee collusion or management override. 68. Management’s attitude toward aggressive financial reporting
b. An auditor should design the audit to provide and its emphasis on meeting projected profit goals most likely
reasonable assurance of detecting misstatements would significantly influence an entity’s control environment
due to errors and fraud that are material to the when
financial statements. a. External policies established by parties outside the entity
c. An auditor should plan the audit to detect misstatements affect its accounting practices.
due to errors and fraud that are caused by departures b. Management is dominated by one individual who is
from GAAP. also a shareholder.
d. An auditor is not responsible for detecting misstatements c. Internal auditors have direct access to the board of
due to errors and fraud unless the application of GAAS directors and the entity’s management.
would result in such detection. d. The audit committee is active in overseeing the entity’s
financial reporting policies.
64. An auditor is required to obtain a basic understanding of the
client's internal control to plan the audit. The auditor may then 69. Which of the following is least likely to be required on an audit?
decide to perform tests of controls on all internal control a. Review accounting estimates for biases.
procedures b. Test appropriateness of journal entries and adjustment.
a. That would aid in preventing fraud. c. Make a legal determination of whether fraud has
b. Documented in the flowchart. occurred.
c. Considered to be weaknesses that might allow errors to d. Evaluate the business rationale for significant unusual
enter the accounting system. transactions.
d. Considered to be strengths for which the auditor
desires further reduction in the assessed level of 70. Which of the following is most likely to be an overall response
control risk. to fraud risks identified in an audit?

AT – Prelim Rev (875 MCQs) Red Sirug Page 41 of 77


a. Supervise members of the audit team less closely and controller for the purpose of discussing accounting
rely more upon judgment. practices that will maximize reported profits.
b. Only use certified public accountants on the engagement.
c. Place increased emphasis on the audit of objective 75. Which of the following methods may be used to commit
transactions rather than subjective transactions. fraudulent financial reporting?
d. Use less predictable audit procedures. a. Understate liabilities
b. Fail to provide adequate disclosure
71. The most likely explanation why the auditor’s examination c. Overstate revenues
cannot reasonably be expected to bring all illegal acts by the d. Each of the above can be used to commit fraudulent
client to the auditor’s attention is that financial reporting
a. Illegal acts are perpetrated by management override of
internal control. 76. The absence of which of the following internal controls
b. Illegal acts by clients often relate to operating increases the opportunity for fraud?
aspects rather than accounting aspects. a. Appropriate segregation of duties or independent checks
c. The client’s internal control may be so strong that the b. Job applicant screening for employees with access to
auditor performs only minimal substantive testing. assets
d. Illegal acts may be perpetrated by the only person in the c. Mandatory vacations for employees with access to assets
client’s organization with access to both assets and the d. The absence of any of the above increases the
accounting records. opportunity for fraud

72. Which of the following relatively small misstatements most 77. Whom should auditors contact when they suspect a fraud?
likely could have a material effect on an entity’s financial a. Senior management
statements? b. Expected perpetrators of the fraud
a. A piece of obsolete office equipment that was not retired. c. Audit committee of the board of directors
b. An illegal payment to a foreign official that was not d. A and C
recorded.
c. A petty cash fund disbursement that was not properly 78. Auditors would normally interview all but which of the following
authorized. individuals as part of their assessment of fraud risk?
d. An uncollectible account receivable that was not written a. Senior management
off. b. Audit committee
c. Various employees whose duties do not include normal
73. Which of the following might be considered a "red flag" financial reporting responsibilities
indicating possible fraud in a large manufacturing company d. All of the above
with several subsidiaries?
a. A consistent record of above average return on 79. After studying and evaluating a client's existing internal control,
investment for all subsidiaries an auditor has concluded that the policies and procedures are
b. The existence of a financial subsidiary well-designed and functioning as intended. Under these
c. Use of separate bank accounts for payrolls by each circumstances, the auditor would most likely
subsidiary a. Perform further control tests to the extent outlined in the
d. Complex sales-transactions and transfers of funds audit program.
between affiliated companies b. Determine the control policies and procedures that should
prevent or detect errors and fraud.
74. Warning signs that cause the auditor to question management c. Set detection risk at a higher level than would be set
integrity must be taken seriously and pursued vigorously. under conditions of weak internal control.
Which of the following may lead the auditor to suspect d. Set detection risk at a lower level than would be set under
management dishonesty? conditions of weak internal control.
a. The client has been named as a defendant in a product
liability suit. 80. An audit plan is a
b. The client has experienced a decrease in revenue from a. Detailed plan of analytical procedures and all substantive
increased import competition. tests to be performed in the course of the audit.
c. A new statutory regulation making customer licenses b. Generic document that auditing firms have developed to
more difficult to obtain may adversely affect the client's lead the process of the audit through a systematic and
operations. logical process.
d. The president and chief executive officer of the client c. Budget of the time that should be necessary to complete
corporation has held numerous meetings with the each phase of the audit procedures.

AT – Prelim Rev (875 MCQs) Red Sirug Page 42 of 77


d. Document that provides an overview of the company d. An understanding of controls established by management
and a general plan for the audit work to be
accomplished, timing of the work, and other matters 87. Which of the following is not a consideration in the
of concern to the audit. development of audit programs?
a. Internal control over the recording of plant asset additions
81. An audit program is and repairs and maintenance expenditures is found to be
a. A generic document that auditing firms have developed to weak.
lead the process of the audit through a systematic and b. The members of the board of directors are elected by
logical process. the stockholders during the annual meeting.
b. An overview of the company and a general plan for the c. The client is a private university located in Southern
audit work to be accomplished. Philippines.
c. The detailed plan of audit procedures to be d. The client constructed a major addition to its central
performed in the course of the audit. manufacturing facility during the year under audit.
d. A budget of the time that should be necessary to
complete each phase of the audit procedures. 88. The element of the audit planning process most likely to be
agreed upon with the client before implementation of the audit
82. Which of the following concepts is most useful in assessing strategy is the determination of the
the scope of an auditor's program relating to various accounts? a. Procedures to be undertaken to discover litigation, claims,
a. Attribute sampling and assessments.
b. Management fraud b. Timing of inventory observation procedures to be
c. Materiality performed.
d. The reliability of information c. Evidence to be gathered to provide a sufficient basis for
the auditor's opinion.
83. One of the primary roles of an audit program is to d. Pending legal matters to be included in the inquiry of the
a. Provide for a standardized approach to the audit client's attorney.
engagement.
b. Serve as a tool for planning, directing, and 89. When a CPA expresses an opinion on the financial statements,
controlling audit work. his responsibilities extend to
c. Document an auditor's understanding of the internal a. The underlying wisdom of the client's management
control. decision.
d. Delineate the audit risk accepted by the auditor. b. Active participation in the implementation of the advice
given to the client.
84. The principal reason for developing a written audit program is c. An ongoing responsibility for the client's solvency.
to help assure that the d. Whether the results of the client's operating
a. Audit work is properly supervised. decisions are fairly presented in the financial
b. Audit report contains only significant findings. statements.
c. Audit work is properly planned and documented.
d. Work of different auditors is properly coordinated. 90. The accuracy of information included in the footnotes that
accompany the audited financial statements of a company
85. An auditor should design the written audit program so that whose shares are traded on a stock exchange is the primary
a. All material transactions will be selected for substantive responsibility of
testing. a. The stock exchange officials.
b. Substantive tests prior to the balance sheet date will be b. The company's management.
minimized. c. The independent auditor.
c. Each account balance will be tested under either tests of d. The Securities and Exchange Commission.
controls or tests of transactions.
d. The audit procedures selected will achieve specific 91. The responsibility for adopting sound accounting policies,
audit objectives. maintaining adequate internal control, and making fair
representations in the financial statements rests
86. In the preparation of an audit program, which of the following a. With management
items is not essential? b. With the independent auditor
a. Assessment of inherent risk c. Equally with management and the auditor
b. The preparation of a budget identifying the costs of d. With the internal audit department.
resources needed
c. A review of material from prior audits 92. Audit standards require an auditor to:

AT – Prelim Rev (875 MCQs) Red Sirug Page 43 of 77


a. Perform procedures that are designed to detect all about a set of financial statements.
instances of fraud. d. The role of the independent auditor is to gain sufficient
b. Provide reasonable assurance that the financial appropriate evidence so as to provide reasonable
statements are not materially misstated. assurance that material misstatements do not exist in any
c. Issue an unqualified opinion only when the auditor is of the assertions made by management.
satisfied that no instances of fraud have occurred.
d. Design the audit program to meet financial statement 98. Which of the following activities is not prohibited for the CPA
users' expectations concerning fraud. firm's attestation service clients?
a. Referral fees on audit jobs.
93. Generally, the decision to notify parties outside the client's b. Competitive bidding on audit jobs.
organization regarding an illegal act is the responsibility of the c. Contingent fees on audit jobs.
a. Outside legal counsel. d. Commissions for obtaining client services on audit jobs.
b. Independent auditor.
c. Management. 99. Family and personal relationships between a member of the
d. Internal auditors. assurance team and a director, an officer or certain
employees, depending on their role, of the assurance client,
94. If requested to perform a review engagement for a nonpublic least likely create
entity in which an accountant has an immaterial direct financial a. Self-review threat.
interest, the accountant is b. Self-interest threat.
a. Independent because the financial interest is immaterial c. Intimidation threat.
and, therefore, may issue a review report. d. Familiarity threat.
b. Not independent and, therefore, may not issue a
review report. 100. The Code of Professional Ethics states, in part, that a CPA
c. Not independent and, therefore, may not be associated should maintain integrity and objectivity. Objectivity refers to
with the financial statements. the CPA's ability to
d. Not independent and, therefore, may issue a review a. Insist on all matters regarding audit procedures.
report, but may not issue an auditor's opinion. b. Determine the materiality of items.
c. Determine accounting practices that were consistently
95. Solicitation consists of the various means that CPA firms use applied.
to engage new clients. Which one-of the following would not d. Maintain an impartial attitude on all matters which
be an example of solicitation? come under his review.
a. Advertisements in the yellow pages of a phone book.
b. Accepting new clients that approach the firm.
c. Taking prospective clients to lunch. SET 6
d. Offering seminars on current tax law changes to potential
clients. 1. Conflict between financial statement users and auditors often
arises because of the
96. Quality control procedures are applicable to the individual a. High cost of performing an audit.
audit engagement. The implementation of such quality control b. Expectation gap.
procedures is responsibility of the: c. Technical vocabulary which the auditor uses in the report.
a. CPA firm. d. Placement of the auditor's report in the back of the
b. Engagement team. client's annual report where it is hard to locate.
c. Quality control reviewer.
d. Manager assigned to the engagement. 2. Which of the following is best considered a fraud?
a. Inability to provide due diligence.
97. Which of the following statements is incorrect? b. Intentional misrepresentation of financial information.
a. In an audit, the independent auditor attempts to c. Declining to finish work on client in light of a valid contract.
corroborate assertions made by the company’s d. Not acting professionally while performing services.
management in connection with each account, class of
transactions, and disclosures found in a set of financial 3. Anyone identified to the auditor by name prior to the audit who
statements. is to be the principal recipient of the auditor's report is a
b. Because of the attest function, financial statements a. Primary beneficiary.
are the responsibility of the independent auditor. b. Third party.
c. The term “materiality” refers to any factor of a size or type c. Foreseen beneficiary.
that would impact an outside decision-maker’s decision d. Secondary beneficiary.

AT – Prelim Rev (875 MCQs) Red Sirug Page 44 of 77


statements or preparation of other records that are the
4. A director, an officer or an employee of the assurance client in subject matter of the assurance engagement.
a position to exert direct and significant influence over the c. A member of the assurance team being, or having
subject matter of the assurance engagement has been a recently been, an employee of the assurance client in a
member of the assurance team or partner of the firm. This position to exert direct and significant influence over the
situation least likely create subject matter of the assurance engagement.
a. Self-interest threat. d. Performing services for an assurance client that directly
b. Advocacy threat. affect the subject matter of the assurance engagement.
c. Intimidation threat.
d. Familiarity threat. 10. A CPA-lawyer, acting as a legal counsel to one of his audit
client, is an example of
5. A former officer, director or employee of the assurance client a. Advocacy threat
serves as a member of the assurance team. This situation will b. Familiarity threat
least likely create c. Self-interest threat
a. Self-interest threat. d. Self-review threat
b. Self-review threat.
c. Familiarity threat. 11. Examples of circumstances that may create familiarity threat
d. Intimidation threat. least likely include
a. A former partner of the firm being a director, officer of the
6. A professional accountant has a professional duty or right to assurance client or an employee in a position to exert
disclose confidential information in each of the following, direct and significant influence over the subject matter of
except: the assurance engagement.
a. To disclose to BIR fraudulent scheme committed by b. Dealing in, or being a promoter of, share or other
the client on payment of income tax. securities in an assurance client.
b. To comply with technical standards and ethics c. A member of the assurance team having an immediate
requirements. family member or close family member who is a director
c. To comply with the quality review of a member body or or officer of the assurance client.
professional body d. A member of the assurance team having an immediate
d. To respond to an inquiry or investigation by a member family member or close family member who, as an
body or regulatory body. employee of the assurance client, is in a position to exert
direct and significant influence over the subject matter of
7. Which of the following is not likely a threat to independence? the assurance engagement.
a. Acting as an advocate on behalf of an assurance client in
litigation or in resolving disputes with third parties. 12. The following are modifications made to the IFAC Code to
b. Long association of a senior member of the assurance consider Philippine regulatory requirements and
team with the assurance client. circumstances, except
c. Threat of replacement over a disagreement with the a. The period for rotation of the lead engagement
application of an accounting principle. partner was changed from five to seven years.
d. Owning immaterial indirect financial interest in an b. Advertising and solicitation by individual professional
audit client. accountants in public practice were not permitted in the
Philippines.
c. Additional examples relating to anniversaries and
8. Examples of circumstances that may create self-interest threat websites
include: wherein publicity is acceptable, as provided in boa
a. Contingent fees relating to assurance engagements. resolution 19, series of 2000, were included.
b. A direct financial interest or material indirect financial d. Payment and receipt of commissions were not permitted
interest in an assurance client. in the Philippines.
c. A loan or guarantee to or from an assurance client or any
of its directors or officers.
d. All of the above 13. The network firms are required to be independent of the client
a. For assurance engagements provided to an audit
9. Examples of circumstances that may create self-review threat client.
least likely include b. For assurance engagements provided to clients that are
a. Potential employment with an assurance client. not audit clients, when the report is not expressly
b. Preparation of original data used to generate financial restricted for use by identified users.

AT – Prelim Rev (875 MCQs) Red Sirug Page 45 of 77


c. For assurance engagements provided to clients that are weak internal control.
not audit clients, when the assurance report is expressly
restricted for use by identified users. 18. Which of the following statements is correct?
d. All of the above a. Sufficiency refers to the quality of evidence, while
appropriateness refers to the quantity of evidence.
14. Which of the following is incorrect regarding independence? b. The reliability of evidence is influenced not by its nature
a. Independence consists of independence of mind and but by its source.
independence in appearance. c. The performance of consulting services for audit
b. Independence is a combination of impartiality, clients does not, in and of itself, impair the auditor’s
intellectual honesty and a freedom from conflicts of independence.
interest. d. A belief that management and those charged with
c. Independence of mind is the state of mind that permits governance are honest and have integrity relieves the
the provision of an opinion without being affected by auditor of the need to maintain professional scepticism.
influences that compromise professional judgment,
allowing an individual to act with integrity, and exercise 19. Which of the following statements is correct?
objectivity and professional skepticism. a. The fair presentation of audited financial statements in
d. Independence in appearance is the avoidance of facts accordance with applicable financial reporting framework
and circumstances that are so significant a reasonable is an implicit part of the auditor’s responsibility.
and informed third party, having knowledge of all relevant b. Professional judgment can be used as the justification for
information, including any safeguards applied, would the decisions made by the auditor that are not otherwise
reasonably conclude a firm's or a member of the supported by the facts and circumstances of the
assurance team's integrity, objectivity or professional engagement or sufficient appropriate evidence.
skepticism had been compromised. c. Appropriateness is the measure of the quality of evidence,
that is, its reliability and persuasiveness.
15. A CPA firm is considered independent when it performs which d. Most CPAs, including those who provide audit and
of the following services for a publicly-traded audit client? tax services, also provide consulting services to their
a. Serving as a member of the client's board of directors. clients.
b. Determining which accounting policies will be adopted by
the client. 20. Individual CPAs, Firms or Partnerships of CPAs, including
c. Accounting information system design and partners and staff members thereof shall register with the BOA
implementation. and the PRC. Assuming that the application for registration of
d. Tax return preparation as approved by the board of Sisip and Co., CPAs was approved on August 30, 2005, which
directors. of the following is true?
a. The registration will expire on Dec. 31, 2007.
16. In connection with the examination of financial statements, an b. The registration must be renewed on September 30,
independent auditor could be responsible for failure to detect a 2007.
material fraud if c. The registration will expire on August 30, 2007 since the
a. Statistical sampling techniques were not used on the validity of the certificate of registration is three years.
audit engagement. d. The registration will expire on Dec. 31, 2007 which is also
b. The fraud was perpetrated by one client employee, who the last day of renewal of certificate of registration.
circumvented the existing internal control.
c. The auditor planned the work in a hasty and 21. Auditor’s responsibility for detecting noncompliance is limited
inefficient manner. to those:
d. Accountants performing important parts of the work failed a. Direct-effect noncompliance.
to discover a close relationship between the treasurer b. Material direct-effect noncompliance.
and the cashier. c. Material indirect-effect noncompliance.
d. All noncompliance since they affect the financial
17. Which of the following conditions suggests auditor negligence? statements directly or indirectly.
a. Failure to detect collusive fraud perpetrated by members
of middle management. 22. Auditors would perform the following steps in which order?
b. Failure to detect collusive fraud perpetrated by members a. Determine audit risk; assess control risk; determine
of top management. detection risk; set materiality.
c. Failure to detect errors occurring outside the internal b. Set materiality; assess control risk; determine detection
control structure. risk; determine audit risk.
d. Failure to detect material errors under conditions of c. Set materiality; determine audit risk; assess control

AT – Prelim Rev (875 MCQs) Red Sirug Page 46 of 77


risk; determine detection risk. a. The auditor should apply judgment in determining the
d. Determine audit risk; set materiality; assess control risk; specific nature, timing and extent of review procedures.
determine detection risk. b. The auditor should apply the same materiality
considerations as would be applied if an audit opinion on
23. Which of the following statements is an not an example of an the financial statements were being given.
inherent limitation of internal control? c. There is a greater risk that misstatements will not be
a. The effectiveness of control procedures depends on detected in an audit than in a review.
segregation of duties. d. The judgment as to what is material is made by reference
b. Errors may arise from mistakes in judgments. to the information on which the auditor is reporting and
c. Most internal controls tend to be directed at routine the needs of those relying on that information, not to the
transactions rather than non-routine transactions. level of assurance provided.
d. The cost-benefit relationship is a primary criterion in
designing internal control, 30. Which of the following is not a document or record that should
be examined early in the engagement?
24. Compliance with the independence requirement is necessary a. Corporate charter and by-laws.
whenever a CPA performs: b. Contracts.
a. Non-assurance services c. Management letter.
b. Professional services d. Minutes of board of directors' and stockholders' meetings.
c. Tax consultancy services
d. Assurance services 31. Philippine Standards on Auditing require auditors to assess
the risk of material misstatements due to fraud
25. It is acceptable for the auditor to prepare: a. For first-time audits.
a. The financial statements for the client. b. Sufficient to find any frauds which may exist.
b. The notes to financial statements for the client. c. For every audit.
c. A draft of the financial statements for the client. d. Whenever it would be appropriate.
d. A draft of the financial statements and notes to the
financial statements for the client. 32. In connection with the examination of financial statements, an
independent auditor could be responsible for failure to detect a
26. When an accountant performs more than one level of service, material fraud if
he generally should issue a report that is appropriate for: a. Accountants performing important parts of the work failed
a. The lowest level of service rendered. to discover a close relationship between the treasurer
b. A compilation engagement. and the cashier.
c. The highest level of service rendered. b. The auditor planned the work in a hasty and
d. A review engagement. inefficient manner.
c. Statistical sampling techniques were not used on the
27. An accountant who reviews the financial statements should audit engagement.
issue a report stating that a review d. The fraud was perpetrated by one client employee, who
a. Is substantially less in scope than an audit. circumvented the existing internal control.
b. Provides negative assurance that the internal control is
functioning as designed. 33. Which of the following is not true regarding planning in an
c. Provides only a limited assurance that the financial electronic environment?
statements are fairly presented. a. The definition of auditing is not changed
d. Is substantially more in scope than a compilation b. The purposes of auditing is not changed
c. The procedures used are not changed
28. Which of the following is required to be performed in an audit d. Auditing standards are not changed
but not in review engagement?
a. Complying with the "Code of Professional Ethics for 34. Which of the following concepts of materiality is incorrect?
Certified Public Accountants" promulgated by the Board a. Materiality is based on quantitative and non-quantitative
of Accountancy factors.
b. Planning the engagement b. Materiality is a matter of professional audit judgment.
c. Agreeing on the terms of engagement c. Materiality does not apply if internal control is highly
d. Studying and evaluating internal control structure effective.
d. Materiality is more closely related to the fieldwork and
29. Which statement is incorrect regarding procedures and reporting standards than to general standards.
evidence obtained in a review engagement?

AT – Prelim Rev (875 MCQs) Red Sirug Page 47 of 77


35. Which of the following statements is incorrect about materiality? b. Inherent risk.
a. The concept of materiality recognizes that some matters c. Control risk.
are important for fair presentation of financial statements d. Detection risk.
in conformity with GAAP, while other matters are not
important. 41. Which of the following does an auditor least likely perform in
b. An auditor considers materiality for planning assessing audit risk?
purposes in terms of the largest aggregate level of a. Understand the economic substance of significant
misstatements that could be material to any one of transactions completed by the client.
the financial statements. b. Understand the entity and the industry in which it
c. Materiality judgments are made in light of surrounding operates.
circumstances and necessarily involve both quantitative c. Gather audit evidence in support of recorded
and qualitative judgments. transactions.
d. An auditor’s consideration of materiality is influenced by d. Obtain an understanding of the client's system of internal
the auditor’s perception of the needs of a reasonable control.
person who will rely on the financial statements.
42. Which of the following is most likely to be an overall response
36. After discovering that a related-party transaction exists, the to fraud risks identified in an audit?
auditor should be aware that the a. Supervise members of the audit team less closely and
a. Transaction is assumed to be outside the ordinary course rely more upon judgment.
of business. b. Use less predictable audit procedures.
b. Substance of the transaction could be significantly c. Use only certified public accountants on the engagement.
different from its form. d. Place increased emphasis on the audit of objective
c. Adequacy of disclosure of the transaction is secondary to transactions rather than subjective transactions.
its legal form.
d. Financial statements should recognize the legal form of 43. If the auditor is convinced that the client has an excellent
the transaction rather than its substance. internal control structure, the amount of audit evidence to be
gathered.
37. Auditors focus on a. Can be significantly less than where internal control
a. Areas where the risk of material errors and irregularities is not adequate.
is least. b. Will not be affected since the auditor must arrive at an
b. Areas where the risk of material errors and independently determined opinion.
irregularities is greatest. c. Must be increased to support the auditor's belief.
c. All areas equally. d. Is not determinable.
d. A random selection of all areas.
44. Why should the auditor plan more work on individual accounts
38. Audit risk components consist of inherent, control and as lower acceptable levels of both audit risk and materiality
detection risks. Which of them is are dependent variable(s)? are established?
a. Inherent risk a. To find smaller errors
b. Control risk b. To find larger errors
c. Detection risk c. To increase the tolerable error in the accounts
d. Inherent and control risks d. To decrease the risk of overreliance

39. The risk that the audit will fail to uncover a material 45. Which of the following most accurately summarizes what is
misstatement is eliminated meant by the term “material misstatement?”
a. When the auditor has complied with generally accepted a. Fraud and direct-effect illegal acts.
auditing standards. b. Fraud involving senior management and material fraud.
b. If a client has strong internal controls. c. Material error, material fraud, and certain illegal acts.
c. If a client follows generally accepted accounting d. Material error and material illegal acts.
principles.
d. Under no circumstances. 46. The risk of fraudulent financial reporting increases in the
presence of
40. The probability of an auditor's procedures leading to the a. Substantial increases in sales.
conclusion that a material error does not exist in an account b. Incentive systems based on operating income.
balance when, in fact, such error does exist is referred to as c. Improved control systems.
a. Prevention risk. d. Frequent changes in suppliers.

AT – Prelim Rev (875 MCQs) Red Sirug Page 48 of 77


a. Before June 30. 2008
47. Which of the following is most likely to be considered a risk b. After June 30, 2008
factor relating to fraudulent financial reporting? c. Before January 1,2008
a. Domination of management by top executives. d. After January 1, 2008
b. Negative cash flows from operations.
c. Large amounts of cash processed. 53. Which of the following is not explicitly referred to in the Code
d. Small high-dollar inventory items. of Ethics as source of technical standards?
a. Commission on Audit (COA)
48. When planning the audit, if the auditor has no reason to b. Auditing and Assurance Standards Council (AASC)
believe that illegal acts exist, the auditor should c. Securities and Exchange Commission (SEC)
a. Make inquiries of management regarding their d. Relevant legislation
policies and their knowledge of violations, and then
rely on normal audit procedures to detect errors, 54. In designing written audit programs, an auditor should
irregularities, and illegalities. establish specific audit objectives that relate primarily to the
b. Still include some audit procedures designed specifically a. Financial statement assertions.
to uncover illegalities. b. Timing of audit procedures.
c. Ignore the topic. c. Cost-benefit of gathering evidence.
d. Include audit procedures which have a strong probability d. Selected audit techniques.
of detecting illegal acts.
55. An audit program provides proof that
49. Which statement best describes the emphasis of the systems a. Sufficient appropriate evidence was obtained.
and substantive approaches in the audit plan? b. The work was adequately planned.
a. A thoroughly designed systems approach to auditing can c. There was a proper study and evaluation of internal
eliminate the need for any substantive procedures. control.
b. The systems approach focuses on detailed testing of d. There was compliance with generally accepted standards
specific accounts for accuracy, while the substantive of reporting.
approach is the testing controls to make sure they are
effective. 56. The audit program usually cannot be finalized until the
c. The systems approach focuses on testing controls to a. Reportable conditions have been communicated to the
make sure they are effective, while the substantive audit committee of the board of directors.
approach is the detailed testing of specific accounts b. Engagement letter has been signed by the auditor and
for accuracy. the client.
d. The systems approach focuses on the use of computer c. Consideration of the entity’s internal control has
systems to aid in the audit while the substantive approach been completed.
focuses on more manual tests. d. Search for unrecorded liabilities has been performed and
documented.
50. Audit programs should be designed so that
a. Most of the required procedures can be performed as 57. A person or firm possessing special skill, knowledge and
interim work. experience in a particular field excluding accounting and
b. The audit evidence gathered supports the auditor’s auditing.
conclusions. a. Quality control reviewer
c. Inherent risk is assessed at a sufficiently low level. b. Multiskilled personnel
d. The auditor can make constructive suggestions to c. Expert
management. d. Taxation specialist

51. The decision as to how much evidence to be accumulated for 58. The Code of Ethics for Professional Accountants in the
a given set of circumstances is: Philippines is applicable to professional services in the
a. Provided in the Philippines Standards on Auditing. Philippines on or before:
b. Provided by following GAAP a. December 31, 2009
c. One requiring professional judgment b. July 1, 2004
d. Determined by statistical analysis c. June 30, 2008
d. June 30, 2009
52. The revised Code of Ethics is mandatory for all CPAs and is
applicable to professional services performed in the 59. Which part of the Code of Ethics applies to professional
Philippines on or: accountants in public practice?

AT – Prelim Rev (875 MCQs) Red Sirug Page 49 of 77


a. Part A b. Be relied upon to disclose violations of truth-in-lending act.
b. Part B c. Encompass a plan to search actively for illegalities which
c. Part A and Part B relate to operating aspects.
d. Part C d. Not be relied upon to provide assurance that illegal
acts will be detected.
60. This fundamental ethical principle prohibits association of
professional accountants with reports, returns, 66. Which of the following statements is true?
communications and other information that contains materially a. It is usually equally difficult for the auditor to uncover
false or misleading information or statements. errors or irregularities.
a. Integrity b. It is usually easier for the auditor to uncover irregularities
b. Objectivity than errors.
c. Professional competence and due care c. It is usually easier for the auditor to uncover errors
d. Professional behavior than irregularities.
d. Usually, none of the above statements is true.
61. The principle professional competence and due care imposes
which of the following obligations on professional accountants? 67. The audit should not assume that management is dishonest,
a. To maintain professional knowledge and skill at the but the possibility of dishonesty must be considered." This is
level required to ensure that a client or employer an example of
receives competent professional service. a. Due diligence.
b. To comply with relevant laws and regulations and avoid b. Unprofessional behavior.
any action that discredits the profession. c. An attitude of professional skepticism.
c. Not to override or compromise his professional or d. An ethical requirement.
business judgment because of bias, conflict of interest or
undue influence of others. 68. In discovering material management fraud and an equally
d. To be fair and truthful. material error, the audit plan
a. Cannot be expected to provide the same degree of
62. Attainment of professional competence requires the following, assurance.
except: b. Provide no assurance of detecting either.
a. Initially, a high standard of education. c. Should be expected to provide the same degree of
b. Specific education, training, and examination in assurance.
professionally relevant subjects. d. Should provide complete assurance of detection.
c. Whether prescribed or not, a period of work experience.
d. A continuing awareness and an understanding of 69. In comparing management fraud with employee fraud, the
relevant technical professional and business auditor's risk of failing to discover the fraud is
developments. a. Greater for employee fraud because of the larger number
of employees in the organization.
63. A draft of statement, studies or standards should be discussed b. Greater for employee fraud because of the higher crime
by the Council en banc. How many members of the AASC are rate among blue collar workers.
required to approve the draft for exposure? c. Greater for management fraud because of
a. Eight management's ability to override existing internal
b. Ten controls.
c. Twelve d. Greater for management fraud because managers are
d. Majority inherently smarter than employees.

64. How many members of the AASC are needed to approved the 70. The risk that the audit will fail to uncover a material
exposed draft as PSA misstatement is eliminated
a. At least 8 a. When the auditor has complied with generally accepted
b. At least 10 auditing standards.
c. At least 12 b. If client has good internal control.
d. Majority of the regular members c. If client follows generally accepted accounting principles.
d. Under no circumstances.
65. An auditor's examination performed in accordance with
generally accepted auditing standards generally should 71. The practitioner’s report on an assurance engagement should
a. Be expected to provide assurance that illegal acts will be always include the following, except
detected where internal control is effective.

AT – Prelim Rev (875 MCQs) Red Sirug Page 50 of 77


a. A description of the engagement and identification of the a. A sole proprietor providing professional services to a
subject matter client.
b. Identification of the standards under which the b. Each partner or person occupying a position similar to
engagements was conducted. that of a partner staff in a practice providing professional
c. Identification of the criteria. services to a client.
d. Reference to the work of an expert. c. Professional accountants employed in the public
sector having managerial responsibilities.
72. Which of the following is required if the professional d. A firm of professional accountants in public practice.
accountant uses experts who are not professional accountants?
a. The professional accountant is discouraged to engage 77. The term receiving accountant includes the following, except;
the services of experts who are not a professional a. A professional accountant in public practice to whom
accountant. existing accountant has referred tax engagement.
b. The ultimate responsibility for the professional service is b. A professional accountant in public practice to whom the
assumed by the expert who is not a professional client of the existing accountant has referred audit
accountant. engagement.
c. The professional accountant must take steps to see c. A professional accountant in public practice who is
that such experts are aware of the ethical consulted in order to meet the needs of the client.
requirements of the profession. d. A professional accountant in public practice
d. Experts who are not professional accountants need not currently holding an audit appointment or carrying
be informed of ethical requirements because they are not out accounting, taxation, consulting or similar
members of the Accountancy profession. professional services for a client.

73. Which of the following is expected of AASC to do? 78. A primary purpose for establishing a code of ethics within a
a. AASC should normally expose its opinion on specific professional organization is to:
queries from a practicing CPA. a. Demonstrate the acceptance of responsibility Jo the
b. AASC should normally expose a proposed interpretation interest of those served by the profession.
of statements. b. Reduce the likelihood that members of the profession will
c. To make the statements on Philippine Standards on be sued for substandard work.
Auditing operative, the final statement shall be c. Ensure that all members of the profession posses
submitted to the Board of Accountancy for approval. approximately the same level of competency.
d. When it is deemed necessary to expose a statement for a d. Require members of the profession to exhibit loyalty in all
comment on proposed interpretations of statements, the matters pertaining to the affairs of the organization.
exposure period is understandably shorter than those of
the regular drafts of standards. 79. The communication to the public of facts about a professional
accountant which are not designed for the deliberate
74. Required auditor communication to the Audit Committee promotion of that professional accountant.
concerning noncompliance with laws and regulations that a. Publicity
were detected includes: b. Indirect promotion
a. All those which are not adequately addressed by c. Advertising
management. d. Solicitation
b. All those that constitute management fraud.
c. All material items. 80. Advertising, as defined in the Code of Ethics, means
d. Any of such acts. a. The communication to the public of facts about a
professional accountant which are not designed for the
75. Firm includes the following except: deliberate promotion of that professional accountant.
a. A sole practicing professional accountant, b. The approach to a potential client for the purpose of
b. An entity that controls a partnership of professional offering professional services.
accountants. c. The communication to the public of information as to
c. An entity controlled by a partnership of professional the services or skills provided by professional
accountants. accountants in public practice with a view to
d. A sole practitioner, partnership or corporation of procuring professional business.
professional accountants. d. Any of the given choices.

76. The term professional accountant in public practice includes 81. Which of the following is least likely the basis of determining
the following, except: audit fees?

AT – Prelim Rev (875 MCQs) Red Sirug Page 51 of 77


a. The skill and knowledge required for the type of work
involved. 87. Which of the following has primary responsibility for the
b. The degree of responsibility and urgency that the work performance of an audit?
entails. a. The partner in charge of the engagement
c. The expected outcome of the engagement. b. The senior assigned to the engagement
d. The required level of training and experience of the c. The managing partner of the firm
person engaged on the work. d. The manager assigned to the engagement

82. Though PSAs do not provide "hard and fast rules," they 88. The most common type of audit report contains a(n):
provide subjective guidance which allow the auditors to: a. Unqualified opinion.
a. Only apply those standards that are important to the audit. b. Qualified opinion.
b. Use adequate professional judgment when applying c. Adverse opinion.
the standards. d. Disclaimer of opinion.
c. Tailor their audit to procedures requested by
management. 89. Assurance services involve all the following except:
d. Accurately interpret the Code of Ethics for CPAs. a. Improving the quality of information for decision purposes.
b. Improving the quality of the decision model used.
83. Which one of the following is not a key attribute that is c. Improving the relevance of information.
essential to perform an assurance service? d. Implementing a system that improves the processing
a. Independence of information.
b. Accounting skills
c. Established criteria or standards 90. Which of the following is the broadest and most inclusive
d. Subject matter knowledge concept?
a. Audits of financial statements.
84. Upon completion of a typical audit, the auditor has b. Internal control audit.
a. No assurance that all material errors and fraud have been c. Assurance services.
found. d. Compilation services.
b. A low level of assurance that all material errors and fraud
have been found. 91. Whenever a CPA professional is engaged to perform an audit
c. High level of assurance that all material errors and of financial statements according to Philippine Standards on
fraud have been found. Auditing, he required to comply with those standards in order
d. Total assurance that al material errors and fraud have to
been found. a. Eliminate audit risk.
b. Eliminate the professional judgment in resolving audit
85. An investor, while reading the financial statements of Star issues.
Corporation, learned that the statements are accompanied by c. Have a measure of the quality of audit performance.
an unqualified auditor's report. From this the investor may d. To reduce the audit program to be prepared by the
conclude that: auditor.
a. The auditor has ascertained that Silver's financial
statements have been prepared accurately. 92. Certain fundamental beliefs called "postulates" underlie
b. The auditor is satisfied that Silver is operationally efficient. auditing theory. Which of the following is not a postulate of
c. Any disputes over significant accounting issues have auditing?
been settled to the auditor's satisfaction. a. Economic assertions can be verified.
d. Informative disclosures in the financial statements but not b. The auditor acts exclusively as an auditor.
necessarily in the notes to financial statements are to be c. An audit has a benefit only to the owners.
regarded as reasonably adequate. d. No long-term conflict exists between the auditor and the
management of the entity under audit.
86. A CPA should maintain objectivity and be free of conflicts of
interest when performing: 93. In all cases, audit reports must
a. All attestation services, but not other professional a. Communicate the auditor’s finding to the general public.
services. b. Be signed by the individual who performed the audit
b. All attestation and tax services, but not other professional procedures.
services. c. Certify the accuracy of the quantitative information which
c. Audits, but not any other professional services. was audited.
d. All professional services.

AT – Prelim Rev (875 MCQs) Red Sirug Page 52 of 77


d. Inform readers of the degree of correspondence
between the quantifiable information and the 100. Examples of circumstances that may create familiarity threat
established criteria. least likely include
a. A member of the assurance team having an immediate
94. The framework for auditing and related services as addressed family member or close family member who is a director
by PSA excludes or officer of the assurance client.
a. Review b. A member of the assurance team having an immediate
b. Compilation family member or close family member who, as an
c. Tax services employee of the assurance client, is in a position to exert
d. Agreed upon procedure direct and significant influence over the subject matter of
the assurance engagement.
95. It refers to the level of auditor’s satisfaction as to the reliability c. A former partner of the firm being a director, officer of the
of an assertion being made by one party for use by another assurance client or an employee in a position to exert
party. direct and significant influence over the subject
a. Confidence level matter of the assurance engagement.
b. Assurance level d. Dealing in, or being a promoter of, share or other
c. Reasonableness level securities in an assurance client.
d. Tolerable level
SET 7
96. Which of the following is true of the report based on agreed-
upon-procedures? 1. In "auditing" accounting data, the auditor is concerned with
a. The report is restricted to those parties who have a. Analyzing the financial information to be sure that it
agreed to the procedures to be performed. complied with government requirement.
b. The CPA provides the recipients of the report limited b. Determining if fraud has occurred.
assurance as to reasonableness of the assertion(s) c. Determining whether recorded information properly
presented in the financial information. reflects the economic events that occurred during the
c. The report states that the auditor has not recognized any accounting period.
basis that requires revision of financial statements. d. Determining if taxable income has been calculated
d. The report should state that the procedures performed correctly.
are limited to analytical procedures and inquiry.
2. Which one of the following is an example of management
97. The three types of attestation services are: expectations from the independent auditors?
a. Audits, review, and compilations a. An expert providing a written communication as the
b. Reviews, compilations, and other attestation services product of the engagement.
c. Audits, compilations, and other attestation services b. Individuals who perform day-to-day accounting functions
d. Audits, reviews, and other attestation services on behalf of the company.
c. AN active participant in management decision-making.
98. Which of the following types of audit uses as its criteria laws d. An internal source of expertise on financial and other
and regulations? matters.
a. Financial statement audit
b. Operational audit 3. Broadly defined, the subject matter of any audit consists of
c. Compliance audit a. Financial statements.
d. Financial audit b. Assertions.
c. Operating data.
99. Which of the following least likely limits the auditor’s ability to d. Economic data.
detect material misstatement?
a. The inherent limitations of any accounting and internal 4. An engagement in which a CPA firm arranges for a critical
control system. review of its practices by another CPA firm.
b. Most audit evidences are conclusive rather than a. Attestation engagement
being persuasive. b. Peer review engagement
c. Audit is based on testing c. Quality control engagement
d. Audit procedures that are effective in detecting ordinary d. Quality assurance engagement
misstatements are ineffective in detecting intentional
misstatements. 5. The risk associated with a company's survival and profitability
is referred to as:

AT – Prelim Rev (875 MCQs) Red Sirug Page 53 of 77


a. Information risk c. Disregard independence in order to find the underlying
b. Business risk truth of the evidence.
c. Control risk d. Establish new criteria by which financial statements may
d. Detection risk be compared.

6. An operational audit differs in many ways from an audit of 11. Which one of the following is not a part of the attest process?
financial statements. Which of the following is the best a. Evaluating evidence against objective criteria
example of these differences? b. Gathering evidence about assertions
a. Operation audits do not ordinarily result in the preparation c. Proving the accuracy of the books and records
of a report. d. Communicating the conclusions reached
b. The usual audit of financial statement covers the four
basic financial statements whereas the operational audit 12. Which one of the following is not a reason why the users of
is usually limited either the balance sheet or the income financial statements desire for an independent assessment of
statement. the financial statement presentation?
c. The boundaries of an operation audit are often drawn a. Complexity of transactions affecting the financial
from an organization chart and are not limited to a statements
single accounting period. b. Lack of criteria on which to base information
d. The operational audit deals with operating profit while c. Remoteness of the user from the organization
financial audit considers both the operating and net d. All of them are potential reasons
profits.
13. An audit which determines whether organizational policies are
7. The audit of historical financial statements should be being followed and whether external mandates are being met
conducted by the CPA professionals in accordance with is referred to as
a. The auditor's judgment. a. A financial audit
b. The audit program. b. A compliance audit.
c. Philippine Financial Reporting Standards. c. An operational audit.
d. Philippine Standards on Auditing. d. None of the above

8. In determining the primary responsibility of the external auditor 14. Which of the following factors most likely would cause a CPA
for an audit of a company's financial statements, the auditor to decline a new audit engagement?
owes primary allegiance to: a. The CPA does not understand the entity's operations and
a. The management of the audit client because the auditor industry.
is hired and paid by management. b. The CPA is unable to review the predecessor auditor's
b. The audit committee of the audit client because that working papers.
committee is responsible for coordinating and reviewing c. Management is unwilling to permit inquiry of its legal
all audit activities within the company. counsel.
c. Stockholders, creditors, and the investing public. d. Management acknowledges that the entity has had
d. The Auditing and Assurance Standards Council, because recurring operating losses.
it determines auditing standards and auditor's
responsibility. 15. Auditing standards are
a. Statutory in nature.
9. Assurance services may include which of the following? b. Rules imposed by the Securities and Exchange
a. Attesting to financial statements Commission.
b. Examination of the economy and efficiency of c. General guidelines to help auditors.
governmental operations d. Rules imposed by the PICPA.
c. Evaluation of a division's performance for management
d. All of the given choices 16. Which of the following best describes what is meant by
generally accepted auditing standards?
10. The auditor of financial statements must make very difficult a. Pronouncements issued by the Auditing and Assurance
interpretations regarding authoritative literature. Additionally, Standards Council.
the auditor must b. Rules acknowledged by the accounting profession
a. Proceed beyond PFRS to assess how the economic because of their universal compliance.
activity is portrayed in the financial statements. c. Procedures to be used to gather evidence to support
b. Force management to make certain decisions regarding financial statements.
their financial statements. d. Measures of the quality of the auditor's performance.

AT – Prelim Rev (875 MCQs) Red Sirug Page 54 of 77


d. Examine all available corroborating evidence.
17. Generally Accepted Auditing Standards (GAAS) and
Philippine Standards on Auditing (PSA) should be looked 24. The exercise of due professional care requires that an auditor
upon by practitioners as a. Use error-free judgment.
a. Ideals to work towards, but which are not achievable. b. Consider internal control, including tests of controls.
b. Benchmarks to be used on all audits, reviews, and c. Critically review the work done at every level of
compilations. supervision.
c. Maximum standards which denote excellent work. d. Examine all corroborating evidence available.
d. Minimum standards of performance which must be
achieved on each audit engagement. 25. An auditor who accepts an audit engagement and does not
possess the industry expertise of the business entity, should
18. Competence as a certified public accountant includes all of the a. Engage financial experts familiar with the nature of the
following except business entity.
a. Consulting others if additional technical information is b. Obtain a knowledge of matters that relates to the
needed. nature of the entity's business.
b. Possessing the ability to supervise and evaluate the c. Refer a substantial portion of the audit to another CPA
quality of staff work. who will act as the principal auditor.
c. Having the technical qualifications to perform an d. First inform management that an unqualified opinion
engagement. cannot be issued.
d. Warranting the infallibility of the work performed.
26. Which of the following underlies the application of generally
19. Which one of the following attributes is required of an auditor accepted auditing standards, particularly the standards of
in relation to audit clients? fieldwork and reporting?
a. Loyalty a. Element of corroborating evidence
b. Rationalization b. Element of reasonable assurance
c. Independence c. Elements of materiality and risk
d. Bias d. Element of internal control

20. To be independent, the auditor: 27. Which of the following is the authoritative body designated to
a. Must be impartial when dealing with the client. promulgate auditing standards?
b. Cannot place any reliance on the client's verbal and a. Financial Reporting Standards Council
written assertions. b. PICPA
c. Is responsible only to third-party users of the financial c. Association of CPAs in Public Practice and PICPA
statements. d. Auditing and Assurance Standards Council
d. Cannot perform any consulting services for an audit client.
28. Which of the following mostly describes the function of AASC?
21. Practitioner's independence: a. To monitor full compliance by auditors to PSAs.
a. Minimizes risk. b. To assist the Board of Accountancy in conducting
b. Defends against liability administrative proceedings on erring CPAs in audit
c. Helps achieve public confidence practice.
d. Achieves compliance with the standards of fieldwork. c. To promulgate auditing standards, practices and
procedures that shall be generally accepted by the
22. The exercise of due professional care requires that an auditor accounting profession in the Philippines.
a. Uses error-free judgment. d. To undertake continuing research on both auditing and
b. Considers internal control, including tests of controls. financial accounting in order to make them responsive to
c. Examines all corroborating evidence available. the needs of the public.
d. Be responsible for fulfilling his or her duties
diligently and carefully. 29. The Philippine Standards on Auditing issued by the Auditing
and Assurance Standards Council (AASC)
23. The exercise of due professional care requires that an auditor a. Are interpretations of generally accepted auditing
a. Critically review the judgment exercised at every standards
level of supervision. b. Are the equivalent of laws for audit practitioners.
b. Attain the proper balance of professional experience and c. Must be followed in all situations.
formal education. d. Are optional guidelines which an auditor may choose to
c. Reduce control risk below the maximum. follow or not follow when conducting an audit.

AT – Prelim Rev (875 MCQs) Red Sirug Page 55 of 77


applied uniformly.
30. Which of the following statements best describes the primary c. Auditing standards are applied uniformly but auditing
purpose of Philippines Standards on Auditing? procedures are optional.
a. They are authoritative statements, enforced through the d. Both auditing standards and auditing procedures are
Code of Ethics for Professional Accountants, that are applied uniformly.
intended to limit the degree of auditor judgment.
b. They are procedural outlines which are intended to 36. Generally accepted accounting principles (GAAP) are
narrow the areas of inconsistency and divergence of distinguished from generally accepted auditing standards
auditor opinion. (GAAS) in that:
c. They are guides intended to set forth auditing procedures a. GAAP are the principles auditors follow when conducting
that are applicable to a variety of situations. an audit, while GAAS are the standards for presentation
d. They are interpretations which are intended to clarify of financial statements and underlying transactions.
the meaning of "generally accepted auditing b. GAAP are the principles for presentation of financial
standards." statements and underlying transactions, while GAAS
are the standards that the auditors should follow
31. In financial statement audits, the audit process should be when conducting an audit.
conducted in accordance with c. When GAAP are violated, sufficiently strong GAAS may
a. Philippine Financial Reporting Standards make up for most GAAP deficiencies.
b. International Accounting Standards d. GAAP are promulgated by the SEC, while GAAS are
c. Philippine Standards on Auditing promulgated by the PFRC.
d. Philippine Accounting Standards
37. How frequent can a professional accountants have press and
32. The Philippine Standards on Auditing issued by AASC other media releases commemorating their anniversaries in
a. Need to be applied on all audit related. public practice by informing the public of their achievements or
b. Require that in no circumstances would an auditor may accomplishments in contributing toward nation building or
judge it necessary to depart from a PSA, even though enhancing the image or standards of the accounting
such a departure may result to more effective profession?
achievement of the objective of an audit. a. 2 years
c. Apply to independent examination of financial b. 3 years
statements of any entity when such an examination is c. 5 years
conducted for the purpose of expressing an opinion. d. 6 years
d. Must not apply to other related activities of auditors.
38. Which of the following is not allowed to be included in a
33. An auditor needs not abide by a Philippines Standard on website of a firm of professional accountants?
Auditing if the auditor believes that a. Names of partners/principals with their educational
a. The amount is insignificant. attainment.
b. The requirement of the PSA is impractical to perform. b. Membership in any professional body.
c. The requirement of the PSA is impossible to perform. c. Awards received
d. Any of the above three is correct. d. Listings of the firm's clients.

34. Auditing standards differ from auditing procedures in that 39. In their fiduciary role, the professional accountants owe their
procedures relate to: primary loyalty to:
a. Measures of performance a. The accounting profession
b. Acts to be performed. b. The general public
c. Audit judgments. c. The client
d. Audit principles. d. Government regulatory agencies

35. Every independent audit engagement involves both auditing 40. Which of the following is a distinguishing mark of the
standards and auditing procedures. The relationship between accountancy profession?
the two may be illustrated by how they apply from a. A drive to excellence
engagement to engagement. The best representation of this b. Acceptance of the responsibility to act in the public
application is that, from one audit engagement to the next, interest
a. Auditing standards are applied uniformly but auditing c. Professional objectivity
procedures may vary. d. Professional skepticism
b. Auditing standards may vary but auditing procedures are

AT – Prelim Rev (875 MCQs) Red Sirug Page 56 of 77


41. A professional accountant is likened to a prudent father to his
son. This relates to what fundamental principle? 46. Which of the following is incorrect regarding integrity and
a. Professional competence and due care objectivity?
b. Confidentiality a. Integrity implies not merely honesty but fair dealing and
c. Integrity truthfulness.
d. Objectivity b. The principle of objectivity imposes the obligation on all
professional accountants to be fair, intellectually honest
42. Which fundamental principle is seriously threatened by an and free of conflicts of interest.
engagement that is compensated based on the net proceeds c. Professional accountants serve in many different
on loans received by the client from a commercial bank? capacities and should demonstrate their objectivity in
a. Integrity varying circumstances.
b. Objectivity d. Professional accountants should neither accept nor
c. Professional behavior offer any gifts or entertainment.
d. Confidentiality
47. If a professional accountant is billing an audit client a number
43. Which of the following values is not necessary for a of hours greater than those actually worked, which of the
professional accountant? following fundamental principles is likely violated?
a. Honesty a. Objectivity
b. Objectivity b. Integrity
c. Integrity c. Professional due care
d. A primary commitment to self-interest d. Confidentiality

44. Professional accountants may encounter problems in 48. Which of the following is not a function of the Board of
identifying unethical behavior or in resolving an ethical conflict. Accountancy as specified in the Philippine Accountancy Act of
When faced with significant ethical issues, professional 2004?
accountants should do the following, except a. To investigate violations of the Accountancy Law and the
a. Follow the established policies of the employing rules and regulations promulgated therewith.
organization to seek a resolution of such conflict b. To look from time to time into the conditions affecting the
b. Review the conflict problem with the immediate superior if practice of the accountancy profession.
the organization's policies do not resolve the ethical c. To create and direct accrediting agencies that are
conflict. entrusted the functions of reviewing higher
c. If the problem is not resolved with the immediate educational institutions' policies and practices
superior and the professional accountant determines leading to accreditation/reaccreditation of BSA
to go to the next higher managerial level, the program.
immediate superior need not be notified of the d. To determine and prescribe minimum requirements
decision. leading to the admission of candidates to the CPA
d. Seek counseling and advice on a confidential basis with licensure examination.
an independent advisor or the applicable professional
accountancy body or regulatory body to obtain an 49. All of the following are represented to the Financial Reporting
understanding of possible courses of action. Standards Council, except:
a. Securities and Exchange Commission
45. As a resolution of the conflict in the application of fundamental b. Bureau of Internal Revenue
principles, the auditor, after considering the ethical issues and c. Commission on Higher Education
relevant facts may do any of the following, except: d. Board of Accountancy
a. Must immediately resign from the engagement or the
employing entity.
b. Should weigh me consequences of each possible course 50. A CPA whose certificate of registration has been revoked:
of action. a. Can no longer be reinstated.
c. Should consult with other appropriate persons within the b. Is automatically reinstated as a CPA by the PRC after two
firm or employing organization for help to finally resolve years if he has acted in an exemplary manner.
the matter. c. May be reinstated by the Professional Regulation
d. The professional accountant may wish to obtain Commission after two years if he has acted in an
professional advice from, the relevant professional body exemplary manner.
without breaching confidentiality if significant conflict
cannot be resolved.

AT – Prelim Rev (875 MCQs) Red Sirug Page 57 of 77


d. May be reinstated as a CPA by the Board of a. Identifies areas and specialized situations where
Accountancy after two years if he has acted in an consultation is required and encourages personnel to
exemplary manner. consult with or in use authoritative sources on other
complex matters.
51. The Philippine Accountancy Act of 2004 provides that all b. Designates individuals as specialists to serve as
Working papers made during an audit shall be the property of authoritative sources and define their authority in
the auditor. These working papers shall include the following, consultative situations.
except: c. Assigns an appropriate person or persons to be
a. Working papers prepared by the CPA and his staff. responsible for assigning personnel to audits.
b. Analysis and schedule prepared and submitted to the d. Specifies the extent of documentation to be provided for
auditor by his client's staff. the result of consultation in those areas and specialized
c. Excerpts or copies of documents furnished to the auditor. situations where consultation is required.
d. Any report submitted by the auditor to his client.
57. According to Philippine Standards on Auditing, because there
52. Who are required to apply for accreditation with the are inherent limitations in an audit that affect the auditor's
Professional Regulation Commission if the applicant is a ability to detect material misstatements, the auditor is:
partnership of Professional Accountants? a. Neither a guarantor nor an insurer of financial
a. Managing partner only statements.
b. All partners only b. A guarantor but not an insurer of the statements.
c. Partners and staff members c. An insurer but not a guarantor of the statements.
d. Partners, principals, and staff members d. Both a guarantor and an insurer of the financial
statements.
53. Individual CPAs, Firms or Partnerships of CPAs, including
partners and staff members thereof shall register with the BOA 58. The working papers prepared by a CPA in connection with an
and the PRC. If the accreditation of Velasco and Co., CPAs, audit engagement are owned by the CPA, subject to certain
was renewed on September 30, 2008, the next renewal must limitations. The rationale for this rule is to
be on or before: a. Protect the working papers from being subpoenaed.
a. September 30, 2010 b. Provide the basis for excluding admission of the working
b. September 30, 2011 papers as evidence because of the privileged
c. December 31, 2010 communication rule.
d. December 31, 2011 c. Provide the CPA with evidence and documentation
which may be helpful in the event of a lawsuit.
54. The APO shall renew its Certificate of Accreditation once d. Establish a continuity of relationship with the client
every how many years after the date of the Resolution whereby indiscriminate replacement of CPAs is
granting the petition for re-accreditation and the issuance of discouraged.
the said certificate upon submission of the requirements?
a. 2 years 59. The responsibility for adopting sound accounting policies,
b. 3 years maintaining adequate internal control, and making fair
c. 4 years representations in the financial statements rests
d. 6 years a. With the management.
b. With the independent auditor.
55. Engagement letters are widely used in practice for c. Equally with management and the auditor.
professional engagements of all types. The primary purpose d. With the internal audit department.
of the engagement letter is to
a. Remind management of its primary responsibility over the 60. The ordinary examination of financial statements is not
financial statements. primarily designed to disclose defalcations and other
b. Satisfy the requirements of the Code of Professional irregularities although their discovery may result. Normal audit
Conduct for CPAs. procedures are more likely to detect a fraud arising from
c. Provide a starting point for the auditor's preparation of the a. Collusion on the part of several employees.
preliminary audit program. b. Failure to record cash receipts for services rendered.
d. Provide a written record of the agreement with the c. Forgeries on company checks.
client as to the services to be provided. d. Theft of inventories.

56. Which of the following is not likely a quality control procedure 61. A principal purpose of a letter of representation from
on consultation? management is to

AT – Prelim Rev (875 MCQs) Red Sirug Page 58 of 77


a. Serve as an introduction to company personnel and an
authorization to examine the records. 67. When management refuses to disclose in the financial
b. Discharge the auditor from legal liability for his statements noncompliance to laws and regulations which are
examination. identified by the independent auditor, the CPA may be
c. Confirm in writing management's approval of limitations charged with unethical conduct for
on the scope of the audit. a. Withdrawing from the engagement.
d. Remind management of its primary responsibility for b. Issuing a disclaimer of opinion.
financial statements. c. Failure to uncover the noncompliance to laws and
regulations during the prior audits.
62. The auditor should not assume that management is dishonest, d. Reporting these activities to the audit committee.
but the possibility of dishonesty must be considered." This is
an example of 68. A procedure in which a quality control partner periodically
a. Expectation gap. tests the application of quality control procedures is most
b. An attitude of professional skepticism. directly related to which quality control element?
c. Due diligence. a. Engagement performance
d. An ethical requirement. b. Independence, integrity, and objectivity
c. Monitoring
63. Which of the following statements is true? d. Personnel management
a. It is usually easier for the auditor to uncover irregularities
than errors. 69. The work of each assistant needs to be reviewed by personnel
b. It is usually easier for the auditor to uncover errors of at least equal competence. Which of the following is not one
than irregularities. of the objectives of this requirement?
c. It is usually equally difficult for the auditor to uncover a. The conclusions expressed are consistent with the result
errors or irregularities. of the work performed and support the opinion.
d. Usually, none of the given statements is true. b. The work performed and the results obtained have been
adequately documented.
64. Generally, the decision to notify parties outside the client's c. The audit objectives have been achieved.
organization regarding a noncompliance with laws and d. All available evidences have been obtained,
regulations evaluated and documented.
a. Independent auditor.
b. Management. 70. Which of the following statements is true when the CPA has
c. Outside legal counsel. been engaged to do an attestation engagement?
d. Internal auditors. a. The CPA firm is engaged and paid by the client; therefore,
the firm has primary responsibility to be an advocate for
65. An audit made in accordance with Philippine Standards on the client.
Auditing generally should b. The CPA firm is engaged and paid by the client, but
a. Be expected to provide assurance that noncompliance the primary beneficiaries of the audit are the
with laws and regulations will be detected if the internal statement users.
control is effective. c. Should a situation arise where there is no convincing
b. Be relied upon to disclose indirect-effect noncompliance authoritative standard available, and there is a choice of
with laws and regulations. actions which could impact client's financial statements
c. Encompass a plan to search actively for noncompliance either positively or negatively, the CPA is free to endorse
with laws and regulations which relate to operating the choice which is best in the client's interest.
aspects. d. As long as CPA firms are competent, it is not required
d. Not be relied upon to provide assurance that all that they remain unbiased.
noncompliance with laws and regulations will be
detected. 71. When CPAs are able to maintain an independent attitude in
fulfilling their responsibility, it is referred to as independence in
66. An auditor who believes that a material irregularity may exist a. Fact.
should initially b. Appearance.
a. Discuss the matter with those believed to be involved in c. Conduct.
the perpetration of material irregularity. d. Total.
b. Discuss the matter with a higher level of management.
c. Withdraw from the engagement.
d. Consult legal counsel.

AT – Prelim Rev (875 MCQs) Red Sirug Page 59 of 77


72. When the users of financial statements have confidence in the
independence of the CPA, it is referred to as in independence 78. When the auditor issues an erroneous opinion as a
in consequence of an underlying failure to comply with the
a. Fact. requirements of generally accepted auditing standards, it
b. Appearance. results to
c. Conduct. a. Business failure.
d. Total. b. Audit failure.
c. Audit risk.
73. When determining whether independence is impaired because d. All of them
of an ownership interest in client company, materiality will
affect whether ownership is a violation of rule of independence 79. The responsibility for the fairness of the financial assertions
a. In all circumstances. that are embodied in the financial statements and in the notes
b. Only for direct ownership. to the financial statements rests:
c. Only for indirect ownership. a. With the audit committee.
d. Under no circumstances. b. With management.
c. With Securities and Exchange Commission.
74. A professional accountant has a professional duty or right d. Equally with management and the stockholders.
disclose confidential information in each of the following,
except: 80. When preparing the financial statements, it is acceptable for
a. To comply with technical standards and ethics the auditor to prepare
requirements. a. The footnotes for client.
b. To disclose to the Bureau of Internal Revenue any b. The statement for client.
fraudulent scheme committed by the client on c. A draft of the statements and footnotes for client.
payment of income tax. d. A draft of the statements for client.
c. To comply with the quality review of a member body or
professional body 81. Which of the following statements best describes the auditor's
d. To respond to an inquiry or investigation by a member responsibility regarding the detection of material errors and
body or regulatory body. frauds?
a. The auditor is responsible for the failure to detect material
75. Which of the following best describes the passing of errors and frauds only when such failure results from the
confidential information from a client to its auditor? The misapplication of generally accepted accounting
information: principles.
a. Should in no circumstances be conveyed to third parties. b. The auditor is responsible for the failure to detect material
b. Is not legally protected and can be subpoenaed by a errors and frauds only when the auditor fails to confirm
competent court. receivables or observe inventories.
c. Can only be released for peer reviews after receiving c. The audit should be designed to provide reasonable
permission from the client. assurance that material errors and frauds are
d. Should be conveyed to the public if it affects the detected.
"correctness" of the financial statements. d. Extended auditing procedures are required to detect
unrecorded transactions even if there is no evidence that
76. A member in public practice may perform for a contingent fee material errors and frauds may exist.
any professional services for a client for whom the member or
member's firm performs 82. The auditor has considerable responsibility for notifying users
a. An audit. as to whether or not the statements are properly stated. This
b. A review. imposes upon the auditor a duty to
c. A compilation used only by management. a. Provide reasonable assurance that material
d. An audit of prospective financial information. misstatements will be detected.
b. Be an insurer of the fairness in the statements.
77. Which one of the following contingent fee is allowed? c. Be a guarantor of the fairness in the statements.
a. All services performed by a CPA film. d. Be equally responsible with management for the
b. Non-attestation services. preparation of the financial statements.
c. Non-attestation services, unless the CPA firm was
also performing attestation services for the same 83. The factor that distinguishes an error from an irregularity is
client. a. Whether it is peso amount or a process.
d. Attestation services. b. Whether it is a caused by the auditor or the client.

AT – Prelim Rev (875 MCQs) Red Sirug Page 60 of 77


c. Materiality. 90. An audit plan is a
d. Intent. a. Detailed plan of analytical procedures and all substantive
tests to be performed in the course of the audit.
84. The reason why an auditor accumulates evidence is to b. Document that provides an overview of the company
a. Defend himself in the event of a lawsuit. and a general plan for the audit work to be
b. Justify the conclusions he has otherwise reached. accomplished, timing of the work, and other matters
c. Satisfy the requirements of the bureau of internal revenue. of concern to the audit.
d. Enable him to reach conclusions about the fairness c. Generic document that auditing firms have developed to
of the financial statements and issue an appropriate lead the process of the audit through a systematic and
audit report. logical process.
d. Budget of the time that should be necessary to complete
85. The auditor gives an audit opinion on the fair presentation of each phase of the audit procedures.
the financial statements and associates his or her name with
them when, on the basis of adequate evidence, the auditor 91. There is generally an agreement within the auditing profession
concludes that the financial statements are unlikely to mislead and the courts that the auditor
a. A prudent user. a. Is both a guarantor and an insurer of the financial
b. Management. statements.
c. The reader. b. Is a guarantor but not an insurer of the statements.
d. Investors. c. Is an insurer but not a guarantor of the statements.
d. Is neither a guarantor nor an insurer of financial
86. The probability that an auditor's procedures leading to the statements.
conclusion that a material error does not exist in an account
balance when, in fact, such error does exist is referred to as 92. The auditor is not liable to his client for
a. Prevention risk. a. Negligence.
b. Detection risk. b. Bad faith.
c. Inherent risk. c. Errors of judgment
d. Control risk. d. Dishonesty.

87. Which of the following is not included in an audit engagement 93. Which of the following services provides the highest level of
letter? assurance to third parties about a company's financial
a. Objectives of the engagement statements?
b. Representations that the financial statements were a. Audit.
prepared in accordance with PFRS b. Review.
c. Management's responsibilities c. Compilation.
d. A clear explanation of the services to be performed on d. Each of the above provides the same level of assurance.
the engagement
94. Most accounting and auditing professionals agree that when
88. Which of the following is least likely included in an auditor’s an audit has failed to uncover material misstatements, and
inquiry of management while obtaining information to identify the wrong type of audit opinion is issued, the audit firm
the risks of material misstatement due to fraud? a. Has failed to follow generally accepted auditing standards
a. Are financial reporting operations controlled by and (GAAS).
limited to one location? b. Should be asked to defend the quality of the audit.
b. Does it have knowledge of fraud or suspect fraud? c. Deserves to lose the lawsuit.
c. Does it have programs to mitigate fraud risks? d. Should not be held responsible for the financial loss
d. Has it reported to the audit committee the nature of the suffered loss suffered by others.
company's internal control?
95. The objective of quality control mandates that a public
89. Which of the following should the auditors normally interview accounting firm should establish policies and procedures for
as part of their assessment of fraud risk? professional development which provide reasonable
a. Senior management assurance that all entry-level personnel
b. Audit committee a. Prepare working papers which are standardized in form
c. Various employees whose duties financial reporting and content.
responsibilities b. Will advance within the organization.
d. All of the given choices c. Develop specialties in specific areas of public accounting.
d. Have the knowledge required to enable them to fulfill

AT – Prelim Rev (875 MCQs) Red Sirug Page 61 of 77


responsibilities assigned. requirement prevails.
c. The Code of Ethics for Professional Accountants in the
96. In pursuing its quality control objectives with respect to Philippines is mandatory for all CPAs and is applicable to
assigning personnel to engagements, a public accounting firm professional services performed in the Philippines on or
may use policies and procedures such as after January 1, 2004.
a. Rotating employees from assignment to assignment on d. Professional accountants should consider the ethical
a random basis to aid in the staff training effort. requirements as the basic principles which they should
b. Allowing staff to select the assignments of their choice to follow in performing their work.
promote better client relationships.
c. Assigning a number of employees to each engagement
in excess of the number required so as not to SET 8
overburden the staff, and interfere with the quality of the
audit work performed. 1. What will an auditor who has been proposed for an audit
d. Requiring timely identification of the staffing engagement usually do prior to accepting a new client?
requirements of specific engagements so that a. Draft the financial statements of the client as a measure
enough qualified personnel can be made available. of goodwill.
b. With the permission from the prospective client,
97. The provision of services by a firm or network firm to an audit contact the predecessor auditor to determine if there
client that involve the design and implementation of financial are any disagreements between the client and the
information technology systems that are used to generate audit firms.
information forming part of a client's financial statements may c. Obtain the potential client's permission to talk to the
most likely create former auditor and review work papers.
a. Self-interest threat. d. Perform a peer review on the potential client in
b. Self-review threat. accordance with professional standards.
c. Intimidation threat.
d. Familiarity threat, 2. Which of the following statements is true with regard to the
relationship among audit risk, audit evidence, and materiality?
98. When determining whether independence is impaired because a. The lower the inherent risk and control risk, the lower the
of an ownership interest in client company,' materiality will aggregate materiality threshold.
affect whether ownership is a violation of rule of independence b. Under conditions of high inherent and control risk,
a. In all circumstances. the auditor should place more emphasis on obtaining
b. Only for direct ownership. external evidence and should reduce reliance on
c. Only for indirect ownership. internal evidence.
d. Under no circumstances. c. Where inherent risk is high and control risk is low, the
auditor may safely ignore inherent risk.
99. A successor auditor is required to communicate with the d. Aggregate materiality thresholds should not change
previous auditor. The primary concern in this communication under conditions of changing risk levels.
is
a. Information which will help the successor auditor in 3. Which of the following is most likely to be an overall response
determining whether the client management has to fraud risks identified in an audit?
integrity. a. Supervise members of the audit team less closely and
b. To learn about client by examining predecessor's working rely more upon judgment.
papers. b. Use less predictable audit procedures.
c. To enable successor auditor to perform a more efficient c. Use only certified public accountants on the engagement.
audit. d. Place increased emphasis on the audit of objective
d. To save successor auditor time and money in gathering transactions rather than subjective transactions.
data.
4. Which of the following represents a procedure that the auditor
100. Which statement is incorrect regarding the Code of Ethics for may use because plausible relationships among financial
Professional Accountants in the Philippines? statement balances are expected to exist?
a. Professional accountants refer to persons who are a. Attributes testing
Certified Public Accountants (CPA) and who hold a valid b. Enterprise risk assessment
certificate issued by the Board of Accountancy. c. Inherent tests of control
b. Where a national statutory requirement is in conflict d. Analytical review
with a provision of the IFAC Code, the IFAC Code

AT – Prelim Rev (875 MCQs) Red Sirug Page 62 of 77


5. Which of the following statements is incorrect regarding superior and the professional accountant determines
obtaining an understanding of the entity and its environment? to go to the next higher managerial level, the
a. Obtaining an understanding of the entity and its immediate superior need not be notified of the
environment is an essential aspect of performing an audit decision.
in accordance with PSAs. d. Seek counseling and advice on a confidential basis with
b. Understanding of the entity and its environment an independent advisor or the applicable professional
establishes frame of reference within which the auditor accountancy body or regulatory body to obtain an
plans the audit and exercises professional judgment understanding of possible courses of action.
about assessing risks of material misstatement in the
financial statements and responding to those risks 9. While assessing the risk of material misstatement, the auditors
throughout the audit. identity risks, relate risk to what could go wrong, consider the
c. The auditor's primary consideration is whether the magnitude of risks and:
understanding that has been obtained is sufficient to a. Assess the risk of misstatements due to noncompliance
assess the risks of material misstatement in the financial to laws and regulations.
statements and to design and perform further audit b. Consider the complexity of the transactions involved.
procedures. c. Consider the likelihood that the risks could result in
d. The depth of the overall understanding that is material misstatements.
required by the auditor in performing the audit is at d. Determine materiality level.
least equal to that possessed by management in
managing the entity. 10. Which of the following is least likely considered a financial
statement audit risk factor?
6. Inquiries directed towards those charged with governance a. Management operating and financing decisions are
may most likely dominated by top management.
a. Relate to their activities concerning the design, and b. A new client with no prior audit history.
effectiveness of the entity's internal control and whether c. Rate of change in the entity's industry is rapid.
management has satisfactorily responded to any findings d. Profitability of the entity relative to its industry is
from these activities. inconsistent.
b. Help the auditor understand the environment in
which the financial statements are prepared. 11. Which of the following is most likely to be considered a risk
c. Relate to changes in the entity's marketing strategies, factor relating to fraudulent financial reporting?
sales trends, or contractual arrangements with its a. Low turnover of senior management.
customers. b. Extreme degree of competition within the industry.
d. Help the auditor in evaluating the appropriateness of the c. Capital structure including various operating subsidiaries.
selection and application of certain accounting policies. d. Sales goals in excess of any of the preceding three years.

7. The underlying reason for a code of professional conduct for 12. Which of the following is correct concerning requirements
any profession is about auditor's communications about fraud?
a. That it is required by congress. a. Fraud that involves senior management should be
b. The need for public confidence in the quality of reported directly to the audit committee regardless of
service of the profession. the amounts involved.
c. That it provides a safeguard to keep unscrupulous people b. All fraud with a material effect on the financial statements
out. should be reported directly by the auditor to the SEC
d. That it allows Professional Regulation Commission to c. Fraud with a material effect on the financial statements
have a yardstick to measure deficient performance. should ordinarily be disclosed by the auditor through the
use of an emphasis of a matter paragraph added to the
8. Professional accountants may encounter problems in audit report.
identifying unethical behavior or in resolving an ethical conflict. d. The auditor has no responsibility to disclose fraud outside
When faced with significant ethical issues, professional the entity under any circumstances.
accountants should do the following, except
a. Follow the established policies of the employing 13. Which of the following factors most likely would heighten an
organization to seek a resolution of such conflict. auditor's concern about the risk of fraudulent financial
b. Review the conflict problem with the immediate superior if reporting?
the organization's policies do not resolve the ethical a. Large amounts of liquid assets that are easily convertible
conflict. into cash.
c. If the problem is not resolved with the immediate

AT – Prelim Rev (875 MCQs) Red Sirug Page 63 of 77


b. Low growth and profitability as compared to other entity's 19. It involves a study or evaluation of the quality of audit of
in the same industry. financial statements through a review of quality control
c. Financial management's participation in the initial measures established by on CPA firms and individual CPAs in
selection of accounting principles. public practice to ensure compliance with accounting and
d. An overly complex organizational structure involving auditing standards and practices
unusual lines of authority. a. External audit
b. Compliance audit
14. Which of the following is most likely to be an overall response c. Peer review
to fraud risks identified in an audit? d. Quality review
a. Only use certified public accountants on the engagement.
b. Place increased emphasis on the audit of objective 20. Which of the following risks is entirely a quality criterion based
transactions rather than subjective transactions. on professional judgment?
c. Supervise members of the audit team less closely and a. Inherent risk
rely more upon judgment. b. Control risk
d. Use less predictable audit procedures. c. Detection risk
d. Audit risk
15. A principal purpose of a letter of representation from
management is to 21. Inherent risk is not a characteristic of the
a. Remind management of its primary responsibility for a. Major types of transactions.
financial statements. b. Client’s business.
b. Serve as an introduction to company personnel and an c. Substantive procedures.
authorization to examine the records. d. Effectiveness of the client’s accountants.
c. Discharge the auditor from legal liability for his
examination. 22. Misstatements must be compared to some measurement base
d. Confirm in writing management's approval of limitations before a decision can be made about the materiality of the
on the scope of the audit. failure to follow GAAP. A commonly accepted measurement
base would be
16. Should the auditor uncover circumstances during the audit a. Net income.
that may cause suspicions of management fraud, the auditor b. Total assets.
must c. Working capital.
a. Withdraw from engagement. d. All of the above.
b. Issue an adverse opinion.
c. Issue a disclaimer. 23. The primary deliverable of an engagement to perform based
d. Evaluate their implications and consider the need to on procedures prescribed by the intended user of the report
modify audit evidence. is/are:
a. The Review Report
17. In connection with the element of professional development, a b. Report of Factual Findings
CPA firm's system of quality control should ordinarily provide c. Management Letter
that all personnel d. The financial statements
a. Possess judgment, motivation, and adequate experience.
b. Seek assistance from persons having appropriate level 24. Fraudulent financial reporting is often called:
of knowledge, judgment, and authority. a. Theft of assets
c. Demonstrate compliance with peer review directives. b. Employee fraud
d. Have the knowledge required to enable them to fulfill c. Management fraud
responsibilities assigned. d. Defalcation

18. The implementation of quality control procedures that are 25. The ordinary examination of financial statements is not
applicable to the individual audit engagement is the primarily designed to disclose defalcations and other
responsibility of the irregularities although their discovery may result. Normal audit
a. CPA firm. procedures are more likely to detect a fraud arising from
b. Engagement quality control reviewer. a. Theft of inventories.
c. Expert contracted by the firm in connection with the audit b. Collusion on the part of several employees.
engagement. c. Failure to record cash receipts for services rendered.
d. Engagement team. d. Forgeries on company checks.

AT – Prelim Rev (875 MCQs) Red Sirug Page 64 of 77


26. Working papers prepared by a CPA in connection with an b. Discuss the matter with those believed to be involved in
audit engagement are owned by the CPA, subject to certain the perpetration of the material irregularity.
limitations. The rationale for this rule is to c. Discuss the matter with a higher level of management.
a. Protect the working papers from, being subpoenaed. d. Withdraw from the engagement.
b. Provide the CPA with evidence and documentation
which may be helpful in the event of a lawsuit. 32. Which of the following statements is correct concerning the
c. Provide the basis for excluding admission of the working auditor's responsibility with respect to illegal acts?
papers as evidence because of the privileged a. An auditor must design tests to detect both direct-effect
communication rule. and indirect-effect illegal acts.
d. Establish a continuity of relationship with the client b. An auditor must design tests to detect both immaterial
whereby indiscriminate replacement of CPAs is and material direct-effect illegal acts.
discouraged. c. An auditor must design tests to obtain reasonable
assurance of detecting material direct-effect illegal
27. The auditor's responsibility for failure to detect fraud arises acts.
a. Whenever the amounts involved are material. d. An auditor must design tests to detect both material
b. When such failure clearly results from direct-effect and material indirect-effect illegal acts.
noncompliance to generally accepted auditing
standards. 33. If specific information comes to an auditor's attention that
c. Only when such failure clearly results from negligence so implies the existence of possible illegal acts that could have a
gross as to sustain an inference of fraud on the part of material, but indirect effect on-the financial statements, the
the auditor. auditor should next
d. Only when the examination was specifically designed to a. Report the matter to an appropriate level of management
detect fraud. at least one level above those involved.
b. Apply audit procedures specifically directed to
28. The factor that distinguishes constructive fraud from actual ascertaining whether an illegal act has occurred.
fraud is c. Seek the advice of an informed expert qualified to
a. Materiality. practice law as to possible contingent liabilities.
b. Quality of internal control. d. Discuss the evidence with the client's audit committee, or
c. Type of error or irregularity. others with equivalent authority.
d. Intent.
34. When planning the audit, if the auditor has no reason to
29. If a CPA recklessly abandons standards of due care and believe that illegal acts exist, the auditor should
diligence while performing an audit, he or she may be held a. Include audit procedures which have a strong probability
liable to unknown third parties for: of detecting illegal acts.
a. Gross negligence. b. Make inquiries of management regarding their
b. Fraudulent misconduct. policies and regarding their knowledge of violations,
c. Gross misconduct. and then rely on normal audit procedures to detect
d. Contributory negligence. errors, irregularities, and illegalities.
c. Still include some audit procedures designed specifically
30. Of the following statements, which best distinguishes ordinary to uncover illegalities.
negligence from gross negligence? d. Ignore the topic.
a. The more material the undetected error the greater the
likelihood of ordinary negligence. 35. When the auditor knows that an illegal act has occurred, the
b. Gross negligence is most probable when the auditor fails auditor must
to detect errors that occurred under conditions of strong a. Issue an adverse opinion.
internal control. b. Withdraw from the engagement.
c. Failure to detect material errors, whether internal control c. Report it to the proper government authorities.
is strong or weak, suggests gross negligence. d. Consider the effects on the financial statements,
d. Failure to exercise reasonable care denotes ordinary including the adequacy of disclosure.
negligence, whereas failure to exercise minimal care
indicates gross negligence. 36. Assurance services differ from consulting services in that they
I. Focus on providing advice.
31. An auditor who believes that a material irregularity may exist
II. Involve monitoring of one party by another.
should initially
a. Consult legal counsel. a. I only.

AT – Prelim Rev (875 MCQs) Red Sirug Page 65 of 77


b. II only. fairness of the client's assertions.
c. Both I and II. d. A review provides positive assurance by attesting the
d. Neither I nor II. reliability of the client's assertions.

37. For assurance engagements which are neither audits nor 44. An attestation engagement is one in which a CPA is engaged
reviews of historical financial information, the following to:
standard applies: a. Assemble prospective financial statements based on the
a. PSAs assumptions of the entity's management without
b. PSAEs expressing any Testify as an expert witness in accounting,
c. PSREs auditing or tax matters, given certain stipulated facts.
d. PSRSs b. assurance.
c. Issue a written communication expressing a
38. Unlike consulting services, in assurance services: conclusion about the reliability of a written assertion
a. There is confirmation that financial statement assertions that is the responsibility of another party.
are accurate. d. Provide tax advice or prepare a tax return based on
b. Misstated account balances are generally corrected by an financial information the CPA has not audited or reviewed.
independent audit.
c. Ineffective internal controls may exist. 45. Assurance engagements encompass the following types of
d. Competing interests may exist between management services, except:
and the users of statements. a. Audit of historical financial statements.
b. Review engagements.
39. The attest function: c. Management consulting.
a. Is an essential part of every engagement performed by a d. Attestation services.
CPA.
b. Includes the preparation of a written report of the 46. Criteria that are embodied in laws or regulations, or issued by
CPA's conclusion. authorized or recognized bodies of experts that follow a
c. Requires a complete review of all transactions during the transparent due process are called:
period under examination. a. Suitable criteria
d. Requires a review of a sample of transactions during the b. General criteria
period under examination. c. Established criteria
d. Specifically developed criteria
40. Which of the following is an assurance service?
a. Performance measurement 47. Which of the following is not a characteristic of suitable criteria?
b. Systems design and installation a. Completeness
c. Tax planning b. Neutrality
d. Personal financial planning c. Relevance
d. Comparability
41. Non-assurance engagements include all of the following
services, except: 48. The criteria to be used for a particular engagement can either
a. Agreed-upon procedures engagements. be established or specifically developed. Established criteria
b. Compilations of financial or other information. are those
c. Management and tax consulting. a. Designed for the purpose of the engagement.
d. Examination of prospective financial information. b. Embodied in laws or regulations or issued by autho-
rized or recognized bodies of experts that follow a
42. Unlike consulting services, assurance services: transparent due process.
a. Make recommendation to management. c. Made available to the intended users.
b. Report on how to use information. d. Not made available to the intended users.
c. Are two-party contracts.
d. Report on the quality of information. 49. A practitioner should accept an assurance engagement only if
a. The practitioner's conclusion is to be contained in a
43. Identify the correct statement. written report.
a. Accounting services do not provide attestation. b. The subject matter is the responsibility of either the
b. Management consulting services provide attestation in all intended users or the practitioner.
cases. c. The subject matter is in the form of financial information.
c. An audit provides limited assurance by attesting to the

AT – Prelim Rev (875 MCQs) Red Sirug Page 66 of 77


d. The criteria to be used are not available to the intended procedures engagement?
users. a. No assurance.
b. Low.
50. Assurance engagement risk is the risk c. Moderate.
a. Of expressing an inappropriate conclusion when the d. Reasonable.
subject matter information is not materially misstated.
b. That the practitioner expresses an inappropriate 57. The term "accountant" has been used by Auditing and
conclusion when the subject matter information is Assurance Standards Council to refer to a CPA in public
materially misstated. practice who is engaged to:
c. Of expressing an inappropriate conclusion when the a. Audit financial statements.
subject matter information is either materially misstated or b. Review financial statements.
not materially misstated. c. Apply agreed-upon procedures.
d. Through loss from litigation, adverse publicity, or other d. Compile financial statements.
events arising in connection with a subject matter
reported on. 58. Which of the following best describes relationships among
auditing, attest and assurance services?
51. The following are components of assurance engagement risk, a. Attest is a type of auditing service.
except b. Assurance is a type of attest service.
a. Detection risk c. Auditing is a type of assurance service.
b. Business risk d. Auditing and attest services represent two distinctly
c. Inherent risk different types of services.
d. Control risk
59. Which of the following is the single feature that most clearly
52. The predominant type of attestation service performed by distinguishes auditing, attestation, and assurance:
CPAs is: a. CPA's approach to the service.
a. Audit. b. Type of service.
b. Review. c. Scope of services.
c. Compilation. d. Training necessary to perform the service.
d. Management consulting.
60. For assurance engagements regarding historical financial
53. Which of the following is incorrect regarding a compilation information, reasonable assurance engagements are called:
engagement? a. Review engagements.
a. The CPA uses his auditing expertise to collect, b. Audit engagements.
classify and summarize financial information. c. Compilation engagements.
b. The CPA should exercise due care. d. Agreed-upon procedures engagements.
c. The engagement ordinarily entails reducing detailed data
to a manageable and understandable form. 61. In some assurance engagements, the evaluation or
d. The procedures performed do not enable the accountant measurement of the subject matter is performed by the
to express any form of assurance. responsible party, and the subject matter information is in the
form of an assertion by the responsible party that is made
54. The type of assurance that is provided by the CPA on a available to intended users. These engagements are called
compilation report is: a. Recurring engagements
a. Limited assurance. b. Non-assurance engagements
b. No assurance. c. Direct reporting engagements
c. Low assurance. d. Assertion-based engagements
d. Negative assurance.
62. What type of assurance engagement is involved when the
55. A report on factual findings is the end product of the auditor practitioner expresses a positive form of conclusion?
when performing: a. Positive assurance engagement
a. Audit. b. Limited assurance engagement
b. Review. c. Reasonable assurance engagement
c. Agreed-upon procedures. d. Absolute assurance engagement
d. Compilation.
63. What type of assurance engagement is involved when the
56. What assurance is provided by the auditor in an agreed-upon practitioner expresses a negative form of conclusion?

AT – Prelim Rev (875 MCQs) Red Sirug Page 67 of 77


a. Negative assurance engagement d. Guaranteed to be free from error.
b. Assertion-based assurance engagement
c. Limited assurance engagement 69. The principal issue to be resolved in cases involving alleged
d. Reasonable assurance engagement negligence is usually
a. The amount of the damages suffered by the users of the
64. Assurance engagements involve: financial statements.
a. Two separate parties: a professional accountant and b. Whether to impose punitive damages on defendant.
intended users. c. The level of care required to be exercised.
b. Two separate parties: a professional accountant and a d. Whether defendant was involved in fraud.
responsible party.
c. Three separate parties: a practitioner, a 70. Privity of contract exists between the
responsible party and intended users. a. Auditor and the Securities and Exchange Commission.
d. Three separate parties: a professional accountant, a b. Auditor and client.
responsible party, and intended users. c. Auditor and third parties.
d. All of the above
65. The auditor's evaluation of the likelihood of material employee
fraud is normally done initially as a part of 71. A CPA is criminally liable if he
a. The assessment of whether to accept the audit a. Refuses to turn over the schedules or working papers
engagement. prepared by the client staff to the client.
b. Understanding the entity's internal control structure. b. Performs an audit in a negligent manner.
c. The Tests of Controls. c. Intentionally allows an omission of a material fact
d. The Tests of Transactions. required to be stated in a financial statement.
d. Was not able to submit the audited financial statements
66. In which circumstance is a CPA firm's independence most on time.
likely to be impaired?
a. An individual on the audit has a close relative who is a 72. The auditor's defense of contributory negligence is most likely
receptionist for the client. to prevail when
b. The father of the audit senior holds a material financial a. Third party injury has been minimal.
interest in the client of which the senior is unaware. b. The client is privately held as contrasted with a public
c. The spouse of a staff member on the audit has an company.
immaterial common stock investment in the audit c. Undetected errors have resulted in materially misleading
client. financial statements.
d. The partner in charge of the office's compensation is d. The auditor fails to detect fraud resulting from
affected by office profitability, a portion of which arises management override of the control structure.
from this audit.
73. An expectation of the public is that the auditor will recognize
67. Which of the following partners is least likely to be considered that the primary users of audit services are:
a "covered member" for purposes of rendering assurance a. The employees
service to of Company A, a nonaudit client, performed by the b. The investors and creditors
head office of a national CPA firm? c. The SEC
a. The partner in charge of the Davao office. d. The board of directors
b. The partner in charge of the entire CPA firm.
c. A partner in the Cebu office of the CPA firm who 74. The overriding objective of the International Auditing
maintains a small, immaterial investment in Company Standards that are issued by the International Auditing
A. Practices Committee of the IFAC is
d. A partner in the Davao office who worked on the a. To override a country’s regulations governing the audit
Company A for a different assurance engagement in of financial statements.
previous years, but currently has no responsibilities with b. To improve the uniformity of auditing practices and
respect to the engagement related services throughout the world.
c. To provide a uniform application of specific audit
68. While performing services for their clients, professionals have procedures that are acceptable worldwide.
always had a duty to provide a level of care which is d. To replace generally accepted auditing standards.
a. Reasonable.
b. Greater than average. 75. Management’s assertions in the financial statements are
c. Superior. relevant to the audit process because:

AT – Prelim Rev (875 MCQs) Red Sirug Page 68 of 77


a. They provide evidence that auditors have prepared a. For assurance engagements provided to an audit
financial statements in accordance with GAAP client.
b. They embody the audit procedures that will be b. For assurance engagements provided to clients that are
performed by the audit engagement team. not audit clients, when the report is not expressly
c. They include representations of financial statements restricted for use by identified users.
in accordance with the applicable reporting criteria c. For assurance engagements provided to clients that are
d. They relate to regulator’s expectations about audit not audit clients, when the assurance report is expressly
results. restricted for use by identified users.
d. All of the above
76. A CPA firm is considered independent when it performs which
of the following services for a publicly traded audit client? 82. One difference between auditors and other professionals is
a. Serving as a member of the client’s board of directors. that most professionals
b. Accounting information system design and a. Need not be concerned about remaining independent.
implementation. b. Do not have requirements for continuing education
c. Tax return preparation as approved by the board of beyond college.
directors. c. Do not have to pass a rigorous examination.
d. Determining which accounting policies will be adopted d. Are not expected to act in public interest.
by the client.
83. The Code of Professional Conduct would be violated if a
77. Jessie Cruz, CPA, forgot to test a client’s assessment of member accepted a fee for services and the fee was
goodwill impairment during an audit. Such an act is probably a. Fixed by a public authority.
an example of: b. Based on a price quotation submitted in competitive
a. Due diligence bidding.
b. Reckless professional behavior c. Based on the result of judicial proceedings.
c. Ordinary negligence d. Payable after a specified finding was attained.
d. Fraud
84. For which of the following services is a CPA professional
78. Similar to auditors in the CPA realm, internal auditors also required to be independent?
strive to possess: a. Audits of historical financial statements
a. Independence b. Review services
b. Competence c. Examination of prospective financial statements
c. Objectivity d. All of the above
d. All of the above
85. For which of the following services is a CPA professional not
79. A review engagement differs in scope as compared to an audit required to be independent?
due to a. Tax returns preparation
a. The subject matter of the audit. b. Audits of historical financial statements
b. The quantity and type of evidence obtained. c. Review engagement
c. Ethical requirements with respect to independence. d. Examination of a forecast
d. The users of the financial statements.
86. It occurs when a firm or a member of the assurance team
80. Which of the following statements concerning the intended could benefit from a financial interest in, or other self-interest
user of a professional accountant's report is incorrect? conflict with, an assurance client.
a. The responsible party and the intended user may both be a. Self-interest threat
within the same organization. b. Self-review threat
b. The intended user should never be established by c. Advocacy threat
agreement between the practitioner and the d. Familiarity threat
responsible party or those engaging or employing
the practitioner. 87. The CPA title was conceived and created, under the first
c. In some circumstances, the intended user may be accountancy law, for professional accountants engaged in
established by law. which of the following?
d. The responsible party may also be one of the intended a. Accounting education
users. b. Public accounting
c. Management accounting
81. The network firms are required to be independent of the client d. Government accounting

AT – Prelim Rev (875 MCQs) Red Sirug Page 69 of 77


promotes, or may be perceived to promote, an assurance
88. Regarding practice of accountancy, which of the following client's position or opinion to the point that objectivity may,
certificates is issued to a CPA first, a certificate of registration or may be perceived to be, compromised.
or a certificate of accreditation?
a. Certificate of registration 94. Eman, a CPA, has a law practice. Eman has recommended
b. Certificate of accreditation one of his clients to Noel, a CPA. Noel has agreed to pay
c. Both are issued at the same time Eman 10% of the fee for services rendered by Noel to Eman's
d. Neither are issued to CPAs. client. Who, if anyone, is in violation of the Code of Ethics?
a. Both Eman and Noel
89. Examples of circumstances that may create self-interest threat b. Eman only
include: c. Neither Eman and Noel
a. Contingent fees relating to assurance engagements. d. Noel only
b. A direct financial interest or material indirect financial
interest in an assurance client. 95. If a member of the assurance team, or their immediate family
c. A loan or guarantee to or from an assurance client or any member receives, by way of, for example, an inheritance, gift
of its directors or officers. or, as a result of a merger, a direct financial interest or a
d. All of the above material indirect financial interest in the assurance client, a
self-interest threat would be created. The following safeguards
90. Which of the following least likely create "self-interest threat"? should be applied to eliminate the threat or reduce it to an
a. Undue dependence on total fees from an assurance acceptable level:
client. a. Disposing of the financial interest at the earliest practical
b. Concern about the possibility of losing the engagement. date.
c. Having a close business relationship with an assurance b. Removing the member of the assurance team from the
client. assurance engagement.
d. Pressure to reduce inappropriately the extent of work c. Either a or b
performed in order to reduce fees d. Neither a nor b

91. The integrated national professional organization of Certified 96. Occurs when, by virtue of a close relationship with an
Public Accountants accredited by the BOA and the PRC per assurance client, its directors, officers or employees, a firm or
PRC accreditation No. 15 dated October 2, 1975. a member of the assurance team becomes too sympathetic
a. Philippine Institute of Certified Public Accountants to the client's interests.
(PICPA) a. Self-interest threat
b. Auditing and Assurance Standards Council (AASC) b. Self-review threat
c. Financial Reporting Standards Council (FRSC) c. Advocacy threat
d. Education Technical Council (ETC) d. Familiarity threat

92. A CPA-lawyer, acting as a legal counsel to one of his audit 97. A former officer, director or employee of the assurance client
client, is an example of serves as a member of the assurance team. This situation will
a. Self-interest threat least likely create
b. Self-review threat a. Self-interest threat.
c. Advocacy threat b. Self-review threat.
d. Familiarity threat c. Intimidation threat.
d. Familiarity threat.
93. Intimidation threat
a. Is not a threat to independence. 98. Immediate family includes
b. Occurs when a member of the assurance team may a. Parent.
be deterred from acting objectively and exercising b. Sibling.
professional skepticism by threats, actual or c. Non-dependent child.
perceived, from the directors, officers or employees d. Spouse.
of an assurance client.
c. Occurs when, by virtue of a close relationship with an 99. Which of the following is not likely a threat to independence?
assurance client, its directors, officers or employees, a a. Acting as an advocate on behalf of an assurance client in
firm or a member of the assurance team becomes too litigation or in resolving disputes with third parties.
sympathetic to the client's interests. b. Long association of a senior member of the assurance
d. Occurs when a firm, or a member of the assurance team, team with the assurance client.

AT – Prelim Rev (875 MCQs) Red Sirug Page 70 of 77


c. Threat of replacement over a disagreement with the examining a sample of evidence.
application of an accounting principle. c. The work, under taken by the auditor is permeated by
d. Owning immaterial indirect financial interest in an judgment.
audit client. d. Fatigue and human weaknesses can cause
auditors to overlook pertinent evidence.
SET 9
7. Which of the following is not one of the general principles
1. This is the traditional service provided by CPA firms. governing the audit of financial statements?
a. Tax services a. The auditor should conduct the audit in accordance with
b. Compilation PSA.
c. Reviews b. The auditor should comply with the Philippine Code of
d. Audits Professional Ethics.
c. The auditor should plan and perform the audit with an
2. The billing arrangement based on actual time spent by audit attitude of professional skepticism.
engagement team is d. The auditor should obtain sufficient appropriate
a. Flat sum basis evidence primarily through inquiry and analytical
b. Per diem basis procedures to be able to draw reasonable conclusions.
c. Maximum fee basis
d. Retainer basis 8. Which of the following statements does not properly describe
an element of the theoretical framework of auditing?
3. Professionals are expected to conduct themselves at a higher a. An audit benefits the public.
level than b. The data to be audited can be verified.
a. Anyone else. c. Auditors act on behalf of management.
b. Their clients. d. Short-term conflicts may exist between managers who
c. Most other members of society. prepare data and auditors who examine the data.
d. Most other members of their profession.
9. Which of the following criteria is unique to the independent
4. A candidate who fails in two (2) complete CPA examinations auditor's attest function?
shall be: a. Independence.
a. Reverted to undergraduate status. b. General competence.
b. Forever banned from taking another set of examinations c. Due professional care.
c. Considered as having passed the examination, under the d. Familiarity with the particular industry of each client.
rule of efforts and professional mercy.
d. Required to take a refresher course before making 10. Auditing is based on the assumption that financial data are
another attempt at the CPA Board exams. verifiable. Data are verifiable when two or more qualified
individuals,
5. Which of the following statements is correct concerning an a. Working together, can prove, beyond doubt, the
auditor's responsibilities regarding financial statements? accuracy of the data.
a. The auditor's report should provide an assurance as to the b. Working independently, can prove, beyond
future viability of the entity. reasonable doubt, the truthfulness of the data.
b. Making suggestions that are adopted about the form c. Working independently, each reach essentially
and content of an entity's financial statements impairs an similar conclusions.
auditor's independence. d. Working together, can agree upon the accuracy of the
c. An auditor's responsibilities for audited financial data.
statements are confined to the expression of the
auditor's opinion. 11. The QRC shall have the following functions:
d. The fair presentation of audited financial statements in a. Conduct quality control review on applicants for
accordance with an applicable financial reporting registration to practice public accountancy and
framework is an implicit part of the auditor's render a report on such quality review.
responsibilities. b. Mete out sentences of imprisonment to CPAs who fail to
present a valid system of quality control.
6. Which of the following statements does not properly c. Revoke the certificate of registration and professional ID
describe a limitation of an audit? of an individual CPA, firm, or partnership of CPAs who
a. Many financial statement assertions cannot be audited. have not observed quality control measures and who has
b. Many audit conclusions are made on the basis of

AT – Prelim Rev (875 MCQs) Red Sirug Page 71 of 77


not complied with the standards of quality prescribed for
the practice of public accountancy. 18. In financial statement audits, the audit process should be
d. All of these. conducted in accordance with
a. The audit program
12. The need for independent audits of financial statements can b. Philippine Accounting Standards
be attributed to all of the following conditions except: c. Philippine Standards on Auditing
a. Validity d. Philippine Financial Reporting Standards
b. Remoteness
c. Consequence 19. The overall objective of forensic auditing is to
d. Complexity of subject matter a. Attest to the efficiency with which resources are
employed
13. The underlying conditions that create demand by users for b. Assist members of the organization in the effective
reliable information include the following except: discharge of their responsibilities
a. Decisions are not time-sensitive. c. Unearth the truth and provide evidence in legal and
b. Transactions that are numerous and complex. financial disputes
c. Users separated from accounting records by distance d. Provide assurance that financial data have been
and time. accurately recorded
d. Financial decisions that are important to investors and
users. 20. Which of the following types of auditing is performed most
commonly by CPA's on a contractual basis?
14. Why does a company choose to have an independent a. Internal auditing
auditor report on its financial statements? b. Income tax auditing
a. Independent auditors will always detect management c. External auditing
fraud. d. Government auditing
b. Independent auditors guarantee the accuracy of the
financial statements. 21. Which of the following types of audits is performed to
c. The company preparing the statements may have a determine whether an entity's financial statements are fairly
vested interest in reporting certain results. stated in conformity with generally accepted accounting
d. An independent audit is designed to search for principles?
deficiencies in the company's internal controls. a. Operational audit
b. Compliance audit
15. Which of the following statements does not describe a c. Performance audit
condition that creates a demand for auditing? d. Financial statement audit
a. Users can directly assess the quality of information.
b. Information can have substantial economic 22. Which of the following has the primary responsibility for the
consequences for a decision maker. fairness of the representations made in the financial statements?
c. Expertise is often required for information preparation a. Audit committee
and verification. b. Client's management
d. Conflict between an information preparer and a user can c. Independent auditor
result in biased information. d. Board of Accountancy

16. Information risk refers to the risk that 23. The best statement of the responsibility of the auditor with
a. The client may not be able to remain in business. respect to audited financial statement is:
b. Errors and frauds would not be detected by the auditor's a. The auditor's responsibility on fair presentation of
procedures. financial statements is limited only up to the date
c. The auditor may express an unqualified opinion on of the audit report.
financial statements that are material misstated. b. The auditor's responsibility is confined to the
d. The client's financial statements may be materially expression of opinion on the financial statements
false and misleading. audited.
c. The responsibility over the financial statements rests with
17. Which of the following is a cause of information risk? the management and the auditor assumes responsibility
a. Voluminous data. with respect to the notes of financial statements.
b. Biases and motives of the provider of information. d. The auditor is responsible only to his qualified opinion but
c. Remoteness of the information. not for any other type of opinion.
d. Each of the above is a cause of information risk.

AT – Prelim Rev (875 MCQs) Red Sirug Page 72 of 77


24. The level of assurance provided by a professional b. Financial statements
accountant on an audit report is: c. Assertions about economic actions and events
a. Low d. Operating data
b. Reasonable
c. Moderate 31. Auditing is based on the assumption that financial data and
d. None statements are
a. Verifiable
25. Reasonable assurance means: b. In conformity with the identified applicable financial
a. Gathering of all available corroborating evidence for the reporting framework
auditor to conclude that there are no material c. Presented fairly
misstatements in the financial statements, taken as a d. Consistently applied
whole.
b. Gathering of the audit evidence necessary for the auditor 32. The firm should establish a system of quality control designed
to conclude that the financial statements, taken as a to provide it with reasonable assurance regarding:
whole, are free from any misstatements. a. Compliance with professional standards
c. Gathering of the audit evidence necessary for the auditor b. Compliance with regulatory and legal requirements
to conclude that the financial statements are free of c. Appropriateness of reports issued by the firm or
material unintentional misstatements. engagement partners
d. Gathering of the audit evidence necessary for the d. All of the above
auditor to conclude that there are no material
misstatements in the financial statements, taken as a 33. A process designed to provide an objective evaluation, before
whole. the report is issued, of the significant judgments the
engagement team made and the conclusions they reached in
26. Which of the following is one of the limitations of an audit? formulating the report.
a. Nature of evidence obtained a. Peer review
b. Inadequacy of the accounting records b. Inspection
c. Confidentiality of information c. Engagement quality control review
d. Scope limitations imposed by the entity d. Monitoring

27. Which of the following statements does not properly 34. A process comprising an ongoing consideration and
describe a limitation of an audit? evaluation of the firm's system of quality control, including a
a. Many audit conclusions are made on the basis of periodic inspection of a selection of completed engagements,
examining a sample of evidence. designed to enable the firm to obtain reasonable assurance
b. The work, under taken by the auditor is permeated by that its system of quality control is operating effectively.
judgment. a. Inspection
c. Fatigue and human weaknesses can cause b. Monitoring
auditors to overlook pertinent evidence. c. Peer review
d. Many financial statement assertions cannot be audited. d. Engagement quality control review

28. Financial statements audits: 35. The firm should obtain written confirmation of compliance with
a. Reduce the cost of capital its policies and procedures on independence from all firm
b. Report on compliance with laws and regulations personnel required to be independent by the Code of Ethics at
c. Assess management's efficiency least:
d. Overlook information risk a. Annually
b. Every 3 years
29. In relation to auditing, which of the following is an incorrect c. Every 2 years
phrase? d. Every 5 years
a. Auditing is a systematic process.
b. Auditing subjectively obtains and evaluates evidence. 36. The firm should obtain acceptance and continuance
c. Auditing evaluates evidence regarding assertions. information as it considers necessary in the following
d. Auditing communicates results to interested users. circumstances:
a. When accepting an engagement with a new client
30. Broadly defined, the subject matter of any audit consists b. When deciding whether to continue an existing
of engagement
a. Economic data

AT – Prelim Rev (875 MCQs) Red Sirug Page 73 of 77


c. When accepting a new engagement with an existing b. Communicating with the prospective client's prior auditor
client to inquire about any disagreements with the client.
d. All of these. c. Understanding the client's reasons for obtaining an audit.
d. Determining the likelihood of issuing an unqualified
37. The following methods are most likely to develop capabilities audit opinion on the client's financial statements.
and competence, except:
a. Professional education. 44. Which of the following is incorrect regarding professional
b. Continuing professional development, including training. competence?
c. Work experience and coaching by less experienced a. Professional accountants may portray themselves as
staff having expertise or experience they do not possess.
d. Self-study modules on professional accounting and b. Professional competence may be divided into two
auditing literature. separate phases.
c. The attainment of professional competence requires
38. Consultation is usually undertaken for: initially a high standard of general education.
a. Difficult or contentious matters. d. The maintenance of professional competence requires a
b. Matters for which there is readily available support. continuing awareness of development in the accountancy
c. Matters for which the judgment of the engagement leader profession.
is deemed sufficient.
d. Straightforward matters involving accounting and auditing. 45. The following procedures relate to Skills and Competence,
except
39. The standard-setting body whose chairman had been or a. Identify criteria which will be considered in evaluating
presently a senior accounting practitioner in any scope of individual performance and expected proficiency.
accounting practice is b. Provide procedures for maintaining the firm's
a. AASC standards of quality for the work performed.
b. FRSC c. Establish qualifications and guidelines for evaluating
c. PICPA potential hirees at each professional level.
d. ACPAPP d. Provide, to the extent necessary, programs to fill the
firm's needs for personnel with expertise in specialized
40. The chairman and the members of the AASC and FRSC shall and industries.
have a term of
a. 1 year. 46. It is essential that users regard CPA firms as
b. 3 years. a. Competent.
c. 3 years with no renewal. b. Unbiased.
d. 3 years renewable for another term. c. Technically proficient.
d. All of the above
41. The benefits of an operational audit generally include all of
the following except 47. The Code of Professional Ethics states, in part, that a CPA
a. Decreased costs. should maintain integrity and objectivity. Objectivity refers to
b. Increased revenue the CPA's ability to
c. Increased productivity. a. Determine accounting practices that were consistently
d. Increased reliability of the financial statements. applied.
b. Maintain an impartial attitude on all matters which
42. The auditor is required to comply with all PSAs relevant to the come under his review.
audit of an entity's financial statements. A PSA is relevant to c. Determine the materiality of items.
the audit when: d. Insist on all matters regarding audit procedures.
I. The PSA is in effect.
II. The circumstances addressed by the PSA exist. 48. A professional accountant may be associated with a tax return
a. I only that
b. II only a. Contains a false or misleading statement.
c. Either I or II b. Contains statements or information furnished recklessly
d. Both I and II or without any real knowledge of whether they are true or
false.
43. Preplanning the audit involves several key activities which c. Omits or obscures information required to be submitted
include the following except: and such omission or obscurity would mislead the
a. Investigating the client's background. revenue authorities.

AT – Prelim Rev (875 MCQs) Red Sirug Page 74 of 77


d. Uses of estimates if such use is generally acceptable b. From the predecessor auditor.
or if it is impractical under the circumstances to c. From the permanent file.
obtain exact data. d. At the client's premises.

49. Prior to beginning the field work on a new audit engagement in 54. When the continuing auditor intends to use information about
which a CPA does not possess expertise in the industry in the entity and its environment obtained in prior periods, the
which the client operates, the CPA should auditor should:
a. Reduce audit risk by lowering the preliminary levels of a. Assess control risk as "high" for the assertions affected
materiality. by the prior-period information.
b. Design special substantive tests to compensate for the b. Seek permission with the client in using the prior period
lack of industry expertise. information obtained by the auditor.
c. Engage financial experts familiar with the nature of the c. Determine whether changes have occurred that may
industry. affect the relevance of such information in the
d. Obtain a knowledge of matters that relates to the current audit.
nature of the entity's business. d. Determine whether to equitably reduce the audit fee due
to lower audit effort expended during the engagement.
50. Which of the following is not one of the duties of the
Commission on Audit 55. Relevant industry conditions include the following, except:
a. Keep the general accounts of the government a. The market and competition, including demand, capacity
b. Promulgate accounting rules and regulations and price competition.
c. Define the scope of its audit and examination b. Regulatory framework for a regulated industry.
d. Assume fiscal responsibility for the government and c. Cyclical or seasonal activity.
its instrumentalities d. Product technology relating to the entity's products.

51. The early appointment of the independent auditor has many 56. An auditor obtains an understanding of the entity and its
advantages to the auditor and the client. Which of the environment in order to
following advantages is least likely to occur as a result of early a. Make constructive suggestions concerning improvements
appointment of the auditor? to the client's internal control.
a. The auditor will be able to plan the audit work so that it b. Understand the events and transactions that may
may be done expeditiously. have an effect on the client's financial statements.
b. The auditor will be able to complete the audit work in c. Evaluate whether the aggregation of known
less time. misstatements causes the financial statements taken as a
c. The auditor will be able to better plan for the observation whole to be materially misstated.
of the physical inventories. d. Develop an attitude of professional skepticism concerning
d. The auditor will be able to perform the examination more management's financial statement assertions.
efficiently and will be finished at an early date after the
year-end. 57. Maintaining or providing access to adequate reference
libraries and other authoritative sources is a procedure that is
52. Which of the following is the most likely first step an auditor most likely performed to comply with the policy of
would perform at the beginning of an initial audit engagement? a. Monitoring
a. Prepare a rough draft of the financial statements and of b. Ethical requirements
the auditor's report. c. Assignment
b. Study and evaluate the system of internal administrative d. Consultation
control.
c. Tour the client's facilities and review the general 58. A process designed to provide an objective evaluation, before
records. the report is issued, of the significant judgments the
d. Consult with and review the work of the predecessor engagement team made and the conclusions they reached in
auditor prior to discussing the engagement with the client formulating the report is called:
management. a. Inspection
b. Documentation
53. An extensive understanding of the client's business and c. Monitoring
industry and knowledge about the company's operations are d. Engagement quality control review
essential for doing an adequate audit. For a new client, most
of this information is obtained
a. From the Securities and Exchange Commission.

AT – Prelim Rev (875 MCQs) Red Sirug Page 75 of 77


59. A CPA firm would be reasonably assured of meeting its 64. The element of the audit planning process most likely to be
responsibility to provide professional services that conform agreed upon with the client before implementation of the audit
with professional standards by: strategy is the determination of the
a. Maintaining an attitude of independence in its a. Methods of statistical sampling to be used in confirming
engagements. accounts receivable.
b. Adherence to generally accepted auditing standards. b. Pending legal matters to be included in the inquiry of the
c. Having an appropriate a system of quality control. client's attorney.
d. Continuing professional education. c. Evidence to be gathered to provide a sufficient basis for
the auditor's opinion.
60. At the beginning of the current audit engagement, the auditor d. Schedules and analyses to be prepared by the
should perform key preplanning activities which include all of client's staff.
the following except:
a. Performing procedures regarding client acceptance or 65. The understanding between the client and the auditor as to
continuance. the degree of responsibilities to be assumed by each is
b. Evaluating compliance with ethical requirements. normally set forth in a(an)
c. Establishing an understanding of the terms of the a. Management letter.
engagement. b. Representation letter.
d. Considering internal control. c. Engagement letter.
d. Comfort letter.
61. When a CPA is approached to perform an audit for the first
time, the CPA should make inquiries of the predecessor 66. Which of the following best describes the purpose of the
auditor. This is a necessary procedure because the engagement letter?
predecessor may be able to provide the successor with a. The engagement letter conveys to management the
information that will assist the successor in determining detailed steps to be applied in the audit process.
a. Whether, in the predecessor's opinion, internal control of b. The engagement letter relieves the auditor of some
the company has been satisfactory. responsibility for the exercise of due care.
b. Whether the engagement should be accepted. c. By clearly defining the nature of the engagement, the
c. Whether the predecessor's work should be utilized. engagement letter helps avoid and resolve
d. Whether the company follows the policy of rotating its misunderstandings between CPA and client
auditors. regarding the precise nature of the work to be
performed and the type of report to be issued.
62. Which of the following will an auditor least likely discuss with d. The engagement letter should be signed by both the
the former auditors of a potential client prior to acceptance? client and the CPA and should be used only for
a. Disagreements with management regarding accounting independent audits.
principles
b. Integrity of management 67. Which of the following statements would least likely appear in
c. Fees charged for services an auditor's engagement letter?
d. Reasons for changing audit firms a. Our engagement is subject to the risk that material errors,
fraud, and defalcations, if they exist, will not be detected.
63. Which of the following statements are factors to consider b. During the course of our audit we may observe
whether the firm has the capabilities, competence, time and opportunities for economy in, or improved controls over,
resources to undertake a new audit engagement from a your operations.
prospective client: c. After performing our preliminary analytical
I. Firm personnel have knowledge of relevant procedures we will discuss with you the other
industries. procedures we consider necessary to complete the
II. The firm has sufficient personnel with the necessary engagement.
capabilities and competence. d. Fees for our services are based on our regular per diem
III. Experts are available, if needed. rates, plus travel and other out-of-pocket expenses.
IV. The firm is able to complete the engagement within
the reporting deadline. 68. Which of the following is an element of "directing an audit
a. I and II only assistant" objective?
b. II and IV only. a. Identifying in advance the staffing requirements of a
c. II, III and IV only particular audit engagement.
d. I, II, III and IV b. Resolving any differences in professional judgment
between audit personnel.

AT – Prelim Rev (875 MCQs) Red Sirug Page 76 of 77


c. Resolution of differences in audit findings. examination at the time of his appointment to the Board.
d. Informing assistants of their responsibilities and the
objectives of the procedures they are to perform. 73. The following statements relate to the RA 9298. Which
statement is true?
69. A CPA establishes quality control policies and procedures for a. The Professional Regulation Commission has the
deciding whether to accept a new client or continue to perform authority to remove any member of the Board of
services for a current client. The primary purpose for Accountancy for negligence, incompetence, or any other
establishing such policies and procedures is just cause.
a. To enable the auditor to attest to the integrity or reliability b. No person shall be appointed as a member of the
of a client. Board of Accountancy unless he has been in the
b. To comply with the quality control standards established practice of accountancy for at least 10 years, among
by regulatory bodies. others.
c. To minimize the likelihood of association with c. Insanity is not a ground for proceeding against a CPA.
clients whose management lacks integrity. d. After three years, subject to certain conditions, the Board
d. To lessen the exposure to litigation resulting from failure of Accountancy may order the reinstatement of a CPA
to detect irregularities in client financial statements. whose certificate of registration has been revoked.

70. Which of the following is prohibited by the Code of 74. Special/temporary permit may be issued by the Board to the
Professional Ethics for CPAs? following persons except:
a. Use of a firm name which includes the name of a retired a. A foreign CPA engaged as a professor or lecturer in the
partner. fields essential to accountancy education in the
b. Accepting an engagement or employment which one Philippines.
cannot reasonably expect to complete or discharge b. A foreign CPA called for consultation or specific purpose
with professional competence. which is essential for the development of the country and
c. Announcement in a newspaper of the opening of a public that there are no Filipino CPAs qualified for such
accounting office. consultation or specific purpose.
d. Engaging in civic activities during business hours. c. A foreign CPA who can prove that the country of
which he or she is a citizen admits citizens of the
71. Which of the following statements best describes why the Philippines to the practice of the same profession
profession of certified public accountants has deemed it without restriction.
essential to promulgate a code of ethics and to establish a d. A foreign CPA with specialization in any branch of
mechanism for enforcing observance of the code? accountancy and his/her service is essential for the
a. The law requires an establishment of a code of ethics. advancement of accountancy in the Philippines.
b. A distinguishing mark of a profession is its
acceptance of responsibility to the public. 75. The following statements relate to RA 9298. Which statement
c. A prerequisite to success is the establishment of an is true?
ethical code that stresses primarily the professional's a. Insanity is not a ground for proceeding against a CPA.
responsibility to clients and colleagues. b. The Professional Regulation Commission has the
d. An essential means of self-protection for the profession is authority to remove any member of the Board of
the establishment of flexible ethical standards by the Accountancy for negligence, incompetence, or any other
profession. just cause.
c. A person shall be considered to be in the
72. Which of the following is not one of the qualifications of the professional practice of accounting if, as an officer in
members of the Board of Accountancy? a private enterprise, he makes decisions requiring
a. He/She must not be a director or officer of PICPA at the professional accounting knowledge.
time of his appointment. d. After three years, subject to certain conditions, the Board
b. He/She must be a natural-born citizen and a resident of of Accountancy may order the reinstatement of a CPA
the Philippines. whose certificate of registration has been revoked.
c. He/She must be a duly Certified Public Accountant
with at least ten years of experience in practice of
public accountancy.
d. He/She must not have any direct or indirect pecuniary
interest in any school, college, university, or institution
offering a BS Accountancy course or institution
conducting review classes in preparation for the licensure

AT – Prelim Rev (875 MCQs) Red Sirug Page 77 of 77

Das könnte Ihnen auch gefallen